Elementary Analysis Through Examples and Exercises
Elementary Analysis Through Examples and Exercises
The titles published in this series are listed at the end of this volume.
Elementary Analysis
through Examples
and Exercises
by
John Schmeelk
Department of Mathematical Sciences,
Virginia Commonwealth University,
Richmond, Virginia, U.S.A.
Djurdjica Takaci
and
Arpad Takaci
Institute of Mathematics,
University ofNo vi Sad,
Novi Sad, Yugoslavia
Preface VII
1 Real numbers 1
1.1 (R, +, .,:::;) as a complete totally ordered field 1
1.1.1 Basic notions . . . . . . 1
1.1.2 Examples and exercises. 5
1.2 Cuts in Q . . . . . . . . . . . . 29
1.2.1 Basic notions . . . . . . 29
1.2.2 Examples and exercises. 31
1.3 The set R as a topological space . 37
1.3.1 Basic notions . . . . . . 37
1.3.2 Examples and exercises .. 39
2 Functions 45
2.1 Real functions of one real variable 45
2.1.1 Basic notions . . . . . . . 45
2.1.2 Examples and exercises .. . 49
2.2 Polynomials, rational and irrational functions 72
2.2.1 Basic notions . . . . . . 72
2.2.2 Examples and exercises. 75
3 Sequences 83
3.1 Introduction 83
3.1.1 Basic notions . . . . . . 83
3.1.2 Examples and exercises. 85
3.2 Monotone sequences . . . . . . 102
3.2.1 Basic notions . . . . . . · 102
3.2.2 Examples and exercises. · 102
3.3 Accumulation points and subsequences · 116
3.3.1 Basic notions . . . . . . 116
3.3.2 Examples and exercises. · 117
3.4 Asymptotic relations . . . . . . · 135
3.4.1 Basic notions . . . . . . · 135
3.4.2 Examples and exercises. · 136
v
VI
5 Continuity 181
5.1 Continuity at a point . . . . .. · 181
5.1.1 Basic notions ..... . · 181
5.1.2 Examples and Exercises · 183
5.2 Uniform continuity . . . . . . . .204
5.2.1 Basic notions ..... . .204
5.2.2 Examples and exercises. .205
6 Derivatives 213
6.1 Introduction . . . . . . . . . . . · 213
6.1.1 Basic notions . . . . . . · 213
6.1.2 Examples and exercises. · 217
6.2 Mean value theorems . . . . . . .244
6.2.1 Basic notions . . . . . . .244
6.2.2 Examples and exercises. .245
6.3 Taylor's formula. . . . . . . . . .252
6.3.1 Basic notions . . . . . . .252
6.3.2 Examples and exercises. · 253
6.4 L'Hospital's Rule . . . . . . . . .262
6.4.1 Basic notions . . . . .. .262
6.4.2 Examples and Exercises .263
6.5 Local extrema and monotonicity of functions .270
6.5.1 Basic notions . . . . . . .270
6.5.2 Examples and exercises. · 271
6.6 Concavity . . . . . . . . . . . . .279
6.6.1 Basic notions . . . . . . .279
6.6.2 Examples and exercises. .280
Bibliography 313
Index 315
Preface
It is hard to imagine that another elementary analysis book would contain ma-
terial that in some vision could qualify as being new and needed for a discipline
already abundantly endowed with literature. However, to understand analysis, be-
ginning with the undergraduate calculus student through the sophisticated math-
ematically maturing graduate student, the need for examples and exercises seems
to be a constant ingredient to foster deeper mathematical understanding. To a
talented mathematical student, many elementary concepts seem clear on their first
encounter.
However, it is the belief of the authors, this understanding can be deepened with a
guided set of exercises leading from the so called "elementary" to the somewhat more
"advanced" form. Insight is instilled into the material which can be drawn upon and
implemented in later development. The first year graduate student attempting to
enter into a research environment begins to search for some original unsolved area
within the mathematical literature.
It is hard for the student to imagine that in many circumstances the advanced
mathematical formulations of sophisticated problems require attacks that draw upon,
what might be termed elementary techniques. However, if a student has been guided
through a serious repertoire of examples and exercises, he/she should certainly see
connections whenever they are encountered.
The seven chapters in this book contain, in the authors' opinion, a wide variety of
problems, exercises and examples implemented by many instructors. The book can
be used both by complete beginners in analysis, as well as by students that already
have gone successfully through some calculus courses. The presented material is
self-contained and the exposition is mostly deductive. Occasionally some notions
are used before their formal definition, though, presumably, they will usually be
known to the reader.
The content is in fact elementary but the strategy employed is to navigate
through a wide assortment of problems connected to the nature of the chapter. A
minimal amount of expository discussion is included at the start of each new section
with a maximum emphasis placed on well selected examples and exercises capturing
the essence of the material. It is the intention of the authors to have students as-
semble a very valuable collection of well-thought-out problems. We have separated
the problems into examples and exercises. The examples contain a complete solu-
tion. In the exercises students are left to solve the problem. Often, answers only are
provided. One should note that the included exercises in some cases are generally
VII
viii PREFACE
found in very advanced texts when they are needed to prove a more profound result.
We feel that the students should study some of the pertinent advanced exercises in
the early developmental stages, so that they can enhance their mathematical skills.
Chapter 1 introduces the field of real numbers from the traditional axiomatic
presentation, as well as the R. Dedekind construction presented in 1872. Comparing
the two methods is, in our opinion, vital to understanding new concepts. Contrast
oftentimes invites the student to see why and where things differ and also where
things go wrong. The induction principle is also included with a very complete set
of examples and exercises implementing the process varying from the traditional
problems to what we term the somewhat unusual problems.
Chapter 2 introduces the concepts of relations and functions. It includes a com-
prehensive review of the basic ideas and terminology implemented to express the
different concepts. Various examples presented are oftentimes not included in a
very elementary text, like, for instance, the Dirichlet function. Also included is the
connection between rational functions and their representation using partial fraction
decomposition.
Chapter 3 introduces the concept of a sequence of real numbers. The traditional
theory surrounding sequences is presented such as limits, monotonicity and Cauchy
sequences. However, it goes on to identify asymptotic behavior for sequences as
well as proving some important estimates on the number e, which is the basis of
what is probably the most important function in analysis, namely the exponential
function. The discrete case is often drawn into the continuous case by examining
such functions as the logarithmic, f(x) = In x, x > 0, and showing its connection to
the discrete version, fn = In n, n E N. Furthermore, iterative schemes are examined
and how they can be implemented to generate converging sequences. Again the
Landau notation commonly called "big oh" and "small oh" are included, showing
how their asymptotic behavior definition can be reflected into sequences.
Chapter 4 introduces classical notion of limit to include the left and right limits.
Special emphasis is placed on their connections to points of accumulation. Several
examples illustrate the change of variable technique and how this process fosters
good arithmetic behavior, so that a precise limit can be computed. The line asymp-
totes for the graphs of the functions are given as the geometrical application of
limits. The Landau notation is again revisited and several examples and exercises
show the students how these relationships control asymptotic behaviors.
Chapter 5 presents the classical notion of continuity and its relationship to points
and points of accumulation. Left-side and right-side continuity, uniform continuity
and theorems relating these properties are presented in a traditional manner. The
delta-epsilon techniques together with their quantifiers are discussed in detail to-
gether with accompanying examples and exercises. Several somewhat unusual exam-
ples are presented to illustrate connections to the notion of the order of discontinuity
at a point.
Chapter 6 introduces the derivative using a detailed discussion involving the left
and right-hand derivatives. The traditional results regarding derivatives and their
geometrical interpretations are included together with a rich assortment of examples
and exercises. It continues to develop differentials and their connections to deriva-
PREFACE ix
tives and the derivative remainders. Again examples illustrating these notions and
calculated for some rather sophisticated functions are given. This chapter devel-
ops many applications for derivatives. The traditional results are included with
several applications not necessarily found in many elementary texts. It identifies
and proves the Laguerre, Hermite and Chebychev polynomials normally termed the
special functions in mathematical physics have real roots. Several inequalities are
proven using the mean value theorem for the differential calculus. The Taylor and
Maclaurin formulas are included with a generous amount of examples and exercises.
Of course, all of the standard information regarding preliminary curve sketching
implementing first and second order derivative tests is included, together with a
presentation of L'Hospital's rule.
Chapter 7 contains a detailed account for graphing functions. For this chapter
the asymptotes to include slanted asymptotes, concavity, (local) maximums and
minimums and points of inflection are presented again with some rather interesting
functions. Let us note here each example is endowed with the appropriate graph of
the given function. The figures were carefully produced, though they primarily serve
as an illustration of the analytically achieved results. For such methodical reasons,
the unit lengths on the x- and y-axis are in some figures nonequal.
The extensive bibliography found at the end of the book is to provide the student
with a wide range of resource material. It is also there to help indicate that this set
of examples and exercises could benefit students studying from among a very broad
spectrum of mathematical disciplines.
The origin of this book lies in several analysis and calculus courses that the
authors gave to students on various levels and with, occasionally, quite different
mathematical backgrounds. The experience we got through these lectures, is en-
dowed, we hope successfully, in the given examples and exercises. We would like to
thank many colleagues and students for their remarks, corrections, new problems
(at least for us), their original, better or more precise solutions, and some other
contribution(s) to this book. In particular, it is our pleasure to thank academician
Dr. Olga Hadzic, from the Institute of Mathematics at the University of Novi Sad,
for the careful reading of some early versions of the manuscript and her numerous
improvements in the text. The hard job of preparing the figures was done by Mr.
Milan Manojlovic.
At last but not least, we wish to express our gratitude to Kluwer Academic
Publishers for their kind and generous support throughout the production of our
manuscript. We are especially thankful to Dr. Paul Roos and Ms. Anneke Pot for
their time and effort in coordinating our work.
J. SCHMEELK
DJ. TAKACI
Novi Sad & Richmond, January 1995. A. TAKACI
Chapter 1
Real numbers
(R9) (\Ix, y E R) x y = yo x
0
1
2 CHAPTER 1.
(RI2) (\:Ix, y E R) x ~ y VY ~ x
(total ordering of the binary relation ~);
(RI3) (\:Ix, y, z E R) x ~ Y =? (x + z ~ y + z)
(compatibility of the binary relation ~ with addition);
(RI5) Let X and Y be two nonempty subsets of the set R with the property
(\:Ix E X) (\:Iy E Y) x ~ y.
(\:Ix E X) (\:Iy E Y) x ~ c~ y
From the axioms (Rl), (R2) and (R3) it follows that (R, +) is a group, while by
(R4) it becomes a commutative (or: Abelian) group. In the same manner, from the
axioms (RS), (R6), (R7) and (R9) it follows that (R \ {O},·) is also a commutative
group. The axiom (R8) gives the distributivity of multiplication over addition. The
axioms (RIO), (Rll) and (RI2) show that R is a totally ordered set with the binary
relation ~, while the axioms (RI3) and (RI4) give the connection of ::; with the
operations of addition and multiplication. The axioms (RI) - (RI4) define R as a
totally ordered field, and, finally, with (RIS) the set R becomes a complete totally
ordered field.
Fram now on we shall define b - a := b + (-a) and call the operation " - "
subtraction, define alb := a· b- 1 , b:l 0, and call the operation "I" division. Instead
of x . y we shall often write xy for multiplication.
Let x and y be two real numbers such that x ~ y. We define another binary
relation " :2: " by
(Vx,y E R) y:2: x ¢=:? x ~ y.
If both x ~ y and x :I y, we shall simply write x < y. Also, we shall define
x >y ¢=:? Y < x.
a, if a> 0;
{ if a = 0;
lal = ~~,
if a < O.
REAL NUMBERS 3
From Definition 1.4 (resp. Definition 1.5) it follows that the infimum is the greatest
lower bound (resp. the supremum is the smallest upper bound) of a set. In Example
1.50, using axiom (R15), we shall prove that every subset of R bounded from below
has an infimum, while every subset of R bounded from above has a supremum.
We shall denote by inf X, sup X, min X and max X the infimum, supremum, mini-
mum and maximum of the set X C R respectively.
Bounded intervals. Let a and b be two real numbers such that a < b. Then the
following subsets of R (a,b), [a,b], (a, b] and [a,b) are bounded intervals with
endpoints a and b :
Note that bounded (resp. unbounded) intervals are also bounded (resp. unbounded)
sets in R in the sense of Definition 1.3.
Definition 1.8. The set of natural numbers, denoted by N, is the smallest in-
ductive set containing the real number 1.
Hence N = {I, 2, 3, ... }.
Theorem 1.9. Assume a subset X of the set of natural numbers N has the following
two properties.
(mipl) 1 EX;
Then necessarily X = N.
Definition 1.10. The set of integers, denoted by Z, includes the natural numbers,
their additive inverses and the additive identity element, zero.
Hence, Z = { ... ,-2,-1,O,1,2,3, ... }.
The set of rational numbers Q satisfies the first fourteen axioms from Definition
1.1, hence endowed with addition, multiplication and the binary relation:::: it also
becomes a totally ordered field. However, the set Q does not satisfy the completeness
axiom (R15).
It is important to note that the Cantor theorem is not true in the set of rational
numbers Q.
In Example 1.51 we shall prove that axiom (RI5) is equivalent with the con-
junction of the Archimedes and the Cantor theorem. More precisely, the system of
fifteen axioms (R1) - (RI5) is equivalent to the system of fourteen axioms (Rl) -
(RI4) plus the Archimedes and the Cantor theorem.
c) for given real numbers a and b the equation a + x = b has a unique solution in
R.
Solutions.
a) From the axiom (R2) it follows only the existence of an additive identity. Suppose
there exist two such additive identities, say 01 and O2 . Then from (R2) and
(R4) it follows
01 = 01 + O2 = O2 + 01 = O2 .
c) Using axioms (R4), (Rl) , (R3) and (R2), let us check that the number x :=
b + (-a) is a solution of the given equation:
Example 1.15. Using the axioms (R1), (R2), ... , (R14) and Example 1.14 show
that for every x, y, x', y' E R it holds
a) X· 0 = 0; b) -x = (-1) . X;
c) -(-x) = X; d) x( -y) = -(xy) = -(x)y;
e) (-x)(-y) = xy; f) X s:; 0 -¢::::::} - X 2 0;
g) X s:; y -¢::::::} -x 2 -y; h) (x < Ol\.y < 0) =} xy > 0;
i) (x s:; Y I\. x' s:; y') =} X +x' s:; y + y'; j) (x < 0 I\. Y > 0) =} xy < 0;
k) 0<1; 1) x > 0 -¢::::::} x-I> O.
Solutions.
x + x ·0 = x . 1 + x . 0 = x . (1 + 0) = x . 1 = x,
hence x + x . 0 = x. Now from Example 1.14 c) it follows that x . 0 is also an
additive identity; from Example 1.14. a) then it follows x . 0 = o.
x + (-1) . x = (1 + (-1)) . x = 0 . x = x . 0 = O.
Thus (-1) . x is also an opposite number to x; from Example 1.14.b) then it
follows (-1)· x = -x.
c) By the definition of the element -( -x) we can write (-x) + (-( -x)) = 0, while
by the definition of the opposite element -x it holds (-x) + x = O. Using the
uniqueness of the solution of the equation a + x = b (see Example 1.14 c)) we
obtain the statement.
(x < OAy < O)::::} (-x> OA -y > O)::::} (-y)(-x) > 0 <===} xy> O.
i) Since x :s: y ::::} x + x' :s: x' + y and x' :s: y' ::::} x' + y :s: y' + y, from (RIO) and
(R4) the statement follows.
j) (x < 0 A 0 < y) ::::} (0 < -x A 0 < y) ::::} (0 < (-x)y) ::::} (0 < (( -l)x)y)
::::} (0 < (-l)(xy))::::} (0 < -(xy))::::} (xy < 0).
k) The axiom (R6) implies 1 of. 0, hence by (R12) it is either 1 > 0 or 1 < O. Suppose
1 < 0; then from h) and 1 of. 0 it holds (1 < 0 Al < 0) ::::} (0 < 1·1) ::::} 0 < l.
This is a contradiction, hence necessarily 1 > O.
1) First of all, X-I of. O. Suppose X-I < O. Then from j) it follows
(x- I < 0 A 0 < x) ::::} (x· X-I < 0) ::::} (1 < 0),
which is in contradiction with k).
Example 1.18. Show that for every x E R the following properties of the absolute
value hold.
a) Ixl = 0 <===} x = 0; b) 1- xl = Ixl;
c) -lxl:S: x :s: Ixl; d) Ixl:S: E <===} -E:S: x :s: E;
e) Ixl < E <===} -E < x < E.
8 CHAPTER 1.
Solutions.
b) For x = 0 this is trivial. For x < 0 it holds -x > 0 (see Example 1.15 f», hence
by Definition 1.2 Ixl = -x and I - xl = -x. Thus Ixl = 1- xl- The third case
(x > 0) is handled similarly.
The equalities c) and d) follow from Examples 1.15 d), e), h) and j).
Example 1.19. Prove that for every x, y E R the following properties of the abso-
lute value hold.
Solutions.
Hence
-Ix - yl S; Ixl-Iyl S; Ix - yl '* Ilxl- Iyll S; Ix - yl·
Exercise 1.20. Show that for every Xl, X2, ••• , Xn E R the following holds.
Ik=l
txkl:= IXI + X2 + ... + xnl S; t
k=l
IXkl·
Solutions.
REAL NUMBERS 9
a) From the definition of the absolute value, for x E (-=, -2) (~ x + 2 < 0)
the given equation can be written as -(x - 1) - (-(x + 2)) = 1, which is
equivalent to 3 = 1. Hence there is no solution in the interval (-=, -2) . For
-2 < x :S: 1 the given equation can be written as -(x - 1) - (x + 2) = 1, so
in the interval (-2, 1] the only solution is x = -1. For x > 1 the equation has
the form x-I - (x + 2) = 1, or -3 = 1, which means that no solutions exist
in the interval (1,+=).
b) Every point from the interval (~, 6) is a solution of the given inequation.
a) Ix + 31 = 5; b) Ix - ll-12x - 71 = 2;
c) Ixl- Ix - 11 + 31x - 21- 21x - 31 = x + 1.
Answers. a) x = -8 V x = 2. _-1-
b) x = 4 Vx
3·
° c) x = -1 V x:::: 3.
5X + 21> 2 3
c) 12x - 3 - , x :f. 2; d)
IX:ll>x:l'
x:f. -1;
Answers.
g) XE(-I,I)U(I,3). h) x E (-~, D.
Example 1.24. Prove that the sum of the first n, n E N, terms of a geometric
sequence with quotient q :f. 1 is given by
l: l-l =
n
qn -1 q :f. 1. (1.1)
k=l q- 1'
10 CHAPTER 1.
Hence
qm+1 - 1
Lq
m+! k
= .
k=l q- 1
Thus we obtained formula (1.1) for n = m + 1. By the mathematical induction
principle it follows that formula (1.1) holds for all natural numbers n.
Example 1.25. Prove the following formula for n EN:
lOn+1 - 9n - 10
3 + 33 + 333 + ... + 33 ... 3 = 7 (1.2)
'----... ' 2
n addends
IOn+! - 9n - 10
Solution. Let us put f(n) := 27 ' n E N. For n = 1 formula (1.2)
becomes
10 2 - 9·1 - 10
3= 27 '
which is true. Assume that (1.2) holds for some n = m. Then we have using Example
(1.24 )
10m +1 - 9m - 10
3 + 33 + 333 •+ ... + 33 ... 3' = + 33 ... 3
, 27 '--v--'
m + 1 addends m + 1 ciphers
= ~. (10 m +! _ 9m - 10 + 81 . _10...,..m..,..+_1_--,--I)
27 10-1
= _1_ . (9. 10m +! _ 81m - 90 + 81· lO m +1 - 81)
27·9
= 2/ 9 . 9 . (lO m +2 - 9( m + 1) - 10) .
Simplifying the last expression on the right-hand side gives
Example 1.26. Using the mathematical induction principle, prove the following
formulas for n E N.
n n
n(n+1).
a) Lk=
2
, b) L(2k - 1) = n 2;
k=l k=l
n
n(n + 1)(2n + 1)., n 2
= n(4n -1).,
c) Lk 2 = d) L(2k _1)2
k=l 6 k=l 3
n
n(n+1)(n+2). n 1 n
e) Lk(k+1)= , f)
k=l 3 E(2k-1)(2k+1) = 2n+1
Solutions.
1·2
a) For n = 1 the given formula reduces to 1 = -2-' which is true. If the formula is
true for n = m, mEN, then
E =E +
m+l
k
m
k (m + 1) = m(~ + 1) + (m + 1) = (m + 1)2(m + 2).
m+1 1
E . "~. m . + 1
k=l
a) En (n)2
E k3 = k , hence by Example 1.26 a) it follows
t e = (n(n+1))2.
k=l 2'
n
b) L k(k + l)(k + 2) = n(n + l)(n + 2)(n + 3)
k=l . .
12 CHAPTER 1.
Solution. a) The formula is true for n = 1. Supposing that the formula is true for
n = m, (for some mEN), let us prove it for n = m + 1. Namely, from the inductive
assumption and Example 1.26.a) it follows
m+l
{; k3
m
= {; k3 + (m + 1)3 = {; k
(m)2 + (2 (m(m+1»)
2 (m + 1) + (m + 1)2)
m)2 +2 (m)
= ( {;k {;k (m+1)+(m+1)2= ( m ) 2 = (m+l)2
(;k+(m+l) (;k
k + (k +
( n) +1
n 1) -_ (nk + 1) ' n EN, kENo, 0 ~ k ~ n- 1.
(a + bt = t
k=O
(~)an-kbk, a,b E R, n E N. (1.3)
(a + W= G)a1-Ob1-1+ G)a1-1b1-O.
Assume (1.3) is true for some n = m. Then we have using a)
(a+b)m+1 (E (7)a m
-
kbk) (a+b)
n cos -
na (n + l)a
. cos -'-------'.-
b) L cos(ka) = 2. a 2 , a ~ {27r£1£ E Z}.
k=l sm"2
. a . 2a
SIn-sm-
Solution. a) For n = 1 the given formula becomes sin a = ~ a 2 ,which is
sm-
2
true. Supposing that the formula is true for n = m, let us prove it for n = m + l.
· ma . (m+1)a
m+I SIn -2- . sm -'---2----'--
L sin(ka) . a + sin((m + l)a)
k=I sm 2
. ma )
· (m+1)a sm
sm -'-----'-- ( 2a (m+1)a
+ 2 cos --'------'--
2 sin _ 2
2
· (m+1)a
(. (m + l)a
sm 2
--~a'--- sm cos -
a
+ cos (m + l)a sm
. a)
-
sin - 2 2 2 2
2
· (m+1)a . (m+2)a
sm . sm 22
. a
sm-
2
Example 1.30. Prove the following formulas.
a)
k=I
n cos 2k = 2 SInsin-
n X x
x'
n
0 < x < 7r;
2n
Solutions.
2m
x
sin x . cos 2m +1 slnx
x . x x
2m. 2 cos - _ . sm 2m + 1 2m+1 . sin 2m +1
2m +1
b) For n = 1 we have the correct equality .J2 = 2 cos ~. Assume the formula is true
for n = m. Then
+ cos
= 2\1 1 - -- 2:+1 = 2 cos (~(_Jr_))
2 2m+1 = 2 cos 2:+ 2 '
n+1)n
c) n! < ( -2- , n> 1.
Solutions.
a) For n = 1 we have (1 + x)l ::::: 1 + 1· x, which is always true. Let us suppose now
that the inequality is true for n = m, i.e., (1 + x)m ::::: 1 + mx. We shall prove
next that it is then true for n = m + 1 and x > -1.
c) For n = 2 the given inequality 2! < (2; 1 )2 is clearly true. Assume the inequal-
ity is true for n = m, then it holds
m + l)m
(m + I)! (m + 1) . m! < (m + 1) ( -2-
2. (m;2)m+1 1
(1+--
1 )m+l.
m+l
(1 + -1)
-
m+l
m+l
1
2: 1 + (m + 1)· - - = 2,
m+l
hence
m + 2)m+1 1 (m + 2)m+1
(m + I)! < 2· ( - - .- = --
2 2 2
Example 1.32. Prove the following inequalities for n E N.
a) Xl +X2 + ... +X n 2: n, where Xk > 0, k = 1, ... ,n,
and XIX2··· Xn = 1;
1 1 1 1
b) - - + - - + ... +->_.
n +1 n +2 2n 2'
1 1 1
c) --+--+···-->1·
n+1 n+2 3n + 1 '
1 1
d) fo < 1 + v'2 + ... + fo < 2..jii, n > 1.
Solutions.
a) We shall use the mathematical induction. For n = 1 the statement is trivial.
Assume it is true for n = m, i.e., that for any m positive numbers whose
product is 1, the given inequality holds.
So let Xl, X2, ... , X m , Xm+l be m + 1 positive numbers whose product is equal
to 1. Then either all Xk-S are equal to 1, in which case the left-hand side of
the inequality becomes just m + 1, or at least one of them is less than 1, say
Xm < 1 and, of course, at least one is greater than 1, say Xm+l > 1. Then the
m numbers Xl, X2, ... , Xm-l, X m · Xm+l satisfy the inductive assumption, hence
Xl + X2 + ... + Xm + Xm+l 2: m + 1.
1
V2 < 1 + V2 < 2V2,
which is true, since 1.41 < V2 < 1.42. Assume that the formula is true for
some n = m. Then we have
111 1
1+-+ .. · + - + >vm+-.
V2 Vm vm + 1 vm + 1
Once we prove that the expression
1
vm+ ~-vm+1 (1.4)
m+1
is positive, the left-hand side of the given formula follows by the mathematical
induction principle. To that end, the expression in (1.4) is equal to
1 ~ / m( m + 1) + 1 - (m + 1) / m( m + 1) - m
vm+ -ym+1= = ,
vm + 1 vm + 1 vm + 1
which is positive.
For the right-hand side inequality in the given formula, from the inductive
assumption
1 1
1+ - + ... + - < 2vm
V2 Vm '
we have
1 1 1 1
1+-+ .. · + - + <2vm+-.
V2 Vm vm + 1 vm + 1
The right-hand side of the given formula will follow once we prove the following
inequality.
2/m(m+1)+1
vm + 1 - 2Vm + 1 < O.
In fact, we have
2/m(m+l)+1 2/m(m + 1) + 1 - 2m - 2
vm+ 1 -2vm+ 1 vm+l
v4m 2 + 4m - v4m 2 + 4m + 1
vm+l
which is clearly less than zero.
REAL NUMBERS 17
n
H:=H(XI,X2,""X n ):= 1 1 l' XI,X2,···,Xn >0,
-+-+ ... +-
Xl X2 Xn
are respectively called the arithmetic, geometric and harmonic mean of the numbers
Prove the following inequalities.
Xl, X2, ... , X n .
Solutions.
a) For n = 1 the inequality reduces to a trivial equality. For n = 2 it becomes
Xl + X2 ~ (VX1 - v'XZ)2
2 - V XIX2 = 2 ;:::: O.
Suppose now that the inequality is true for n = m > 2. Then
Xl + X2 + ... + Xm
Xl + X2 + ... + Xm + Xm+1 m
m
+ Xm+l
m+l m+l
>
m y'XI . X'2-;~ + Xm+1
m+l
From the last inequality we have
+ X2 + ... + Xm + Xm+l
m +1
Xl
- m+ijXl . X2 •.. Xm+l
m y'XI-:X;-·--;-;.1;';; + Xm+l
m+~XI . X2 ... Xm . Xm+l
m+1
= p --
- q mp (m - qp m-l - q2 pm-2 - ... - q m)
m+1
Example 1.35. Show that the sum of two natural numbers is again a natural num-
ber.
The set N (of natural numbers) is an inductive one (see Definitions 1.7 and 1.8),
which means that for every mEN it follows m +1 EN, hence 1 E N 1. Assuming
that n E N I , i.e., m+n E N for every mEN, it follows from axiom (R1) that the
number m + (n + 1) = (m + n) + 1 is also naturaL Hence, by the mathematical
induction principle Nl = N.
REAL NUMBERS 19
Example 1.36. Using the mathematical induction principle, prove the following
properties for the set of natural numbers.
a) Let n = 1. Then from the assumption it follows m > 1, hence there exists an
element pEN such that m = p + 1. This is equivalent to p = m - 1, which
means that the statement is true for n = 1. Suppose now that it is true for an
element kEN; i.e., that it holds m > k =? m - kEN. Assume m > k + 1.
Then m - k > 1, so from the (already proved) statement for n=l, it follows
b) Since by (R6) 1 EN, we have to prove that for every n E N it holds 1 ~ n. For
n = 1 this is trivial. Let us suppose that 1 ~ k for some kEN. Then from
Examples 1.15 k) and 1.15 i) it follows
(1 ~ k 1\ 0 ~ 1) =? 1 ~ k + 1.
d) Let us suppose the contrary, i.e., that there exists a nonempty set A C N which
has no minimum. From b) it follows 1 rf- A. Let us define a set B by
Exercise 1.37. Let m, n E N. Prove that there exist unique numbers q E No and
rENo, 0:::; r < n, such that m = nq + r.
Exercise 1.38. Prove that the product of natural numbers (resp. integers) lS a
natural number (resp. an integer).
Exercise 1.40. Prove that (Q, +,',:::;) is a totally ordered field, i.e., that Q satisfies
the axioms (Rl) - (R14).
Example 1.41. If a and b are two rational numbers, a < b, prove then that there
exists an element cEQ, such that a < c < b.
a a a b b b
a=-+-<-+-=c<-+-=~
2 2 2 2 2 2
(This procedure shows that actually there exist infinitely many rational numbers
between any two given rational ones.)
x2 = 2 (1.5)
Example 1.43. Prove that the equation (1.5) has a solution in the set R.
REAL NUMBERS 21
r +1 r2 - 2
r - rl = r - 2 - - = - - <0
r+2 r+2 '
which means that rl E Y, hence rl E X n Y - a contradiction, since the last
intersection is empty. The assumption r2 > 2 gives a contradiction in an analogous
way, so the third possibility remains, namely r2 = 2.
Remark. By Example 1.42, any solution x of the equation (1.5) is not a rational
number. In fact, there are two real numbers satisfying x 2 = 2, namely V2 and
-V2.
Example 1.44. Show that
Solutions.
a) Let us assume the contrary, i.e., vn = E,q where p and q are integers with largest
common divisor 1 and Iql i- 1. Then n =
pm, which implies that p and q have
qm
a common divisor different from 1, which is a contradiction.
22 CHAPTER 1.
n2<n(n+l)«n+l?,
jn + Vn = ..jb2 + b = jb(b + 1)
it follows that jn + fo is not rational.
Example 1.46. Prove that the Archimedes theorem (Theorem 1.12) holds in the
set of rational numbers, i.e.,
Solution. If a > 0 and b < 0, then we can take n = 1. So we can suppose that
ml nl
a = - , b = - , for some natural numbers ml, m2, nl and n2. Put n := m2nI + 1;
m2 n2
then
ml nl
na = (m2nl + 1) - > ml nl ~ - = b,
m2 n2
since mIn2 ~ 1.
Example 1.47. Using the Archimedes theorem in R (see Theorem 1.12), show that
. 1
a) for every x E R, x 1: 0, there exzsts an n E N such that 0 < - < Ixl;
n
c) for every x E R there exist numbers nI, n2 E N such that -nI < x < n2;
Solutions.
a) Assume x i' 0, then Ixl > 0. Thus from Theorem 1.12 it follows that for the real
numbers 1 and Ixl there exists an n E N such that nlxl > 1, or ~n < Ixl.
b) If we suppose that x > 0, then from a) it follows that there exists an n E N such
that x > ~. (Put b = 1 and a = Ixl in the Archimedes theorem.)
n
°
c) Since 1 > (see Example 1.15 k», there exists an n E N such that n· 1 > x.
Analogously, there exists an n' E N such that n' . 1 > -x or -n' < x. Putting
nI = n' and n2 = n, we get the statement.
d) Applying the Archimedes theorem, there exists a kEN such that k·l > Ixl ~ x,
hence k > x. Let ko be the largest among the numbers 0, ±1, ... , ±k which
are not greater than x. Then ko ~ x < ko + 1. Clearly, by the construction,
the integer ko is unique.
Remark. The greatest integer part of a real number x denoted by [xl is defined
to be the largest integer less or equal to x.
··
a contrad IctIon. P +-
S·mce - 1 E Q an d x p +-1 < y, £or th e ratlOna
<- . 1 numb er
n n
p+l
r we can take - - .
n
Remark. From Example 1.47 e) it immediately follows that between any two real
numbers a and b, a < b, there are infinitely many rational numbers and infinitely
many irrational numbers.
Example 1.48. 1 Let [an, bnl, n E N, be a sequence of closed intervals with the
following properties
(ii) n~oo
lim (bn - an) = 0 (the lengths of intervals tend to zero).
Prove that then there exists exactly one real number a belonging to every interval
[an, bnl·
Solution. By the Cantor theorem, the intersection n [an, bnl is not empty. Hence,
nEN
there exists a real number a which belongs to all intervals [an, bn ], i.e., an :=:; a :=:; bn
for every n E N. We have to prove that a is the only such real number. Assume
there exists another real number fJ i' a such that an :=:; fJ :=:; bn for every n EN.
Assume a < fJ, then it holds an :=:; a < fJ :=:; bn for every n E N. This implies
o < fJ - a < bn - an, a contradiction with the condition (ii). The assumption a > fJ
is handled in an analogous way.
Example 1.49. Is the Cantor theorem (Theorem 1.13) true when applied to afam-
ily of arbitrary intervals (In)nEN such that
b) Prove that if in the system of axioms (Rl) - (R15) one replaces the last one,
namely (R15), with (R15) , then the system of axioms (Rl) - (R14) and (R15) ,
becomes equivalent with the system (Rl) - (R15).
(Then we say that the axiom (R15) is equivalent with (R15)'.)
Solution.
a) We shall prove only the statement that if a nonempty set X C R is bounded
from above, then it has a supremum. Let the set Y be defined by
Y = {y E RI (Vx E X) y 2 x}.
By assumption, the set Y is non empty and it holds
(Vx E X) (Vy E Y) x:=:; y,
b) In a) we proved that the system (R1) - (R15) implies (R15)'. We have yet to
prove that if the axioms (R1) - (R14) and (R15)' hold, then the statement in
(R15) (from Definition 1.1) follows.
Assume that X and Yare two nonempty sets from R such that
The set Y is then bounded from below, hence by (R15)' the set Y has an
infimum c. By (i1) from Definition 1.4, for every y E Y it holds c ::; y. Finally,
let us show that ('<Ix E X) x::; c. Otherwise, there exists an element Xl E X
such that Xl > c. But then the number Xl is a lower bound for Y greater than
c, which is in contradiction with (i2) from Definition 1.4.
Example 1.51. Prove that the axiom (R15) is equivalent with the conjunction of
the Archimedes and the Cantor theorem (see Theorems 1.12 and 1.13).
Solution. We shall omit the proof of the statement that the system of axioms
(Rl) - (R14) plus (R15)', being equivalent to the system (R1) - (R15), implies the
Archimedes and the Cantor theorem. We shall only prove that assuming axioms
(R1) - (R14) and both the Archimedes and the Cantor theorem, every nonempty set
bounded from below has an infimum. Then by Example 1.50 b), the axiom (R15)
follows.
Assume that the nonempty set B C R is bounded from below with a number aI,
and let bl E B. Then we claim that for every mEN there exists the largest number
nm E No such that
nm
am := al +- m 2
is still a lower bound for B. In fact, if al = inf B, then nm = o. If al oj:. inf B, then
there exists nm E N such that
('<Ix E B) am = al + ~: ::; x.
If this were not true, then for some mEN there would be no such n = n m , but
n
rather it would hold al + 2m ::; X for every n E N and every X E B. But then
2: ::; X - al for every n E N, which contradicts the Archimedes theorem.
1 nm + 1
Put now bm := am + -2 m = al + ---; 2m then bm E B for every mEN. By
construction,
al ::; a. ::; ... ::; am ::; ... ::; bm ::; ... ::; b. ::; bl ,
hence
[aI, bl ] ~ [a., b.] ~ ... [am, bm] ~ ....
By the Cantor theorem, there exists a real number b, such that
Then it holds am :::; b:::; bm for every mEN. Let us show next that for every 6 > 0
there exists an mEN such that b - 6 :::; am. In the contrary, there would exist an
6> 0 such that for every mEN it holds am < b - 6. This would imply
= al + -nm nm
al + --- - +1 = a m + -21m -
am 2m < b - 6 <
- 2m 6 6.
Thus it follows
1 m 1
6 < -m
<¢:::=} 2 <-.
2 6
Using the Bernoulli inequality (see Example 1.31 a)) it would then follow
1
- > 2m = (1
6
+ l)m 2: 1 + 1 . m
for every mEN, which contradicts the Archimedes theorem.
So we obtained that for given 6 > 0 there exists an mEN such that b- 6 :::; am.
In an analogous way, one can prove that for given 6 > 0 there exists an mEN such
that b:::; bm :::; b + 6 (do that!).
Putting these conclusions together, we obtain that for given 6 > 0 there exists
an mEN such that
b - 6 :::; am :::; b :::; bm :::; b + 6. (1.10)
Finally, let us show that b = inf B. By the construction, for every x E B there exists
an element bm 2: b such that bm :::; x, hence b is a lower bound for B. Further on,
from relation (1.10) it follows that b is also the greatest lower bound for the set B.
Remark. This example shows that the Cantor theorem does not hold for the set of
rational numbers Q. Namely, as we shall see in Examples 3.33 and 3.36, the subset
X of Q defined by
X:= {(I +~) n I n EN}
is bounded from above, but has no supremum in Q. In fact, it does have a supremum
in R and it is the irrational number e.
Example 1.52. Let the set A be given by A = {~ ± 3n: 11 n EN}. Prove that
2
inf A = 0 and sup A = 3'
a contradiction.
Let us prove now that the supremum of A is ~. Firstly, ~ is an upper bound of
A, since it holds for every n E N
2 (1- ± - - >
--
3 3
n) -3(3n+l)
3n+l
1 >0
.
Next, for every e > 0 we choose no as above; then for every n > no it holds
2 1 n 2
- - e < -
3 3
+- - < -.
3n +1 - 3
2
The last inequality implies that 3" is the supremum of the set A, since it means
that for every e > 0 there exists an element x from A which belongs to the interval
( ~3 - e ~).
'3
Example 1.53. For a nonempty set X C R define
Prove that
provided that in a) (resp. in b)) X is bounded from above (resp. from below).
(Ve > 0) (3XI E X) Xl > M - e -¢=? (Ve > 0) (3XI EX) - Xl < -M + e.
Thus it follows
(Ve> 0) (3X2 E (-X)) X2 < -M +e. (1.12)
The relations (1.11) and (1.12) mean that -M is the infimum of the set -x.
Example 1.54. Let X and Y be two nonempty subsets of R bounded from below
(resp. from above). Put
S := {s = x + yl x E X, Y E Y}.
Prove that S has an infimum (resp. a supremum) and it holds
Solution. In view of Example 1.50, if the sets X and Yare bounded from below,
then they have an infimum; let us denote by ml = inf X and m2 = inf Y. Hence it
holds
(VX E X) x 2 ml and (Vy E Y) y 2 m2.
Let s be an arbitrary element from S; then there exist x E X and y E Y such that
s = x + y. Then we have
s = x + y 2 ml + m2,
hence the set S is bounded from below by the sum ml + m2. Let us show that the
last number is also the greatest lower bound for S. For given $, there exist Xl E X
and Yl E Y such that
$ $
Xl < ml +2 and Yl < m2 + 2·
This implies
(V$ > 0) (:lSI := Xl + Yl E S) Sl = Xl + Yl < ml + m2 + $,
and thus we obtain that ml + m2 is the infimum of the set S.
Exercise 1.55. Find, if any, the infimums and supremums of the following sets and
check whether they are also their minimums or maximums.
5n --
a) X= { - 11 nEN }; b) X={~lnEN};
7n +2
c) X {I + 2.(~I)n In EN};
= d) X = {E 2\ I n EN} ;
e) X={:lm,nEN,m<n}.
Answers.
1.2 Cuts in Q
1.2.1 Basic notions
In Subsection 1.1 we gave the axioms (R1) - (R15) that defined the set of real
numbers R as a complete totally ordered field, (R, +,', ~). One can prove that,
up to an isomorphism, there exists a unique totally ordered field (Q, +"'~) which
satisfies the axioms (R1) - (R14). However, as one can see from Example 1.51, in
Subsection 1.1.2, Q does not satisfy the last axiom (R15).
The goal of this section is to expose an effective construction of the set of real
numbers R starting from the set of rational numbers Q. This method, developed
firstly by the German mathematician R. Dedekind in 1872, uses the so-called cuts
in Q. 2
We also put 0: :5 ~ if either 0: = ~ or 0: -< ~. The notations" »- " and " ~ " are
interpreted analogously. In Example 1.62 we shall prove that :5 is a total ordering
in R.
2 Another important construction of the set R starting from Q uses the so-called Cauchy se-
quences (see Definition 3.8), and was elaborated by another German mathematician, namely G.
Cantor. It is interesting to note that Cantor and Dedekind almost simultaneously announced their
constructions to the mathematical community.
30 CHAPTER 1.
If a cut a satisfies a >- 0* (resp. a t 0*) then it is called a positive cut (resp.
nonnegative cut). The set of positive cuts will be denoted by R+. The negative
and nonpositive cuts are defined analogously.
The addition in R, denoted by EB, is defined by
a EB 13 := {x + YI x E a, y E f3} (1.15 )
for arbitrary cuts a,f3 E R. We shall show that (R, EB) is an Abelian group, with
the additive identity 0*.
The absolute value lal of a cut a is defined by
for arbitrary positive cuts a,f3 E R+. It is an easy task to show that a 0 13 is a
positive cut when a,f3 E R+. Next, we put for every cut a
a 0 0* = 0*.
a0f3 := { lal 0 1131, if eithe~ a :::S 0* and 13 :::S 0*, or a t 0* and 13 t 0*; (1.18)
81a101f3l, otherWIse.
We shall show in Example 1.66 that (R \ 0*,0) is an Abelian group, with the
multiplicative identity 1*.
Moreover, the distributive law holds in R.
So we obtain that (R, EB, 0,:::s) is a totally ordered field. In Example 1.70 we
shall show that it is isomorphic to the field (R, +, " ::::;). This means that there is a
bijection3 <ft : R -+ R such that for every a,f3 E R it holds
a :::S 13 {=::> <ft( a) ::::; <ft(f3); <ft( a EB (3) = <ft( a) + <ft(f3); <ft( a 0 (3) = <ft( a) . <ft(f3).
Moreover, <ft can be chosen with the property that its restriction to the set of rational
cuts Q satisfies the condition
(Vr E Q) <ft(r*) = r.
3See Subsection 2.1.1.
REAL NUMBERS 31
Solution. Assume that there exists ayE a such that y 2 x. Then by property
(c2) from Definition 1.58 it follows that x E a, contradicting the assumption x rf- a.
Example 1.61.
a) Prove Theorem 1.59, i.e., that the set r* = {p E QI p < r} is a cut for every
rational number r.
b) Let a be a cut. Prove that the set Q \ a has a minimum iff a is a rational cut.
Solutions.
a) Take a rational number r and define the set r* by (1.13). We have to check the
three conditions (cl) - (c3) from Definition 1.58.
• The set r* is nonempty, since it contains, for instance, the rational number
r - 1, and since r + 1 rf- r*, it is a proper subset of Q.
• The second condition is satisfied by definition.
• Clearly, the supremum of r* is r, which by definition is not in r*.
Example 1.62. Prove that the binary relation ~ is a total ordering in the set of
cuts R.
a C (3 or (3 C a. (1.20)
Assume the contrary to 1.20. Then there exist two rational numbers a and b such
that a E (a \ (3) and b E ((3 \ a).
We have
From (1.21) we obtain that b E (3 implies b < a, while a E 0: implies a < b, hence
we have a contradiction.
Example 1.63.
a) If 0: and (3 are arbitrary cuts, then the set, := 0: Ell (3 defined by relation (1.15),
is also a cut.
b) Prove that the product of two positive cuts given by relation (1.17) is again a
positive cut.
Hence by relation (1.18), the product of two arbitrary cuts is also a cut.
• Since and (3 are cuts, then (cl) implies that there exist two rational numbers
0:
a E and bE (3, hence the rational number c := a + b is in ,. On the other
0:
hand, again (cl) implies that there exist rational numbers a' 1. 0: and b' 1. (3,
hence the rational number c' := a' + b' is not in ,. Thus, is neither empty
nor equals to the whole of Q, which means that it satisfies (cl).
• Finally, we have to prove that, has no maximum. Assume the contrary, i.e.,
that there exists a maximum Co in ,. Then by the definition of Ell given in (1.15)
there exist two rational numbers ao E 0: and bo E (3 such that Co = ao + boo
By (c3), the cuts 0: and (3 have no maximums, hence there exist al E 0: and
bI E (3 such that al > ao and bI > boo But then al + bI is an element from,
such that
al + bI > ao + bo = Co,
which means that Co is not the maximum of,. Hence, has no maximum, and
thus it satisfies (c3).
REAL NUMBERS 33
Exercise 1.64. Prove that the operations EEl and 129 satisfy the commutative law,
(Va, (3, , E R) (a EEl (3) EEl, = a EEl ((3 EEl ,), (a 129 (3) 129, = a 129 ((3 129 ,),
(Va, (3" E R) a 129 ((3 EEl ,) = (a 129 (3) EEl (a 129 ,).
b) the additive in verse for a E R for the operation EEl is the set 8a, where
8a:={-xEQlxE(Q\a), xf;a},
al = a, or equivalently al C a A a Cal'
• If y E aI, then by (1.15) there exist rational numbers a E a and x < 0 such
that y = a + x. Hence y < a, which by (c2) means that yEa. Thus al Ca .
• If a E a, then by (c3) there exists a' E a, such that a' > a. Hence
Since a' E a and (a - a') E 0', it follows that a E al' Thus a Cal-
We proved that a EEl O· = a. By Example 1.64, it also holds
0* EEl a = a EEl O· = a.
Remark. From Examples 1.63 and 1.65 and Exercise 1.64 it follows that (R, EEl) is
an Abelian group.
b) for given 0' E R+, the set 0'8 1* is the multiplicative inverse element of 0' for 0,
where
0'8 1*:= {x E QI x ~ O}U{~I x E (Q\a)' x =la,} (1.23)
Solution. We shall prove only part b) for the case 0' >- 0*. First, we shall prove
that the set 0'8 1* is a cut.
• Since 0' is a cut, it follows from (c1) applied to 0' that 0'8 1* is nonempty and
is a proper subset of Q.
• Let x E 0'81* and assume that the rational number y satisfies y < x; we have
to prove that y E 0'8 1*. If y ~ 0, then there is nothing left to prove. So we can
assume that 0 < y < x, which implies 0 < ~x < ~.
y
Using the last inequality, it
holds
x E 0'8 1* => ~ E (Q \ 0') => ~ E (Q \ 0') => y E 0'81*.
X Y
• Finally, we have to prove that 0'81* has no maximal element. The set Q \ 0'
either has a minimum a > 0, or it has not.
In the first case, it holds by definition of 0'8 1* (see relation (1.23)) that the
positive rational number 1/ a is not in 0'81*. Thus 0'81* has then no maximum.
In the case when Q \ 0' has no minimum, the following holds
But -1IS
. .III 0'
81*
.
x'
• Let now z E 1*. If z :::; 0, then z E a®a 81 ' by the definition of the last product.
So we can assume that 0 < z < 1. Since a is a cut, there exists a positive
rational number x E a such that ~ = y f!. a; otherwise the complement of a
z
. x
m Q would be empty. Put y:= - E Q \ {a}, then
z
Exercise 1.67. Prove that (Q, 81, ®,:::S) is a totally ordered field.
b) if a and (3 are cuts, then there exists a rational cut r* such that a -< r* -< (3;
Example 1.69. Prove that there exists a positive irrational cut (3, i. e., a cut with
the property that Q \ (3 has no minimum.
Solution. Let us show the existence of the cut ,. The class A either has a maximum,
or it has not. In the first case, put c := max A. Then the sought cut is , := c*. Next
we assume that A has no maximum. Let us put
G = {x E QI x* E A}.
We shall prove that G is a cut (i.e., we shall check the properties (cl) - (c3) from
Definition 1.58) and, moreover, show that G satisfies the inequalities in (1.24). This
will then imply G = , is a cut.
{3 -< p* =} p* E B =} p* rt A=} P rt G.
Thus the set G is neither empty nor does it equal the whole of Q.
• Let rEG and take some rational number rl < r. Then it holds
r* E A=} r; E A=} rl E G.
• In order to show property (c3) for the set G, we have to prove that G has
no maximum. Let us assume the contrary, i.e., that there exists a maximum
m E Q of G. We claim that then m* is the maximum in A, of course for the
binary relation ~ . Once we prove this, we shall get a contradiction with our
assumption that the class A has no maximum. This will then mean that G
can not have a maximum, i.e., (c3) holds for G. In order to prove that m* is
the maximum of A, under the assumption that G has a maximum m, we shall
suppose that m* is not the maximum of A. Then there exists a cut a E A
such that a >- m*. By Example 1.68 b), there exists a rational cut m~ such
that m* -< m;' -< a. But then m can not be the maximum of G, since ml > m
and ml E G.
Thus it follows that G is a cut. Let us take any a E A. Since A has no maximum,
there exists another cut al E A such that al >- a. From Example 1.68 b) we find
a rational cut r* such that a -< r* -< al' By definition, the rational number r is in
G, hence G >- a. Let now {3 E B. Then either G ~ {3 (as we claim), or G >- {3. In
the latter case there exists a rational cut p* E B, such that {3 -< p* -< G. But then
REAL NUMBERS 37
pEG, hence p* E A, which contradicts to the assumption that the classes A and E
are disjoint.
So we obtained that the cut G has the property
Exercise 1.71. Prove th(J,t every nonempty set X C R, bounded from above, has
a supremum in R, using' Dedekind 's theorem, Example 1.70 (compare to Example
1.50).
Exercise 1.72. Starting from the set of rational numbers Q given by axioms (Rl)
- (R14), the construction of cuts in Q gave the set of cuts. The last turned out to
be isomorphic to the set R given by the axioms (Rl) - (R15).
Now, if one starts from the set of real numbers R given by axioms (Rl) - (R15),
and defines the cuts in R in the manner of Definition 1.58, does one get a set
nonisomorphic to R?
Answer. No. In fact, Dedekind's theorem shows that the cuts in R give again real
numbers, hence no new elements arise from the above construction in R.
A set is bounded if it is both bounded from below and from above (compare to
Definition 1.3).
A set is infinite if there exists a bijection 4 that maps it onto one of its proper
subsets. The number sets N, Z, Q and R are all infinite sets. The sets that are not
infinite are finite. An infinite set X is countable if there exists a bijection between
X and the set of natural numbers N. The sets N, Z and Q are countable, however
the set R is not.
In other words, if {Oi liE I} is a collection of open sets whose union covers A, then
the Heine-Borel property of A implies the existence of a finite set 11 C I such that
AC U Oi.
iEh
Remark. In the Hausdorff topological spaces, (see Example 1.89) the Heine-Borel
property is often used for the definition of compactness. In Example 1.96 we shall
prove that a set in R is compact (= bounded and closed) iff it has the Heine-Borel
property.5
Ill}.
a) A=[O,l]; b) B = { 0, 1, "2' 4"' ... , 2n ' • .. ,
Answers.
a) AO = (0,1), A = [0,1], A' = [0,1]' no isolated points of A, aA = {O, I}.
b) W = 0, B = B, B' = {O}, all points from B \ {O} are isolated, aB = B.
c) Co = 0, C= CU{O}, C' = {O}, all points from C are isolated, ac = C.
d) DO = 0, D = D, D' = 0, all points from D are isolated, aD = D.
Solution. Let U( 0:) be a neighborhood of 0:, then by Definition 1.73 there exists
an interval (0: - C1, 0: + C1) c U(o:). This interval is also a neighborhood of 0:. Now
since 0: is an accumulation point of A, by Definition 1.76 c), there exists an element
0:1 i- 0: such that
0:1 E (0:-Cl,O:+C1)nA.
Put C2 := (10: - 0:11)/2. Then in the interval (0: - C2,0: + C2) there exists an
element 0:2 E A which is different both from 0: and 0:1. Continuing this procedure
ad infinitum, we get infinitely many points from A in the interval (0: - C1, 0: + cd,
hence also in the given neighborhood U(o:) of the point 0:.
5However, there exist topological spaces in which there exist sets that are bounded and closed,
but still do not satisfy the Heine-Borel property!
40 CHAPTER 1.
Example 1.83. Prove that every accumulation point of a set A c R is also in the
closure of A. Is the opposite statement true'?
Solution. Let 0' be an accumulation point of the set A. Then in every neighborhood
U(O') of 0' there exists a point f3 E A, f3f. 0'. By Definition 1.76 b), 0' is then in the
closure of A.
The opposite statement is not true, as will be seen from the following example.
Let A := (0-,1) U{2}. Then the closure of A is A = [0,1] U{2}, while the set of
accumulation points of A is A' = [0,1]. Namely, the point 2 is not an accumulation
point of A, but rather its isolated point, since in the interval (2 - ~,2 +~) there are
no points from A different from 2 itself.
Example 1.84. Prove that for every set A c R it holds A = AUA', i.e., the
closure of a set is equal to the union of that set and its accumulation points.
Solution. Let A be the closure of the set A c R. We shall prove that the set
B := R \ A is open. Let f3 E B. By Example 1.84,
Exercise 1.86. A set is closed iff it contains all of its accumulation points.
Example 1.87. The closure A of a set A c R is the smallest closed set that con-
tains A.
Exercise 1.88. The open sets in R satisfy the following three properties.
Example 1.89. Prove that for every two real points x and y, x -=J y, there exist
two disjoint neighborhoods of these points.
. . r r r r
SolutIon. Let r := Ix - yl > 0, then the mtervals (x - 3' x + 3) and (y - 3' y + 3)
are disjoint neighborhoods of x and y respectively.
Remark. A topological space with this property is called Hausdorff space. Thus
we proved that the set R with the topology given with the open sets from Definition
1. 74, is a Hausdorff space.
Exercise 1.90. Check whether the sets from Example 1.Bl are compact.
Answers.
b) Firstly, the set B is bounded, since B C [0,1]. The only accamulation point of
B is 0, which belongs to B, hence it is also closed. Thus B is compact.
c) The set C is bounded, but does not contain its accumulation point 0. Hence C
is not closed, which implies that it is not compact.
Example 1.91. Show that every nonempty subset S of a compact set K C R has
an infimum and a supremum, which both belong to K.
Exercise 1.93. Prove that a closed subset of a compact set is itself compact.
Example 1.94. A set K c R is compact if and only if its every infinite subset has
an accumulation point which belongs to K.
Exercise 1.95. Prove that every infinite sequence of elements from a compact set
has a convergent subsequence (see Definition 3.44).
Example 1.96. Prove that a necessary and sufficient condition for the compactness
of a set K C R is that it has the Heine-Borel property.
REAL NUMBERS 43
Solution .
By construction, for n > no every interval [an, bn] has infinitely many elements
from K, thus it follows that 0: is an accumulation point of K. A closed set
contains its accumulation points, hence 0: E K.
The collection of sets {Oil i E I} was assumed to be an open covering of K,
hence there exists an index i l E I such that 0: E Oil' Moreover, there exists an
integer nl with the property that for every n > nl it holds [an, bn] C Oil' So
we covered all these intervals with only one open set Oil' though we assumed
that no finite sub covering could cover any of them .
but since the set S is infinite, it is impossible to find a finite sub covering of
K, which is a contradiction with the Heine-Borel property.
Example 1.97. Does the set A = {;'I n EN} have the Heine-Borel property?
44 CHAPTER 1.
Solution. We shall show in two ways that A does not have the Heine-Borel prop-
erty.
• First method. The number 0 is the only accumulation (cluster) point of the
set A. Thus A is not a closed set implying that that it is neither compact. By
Example 1.96 this is equivalent with the statement that A has no Heine-Borel
property.
On = (~n - ~,
n n
~ + ~),
n
n E N.
Clearly A c U On, but the family {On I n E N}, has no finite sub covering of
nEN
the set A. This follows from the following relations:
1 n2 - 1 n2 - 1 n2 - 1 1 1
--= = <--=---
n+l (n+l)(n 2 -1) n 3 +n 2 - n - l n3 n n3 '
Exercise 1.98. Let uS) define a function d : R X R -+ [0, +=) called distance by
the formula
d(x,y) = Ix - yl, X,y E.R.
Prove the following properties of the distance d.
Functions
(£2) if the pairs (x, Yd and (x, Y2) are in f, then necessarily Yl = Y2·
Then we write f : A -+ B and say that "f maps A into B". If the pair (x, y) E f,
we shall write Y = f (x). The set A is then called the domain, while the set B is
called the codomain of the function f. The element x from the domain A is called
the independent variable, while the element y from the codomain B is called the
dependent variable.
The set
f(A) = {y E BI (3x E A) f(x) = y}
is called the range of the function f. By Definition 2.1, f(A) C B and, in general,
the range is a proper subset of the codomain.
Clearly, a function f is determined with the triple (A, B, f). This also means
that two functions II : Al -+ Bl and h : A2 -+ B2 are equal if and only if their
domains are equal, i.e., Al = A 2, their codomains are equal, i.e., Bl = B 2, and, of
course, it holds fl (x) = f2 (x) for all x E A.
A function f : A -+ B is a one-to-one function or injective if for every pair
Xl and X2 from the domain A it holds
Xl f- X2 =} f(Xl) f- f(X2).
45
46 CHAPTER 2.
Xl < X2 ::::} f(xd < f(X2) (resp. Xl < X2 ::::} f(xd > f(X2))'
FUNCTIONS 47
In the following, we usually omit the terms local and global and speak just about
. .
mInImUm or maXImum.
A function f : A - t B is bounded on the set X c A if there exists a number
C > 0 such that
(Vx EX) If(x)1 ~ C.
Geometrically, this means that the graph of the function f over the set X is settled
between two horizontal lines, namely y = C and y = -C.
The set X c R is symmetric (to the origin) if for every point X E X it holds
that the point -x is also in X. (Notice that for X -I- 0 the points X and -x are
symmetric to the origin.)
Suppose that the domain A of a function f : A - t B is symmetric. Then f is
an
even function if for every X E A it holds f( -x) = f(x);
Geometrically, the graph of an even function is symmetric to the y-axis, while the
graph of an odd function is symmetric to the origin.
A number T -I- 0 is called the period of the function f : A - t B if for all x E A
the points x + T and x - T are also in A and it holds
The smallest positive period, if it exists, is called the basic period of the func-
tion f. Clearly, if we know the basic period T of a function, then it is enough to
draw its graph on any set X C A of the length T.
48 CHAPTER 2.
Geometrically, if a function f is concave downward on the interval (a, b), then the
segment connecting any two points on its graph is under the graph. I
The basic elementary functions are the following ones:
y = smx, x E R; y = cos x, x E R;
• the inverse functions (with the possible restriction of the domains) of the
up to now mentioned basic elementary functions.
In particular, such are the so-called inverse trigonometric functions:
a) Determine the number of all functions f : A -+ B, i.e., that map the set A into
the set B.
Solutions.
Solutions.
b) Let y E f(X U Y). Then there exists an element x E XU Y, such that y = f(x).
This implies x E X or x E Y, hence y E f(X) or y E J(Y). SO we obtain that
y E J(X) U J(Y). Thus we proved the inclusion J(X U Y) c J(X) U J(Y).
The opposite inclusion J(X) U J(Y) c J(X U Y) is proved in an analogous
way (do it!).
50 CHAPTER 2.
Example 2.5. Let f : A --t Rand 9 : B --t R be two functions. Determine whether
the functions f and 9 given below are equal.
a) f(x) = #, A = R, g(x) = x3 , B = R;
b) f(x) = #, A = [0,+00), g( x) = x3 , B = [0, +00 );
2x
c) f(x) = -,
x
A = R \ {O}, g(x) = 2, B = R \ {o};
2x
d) f(x) = -,
x
A =R \ {O}, g( x) = 2, B = R;
Solutions.
a) The functions f and 9 are not equal, since f( -1) = 1, while g( -1) = -1.
(However, the restriction of their domains from the whole R to the interval
[0, +00) makes them equal (see b).)
d) and g) The functions f and g are not equal, since their domains are different.
Example 2.6. Determine the largest set A C R such that the following analytical
expressions have sense. (We call then A the natural domain of the function given
with that formula.)
FUNCTIONS 51
a) f(x) = (x - 3) - ;
2-x ~ b) f(x) = x· vcos Vi;
e) f(x) = arccos ( ~) ;
l+x
f) f (x) = In (arcsin G~ ~) );
g) f(x) = In(x + 4); h) f(x)=ln(sinv'-x2).
~
Solutions.
a) First, the given formula has no sense for x = 2, since for that value of x the
denominator becomes zero. Further on, since the natural domain of the square
root function g( t) = v't is the interval [0, +00 ), (or, equivalently, if t 2: 0), the
given formula has sense if and only if
b) The natural domain of the function g( t) = v't is the set {t E RI t 2: O}, which
means that A C [0, +00). Further on,
coss2:0 ~ sE U [(4k-1)~,(4k+1)~],
kEZ 2 2
or, equivalently,
Jr Jr
(4k-1)"2:Ss:S(4k+1)"2' kEZ.
Using the condition A C [0, +00) and putting in the last relation s = Vi, x 2:
0, we obtain that if x 2: 0, then cos( Vi) 2: 0 holds if and only if
for some natural number k. This means that the natural domain of f is the
set
= (u
kENo
[(V2k7r,V(2k+1)1I"J) u(U kENo
[-V(2k+1)1I",-V2k7r]).
d) The natural domain of the logarithmic function g(t) = In t is the open interval
(0, +00), hence the natural domain A of the function f will be the set of all
x E R such that sin( ~) > o. The last inequality is true iff there exists an
x
integer k such that
11"
2k7r < - < (2k + 1)11".
x
Solving these inequalities by x gives three cases.
1
• If k = 0, then 0 < - < 1, hence x E (1,+00);
x
-
A _ ( U (2k+1'2k
_1 ~)) U ( U (2k+1'2k
_ 1 1 )) U(l,+oo).
kEZ,k<O kEN
2x 2 2
-1 -< -
1 +-
x 2 -< 1 ~ -1 - x -< 2x -< 1 + x ,
f) The formula g(u) = In u has sense for u > 0, which means that we must have
arcsin-- >
x+2
5-x
° ~ --
x+2
5-x
>° ~ x E (-2,5).
Ix+21~1 x+2
I.e., -1<--<l.
5-x -5-x-
Thus we have
x+2
1--1
5-x
~ 1 ~ x E (-00,3/2].
Example 2.7. The function f is defined on the closed interval [0,1]. Determine the
set A on which the following composite functions can certainly be defined.
Solutions.
b) The largest set for which sin( x) E [0,1] is the union U [2h, (2k + 1)11-].
kEZ
x E [0,1] ~ x E [-3,-2] =: A.
Example 2.8. Let the function f : (0, +00) -+ R be given with the formula
f(D=x+~.
Find the formula for f.
54 CHAPTER 2.
Example 2.9. Let the function f : R \ {-I} --+ R be given with the formula
x
f(x) = - - . Find the function fn, n E N, where
l+x
fl = f, f2 =f 0 fl and fn =f 0 fn-l for n = 2,3, ....
Determine also the natural domains of these composite functions.
Solution. The range of the function f is the set R \ { -I}. Then it holds
x
x
h(x) =f h(x) = f(f(x)) = f (_X_) = ~
1 + 2x
0
l+x 1+--
l+x
Clearly, the natural domain of the last formula is the set R \ {-1/2}, though the
definition of f2 reduces its domain to the set R \ {-I, -1/2}.
Let us prove by mathematical induction that for n = 2,3, .. , it holds
fn(x) = - -x , x ER \{ II
-1'--2' }.
... ,--
1 + nx n
We proved already this formula for n = 2. Suppose it holds for n k, for some
natural number k > 1. Then
Using the mathematical induction, the reader should check that for every n = 2,3, .. ,
it holds
x
fn(x) = f(Jn-l(X)) = I X E R.
1 + nx
2'
Solutions.
d) f(x) = 1 - X 3/ 2 , X 2': o.
Example 2.12. Show that the function f : R -+ R given by f(x) = g(lxl), x E R,
is an even function for any function g : R -+ R.
Example 2.13. Prove that every function f whose domain A is a symmetric set
can be written as a sum of an even and an odd function.
56 CHAPTER 2.
1
If we put fl(X) := 2(f(x) + f( -x)), x E A, then
1
fl(-X) = 2(f(-x) + f(-(-x))) = fl(X) for every x E A,
a) the sum of two even (resp. of two odd) functions is an even (resp. an odd)
function,-
b) both the product of two even or of two odd functions is an even function,-
c) the product of an even and an odd function is an odd function.
Solutions.
a) Suppose f and 9 are odd functions, both defined on a symmetric set A c R.
Then for their sum f + 9 it holds
Remark. One must distinguish the product fg from the composition fog of two
functions f and g.
Solution. We shall prove this statement first for positive integers k, using the
principle of the mathematical induction. By assumption, the statement is true for
k = 1. If it is true for kEN, then
Example 2.17. Check whether the following functions are periodic; if yes, find their
basic periods T, if any.
Solutions.
a) It holds that the sin 2 x = ~(1 - cos(2x)), x E R. From Example 2.16 it follows
that the function g(x) = cos(2x), x E R, is periodic with the basic period
27r /2 = 7r. Hence, the function f is also periodic with the basic period 7r.
b) The zeros of the function f are of the form ±>/h, kEN. Let us show that the
distance between the zeros of f tends to zero as k -t 00, see Subsection 4.1.1.
lim
k-+oo /j(k + 1)7r - Jk;/ = }~~ /(k + 1~7r + Jk; = O.
c) Only the numbers 27r and 7r are candidates for the basic period of f. However,
f (~)
4
= v'2
2'
f (7r + ~)
4
= f (57r) = _ v'2
4 2·
Hence, the function f is not periodic.
Example 2.18. Determine the basic periods of the following functions, if they exist.
x l I
a) f(x) = cos 4" + cos x + 2" cos(3x) + :3 cos(5x), x E R;
2 7r 2 7r 2)
d) f(x)=tanyfx, 7r )
xE [0,- U ( (2k-l)2-,(2k+l)2- .
4 kEN 4 4
Solutions.
a) Firstly we have
X4
f(x + T) - f(x) = ( cos - + T- - cos 4"X) + (cos(x + T) - cosx)
1 1
+ 2" (cos(3(x + T)) - cos(3x)) + :3 (ms(5(x + T)) - cos(5x))
. T . 2x + T 2' T . 2x + T
- 2 Sln-Sln--- - sm-sm---
8 8 2 2
. 3T . 6x
- sm-sm
+ 3T - 2: 5T . lOx + 5T
-sm-sm----
2 2 3 2 2
Now f(x + T) - f(x) == 0 if and only if for some m, n,p, q from N it holds
2p7r 2q7r
T = 8n7r = 2m7r , T = -3 = -5
From the last equalities we have that m = 4n, p = l2n, q = 20n, and therefore
the given function is periodic with basic period T = 87r.
b) The function g(x) = cosx, x E R, is periodic with the basic period 27r, while
the function h(x) = cos(V3x), x E R, is periodic with the basic period ~.
However, the function f = 9 +h is not periodic, because there exist no nonzero
27r
integers k and I such that 27rk = V31.
sinT
- cos(x + T) cos x (jtan(x + T) + jtan(x)) ,
FUNCTIONS 59
°
the last expression is identically equal to iff T = k1r for some k E Z. Therefore
the given function is periodic with basic period T = 7r.
d) Not periodic.
Example 2.19. Prove that the function f given with the formula
f (x) = x - [x], x E R,
is periodic and find its basic period (if it exists), where [x] is the greatest integer part
of x, see Example 1.47 d).
Solution. For x = z + r, where z E Z and °: ;
r < 1, it holds [x] = z. (Note that
[x] = x iff x E Z, otherwise x-I < [x] < x.) Then
f(x) = f(z + r) = z + r - z = r.
If x E Z, then f(x) = 0. Thus
(Vx E Z) f(x) = f(x + 1).
If x E R \ Z, then from the decomposition x = z + r, where z E Z and °< r < 1, it
follows
f(x + 1) = (x + 1) - [x + 1] = x + 1 - (z + 1) = x - z = r = f(x).
Hence the basic period of f is 1.
Example 2.20. Check whether the Dirichlet function given by the formula
D( )
x
= {I, 0,
if x is a rational number;
if x is an irrational number,
is periodic.
Solution. Let us prove first that every rational number r is a period of the function
D. Namely, since the sum of two rational numbers is again rational, it follows that
D(x + r) = 1 = D(x) for every rational number x, while the sum of a rational and
°
an irrational number is an irrational number. Hence D( x + r) = = D( x) for every
irrational number x. Thus we proved that every rational number r is the period
of D. However, since there does not exist a smallest positive rational number, it
follows that the Dirichlet function has no basic period.
(Let us add that no irrational number is the period of D - check that!)
Example 2.21. Show that the following functions f : A ---+ B are bijections and
find their inverse functions.
a) f(x) = 3x + 4, A = B = R;
x2
b) f(x) = 3' A = (-00,0]' B = [0,+00);
f (x) = 3 G (y - 4)) + 4 = y,
which implies the surjectivity of f. Since for every pair xI, X2 E R it holds
Xl < X2 ~ 3XI < 3X2 ~ 3XI + 4 < 3X2 + 4 ~ f(XI) < f(X2),
it follows that f is monotonically increasing on R. Applying Theorem 2.2 we
obtain that f is a bijection. Then we know that there exists an inverse function
g. In order to find its analytical expression, we formally replace x and y in the
given formula y = 3x + 1 and then attempt to solve the equation for y. This
gIves us
1
x = 3y + 4 ~ y = 3(x - 4).
Hence the inverse function 9 : R -t R is given by the formula
1
g(x) = 3(x - 4).
c) Let y such that Iyl > 1 be given. Then solving the equation
I-x
y=l+x
= y ± fj+l
2x
e -1
y =- --
2e
~ 2ye X - e2x +1 = 0 ~ eX y2 + 1.
Hence
x = In(y + fj+l).
Clearly, for given y E R this x E R has the property y = f(x). Thus f is a
surjection and from the monotonicity of f it follows that it is a bijection. The
formula that gives the inverse function 9 : R ----* R is
b) Yi c Y 2 ~ f- 1 (y1 ) c r 1 (y2 );
d) Analogous to c).
Exercise 2.23. Find the largest sets A and B such that the function f : A ---+ B is
a bijection, and then find its inverse function g : B ---+ A, if
1 x
a) f(x) = x 5 ; b) f(x) = x + 1·
Answers ..
a) A = B = R \ {O}, g(x) = X- 1/5 .
x
b)A=R\{-I}, B=R\{I}, g(x)=-.
I-x
Example 2.24. Using the graph of the function g(x) = x 2 , X E R, sketch the graph
of the following functions.
1 .
a) f(x) = 1 +x 2 ' b) f(x) = ex2 ;
y
I
\ I
y y.K \ I
\ I
\ I
\ I \ I
\ I
,,
1
\ I
, .... ., I
% , .- %
o 1 0
I
1
1
Fig. 2.1. f(x) = 1 + x 2 Fig. 2.2. f(x) = ex2
a) The function is defined on the interval (-00, +00), is even and has no zeros. It
has a maximum at x = 0 (both local and global), is increasing on (-00,0) and
decreasing on (0,+00), concave upward. (Figure 2.1.)
FUNCTIONS 63
b) The function f(x) = ex2 is defined on the interval (-00,+00), is even and
has no zeros. It has a minimum at x = 0, is decreasing on (-00,0) and is
increasing on (0, +00), concave upward. (Figure 2.2.)
c) The function f( x) = sin x 2 is defined on the interval ( -00, +00 ), is even, but not
periodic (see Example 2.17 b)). The zeros of f are at x = ±.Jh, k = 0,1, ....
This function has local maximums at x = ±J% + 2k7r, k = 0,1, ... , and local
minimums at x = ±J3; + 2br, k= 0,1, .... (Figure 2.3.)
y
y
Example 2.25. Using the graph of the function g(x) = cos x, x E R, (see Figure
2.5), sketch the graph of the following functions.
1
a) f(x) = - ; b) f(x) = cos n x, n E N;
cos x
1
c) f(x) = Icos xl; d) f(x) = -Icos
-I;x
1
e) f (x) = In I cos x I; f) f(x) = In I cosx I·
Solutions.
where it holds cos x 01 o. The function f is even, periodic with basic period
271" and has no zeros.
64 CHAPTER 2.
1
0< cos x < 1, i.e., - - >- 1 ,
cos x
meaning that on this interval the function has a mInImUm at x = O. On
the interval (~, 3;,)
the function has a maximum at x = 7f. The vertical
7f 7f
asymptotes of the graph are x = -2 and x = 2· (Figure 2.6.)
J
y
U.II
1\
y I
1t "
~r
1
Fig. 2.5. f( x) = cos x Fig. 2.6. f(x) = cos x
b) The function f(x) = cos n x is defined on the interval (-00, +(0) is even and its
zeros are at the points ~ + k7f, k = 0, ±1, ±2, ....
If n = 2k, kEN, the function is periodic with basic period 7f. On the interval
[- ~, ~] , the function has minimums at - ~, ~, and a maximum at O. (Figure
2.7 for n = 2.)
If n = 2k + 1, kEN, the function is periodic with basic period 27f. On the
interval [-~, 3;] , the function has a minimum at 7f and a maximum at O.
(Figure 2.8 for n = 3.)
y ~
1
o 1
c) The function f( x) = 'cos x, is defined on the interval (-00, +00), is even and its
zeros are at the points ~ + k1r, k = 0, ±1, ±2, .... This is a periodic function
with basic period 7r.
On the interval [-~, %] , the function has minimums at the points -~, ~,
and a maximum at O. (Figure 2.9.)
r.Y
o 1 Jr/2 1t x
1
Fig. 2.9. f(x) = Icosxl Fig. 2.10. f(x) = I cosxl
e) The function f(x) = In Icos x, is defined on the intervals ((2k - 1)%, (2k + 1)%) ,
k = 0, ±1, ±2, .... The function is even, periodic with basic period 7r and has
zeros at the points k7r, k = O. ± 1 ± 2, ....
1
Fig. 2.11. f(x) =lnlcosxl Fig. 2.12. f(x) = 'nlcosx
:It x
, ,,
,, ,
b) The function f (x) = x sin x is defined for every x E R. This is an even not
periodic function, which has zeros at the points br, k = 0, ±1, ±2, .... At
the points %+ 2k1r, k = 0, ±1, ±2, ... , it holds f(x) = x and at the points
1 :y
," '"", ,
,, ,,
y=-1 " "
c) The function f(x) = sin ~x is defined for every x E R \ {o}. This is an odd, not
1
periodic function with zeros at the points br' k = ± 1, ±2, .... It is a bounded
1
function, namely -1 ::::: sin - ::::: 1.
x
1
The maximums of f are at the points ( /)- k ' k = 0, ±1, ±2, ... , and
7r2+27r
1
minimums at the points (37r /2) + 2h' k = 0, ±1, ±2, .... (Figure 2.15.)
d) The function f (x) = x . sin ~ is defined for every x E R \ {o}. This is an even,
x
1
not periodic function with zeros k7r' k = ±1, ±2, .... (Figure 2.16.)
Solutions.
y y
~2
lit
-1
-~2 lit
, __ ~M I" __ !~~__
--~----
:t
_____ ________
~ ~:M
o 1
x
+ arccot x = 2'
7r
e) arctan x x E R;
Solutions.
eX _ e- x
a) f(x) = sinh x = ; b) f(x) = cosh x = eX + e- x .
2
eX _ e-X
c) f(x) = tanh x = eX +
e- x ; d) f(x) = coth x = eX+ e-
X
eX - e- X
Solutions.
"
"
%
%
o 1
b) The function f : R --t [1, +00) is even and has a minimum at the point x = o.
(Figure 2.22.)
d) The function f : (-00,0) U (0, +00) --t R is monotonically decreasing and odd.
(Figure 2.24.)
FUNCTIONS 71
"
L
-----,
%
.!..=..1 o 1 ,,=-1
---------
%
1
~------~--------.!..=..-!--
tanh x ± tanh y
e) tanh( x ±)
y - .
- 1 ± tanh x . tanh y ,
2 tanh(x)
f) tanh(2x) = 1 + tanh 2 x .
Solutions. Before we prove the given relations, let us note that by the definitions
of the functions sinh and cosh, it follows
for every x E R. Using the main property of the exponential function, namely
eX+Y = eX . eY , x E R, and the equalities in (2.2), we obtain for every x, y E R
and
a) Summing (resp. subtracting) the equality (2.4) with (resp. from) the equality
(2.3), we obtain the given equality.
b) Putx=yina).
sinh(x ± y)
tanh(x ± y) = cosh(x ± y)
. x+y. x-y
b) cosh x - cosh y = 2 smh -2- smh -2-;
c) 'h x
sm + SIn
'h y = 2 SIn x2
'h - + -y cos h -2-;
X - y
d) sm
. h x - SIn x + y sIn
. h y = 2 cos h -2- . h -2-;
x - y
coth x . coth y ± 1
e) coth(x ± y) = cothx ± cothy .
where the coefficients aj, j = 0,1, ... , n, are real numbers, is called polynomial
function (shortly: polynomial) of degree n E N if the coefficient an :I 0. 2
2In (2.5), the subscript n in Pn(x) stands only to denote the degree of that polynomial.
FUNCTIONS 73
If Xo is a real zero of the polynomial Pn(x), then there exists a unique polynomial
Qn-1 with real coefficients,
The method of quick determination of the polynomial Qn-1 (x), the so-called Horner
scheme, is explained below.
In view of Theorem 2.32 and the factorization (2.7), the number Xo is called zero
of order m, mEN, 1 :S m :S n, of the polynomial Pn(x) from (2.5) if there exists
a polynomial Rn-m(x) of degree n - m, such that Rn-m(xo) i- 0 and it holds
Any polynomial Pn(x) with real coefficients of the form (2.5), can be written in
a unique way as the product
Pn(X) = an(x - X1)m , ... (x - x r )m'(x 2 + b1x + C1)11 ... (X2 + bsx + cs)l" (2.8)
where for the natural numbers mj, j = 1, ... ,r, and lk' k = 1, ... ,s, it holds
In (2.8), the numbers Xj are mutually different zeros of order mj, j = 1, ... ,r, of the
polynomial Pn (x), while the zeros of the polynomials X2 + bkx + Ck, k = 1, ... , s, are
conjugate complex numbers.
In general, it might be a tricky task to find the zeros of a polynomial. However,
in the special (but quite often) case when the polynomial Pn(x) from (2.5) has
only integer coefficients, then the candidates for rational zeros can be found quickly.
Namely, ifthe reduced fraction E, where p E Z and q EN, is the zero of Pn(x), then
q
necessarily the numerator p is a divisor of ao (we write that shortly plao), while the
74 CHAPTER 2.
or
bk=xobk+1+ak+b k=0,1, ... ,n-1,
and the remainder r is given by
r = xobo + ao.
Clearly, if r = 0, then the number Xo is the zero of the polynomial Pn (x) and
relation (2.9) coincides with (2.7). However, if r 01 0, then the number Xo is not a
zero of the polynomial Pn(x).
The Horner scheme is most easily performed as follows.
First one writes in a row all the coefficients of the given polynomial function Pn(x),
including those that are equal to zero. Using the upper formulas for bk, k =
0,1, .. , n - 1, and the one for r, we make the following two-row scheme:
an an-l an-2 ak ao I Xo
bn - l bn - 2 bk bo Ir.
All we have to do now is to apply the upper analysis, i.e., to check whether the
remainder r is zero or not.
The rational function is the quotient of functions
Pn(x)
R(x) = Qm(x)' Qm(x) 01 0, (2.11)
A Bx+C
(x_a)i,jEN, and (x 2 +bx+c)k' kEN, (2.12)
for some natural numbers j and k. In relation (2.12), A, B, C, a, band c are constants
and the zeros of the polynomial x 2 + bx + c are conjugate complex numbers.
FUNCTIONS 75
Theorem 2.33. A rational function of the form (2.11) can be written in a unique
way as the sum of partial fractions of the form (2.12) and, if n 2': m, a polynomial
of degree n - m. (If n = m, then the last polynomial reduces to a constant.)
e) P(x) = x7 + 6x 5 - 5x 4 + 9x 3 - 10x 2 + 4x - 5.
Solutions.
b) For the polynomial P( x) we shall find the divisors of 12, since ao 12. The
divisors are
±1, ±2, ±3, ±6, ±12.
Further on, since an = a3 = 1, the only possible rational zeros of the polyno-
mial P(x) are integers from the upper set of divisors.
Let us check whether x = 1 is a zero of P(x)
1 3 -4 -12 I x=l
1 4 0 1-12.
Since the last number in the second row is -12 (i.e., r = -12), the number
x = 1 is not a zero of P(x).
Next, we check whether x = 2 is a zero of P(x)
1 3 -4 -12 I x=2
1 5 6 10.
76 CHAPTER 2.
Now the last number in the second row is 0, (i.e., r = 0), hence x = 2 is a zero
of the given polynomial. So we obtained the factorization
x3 + 3x 2 - 4x - 12 = (x - 2)(x 2 + 5x + 6).
x 2 + 5x + 6 = 0,
or as we shall do, using the Horner scheme.
Let us try the number x = -2
1 5 6 I x =-2
1 3 10.
Hence, the second zero of the polynomial P3(X) is the number x = -2. So we
obtained the factorization
The positive integers that are divisors of 2 are 1 and 2, hence the possible
rational zeros of the given polynomial are
1 3
±1, ±2, ±3, ±6, ±2"' ±2".
U sing the Horner scheme, we obtain that x = 1, x = 2 and x = 3 are the zeros
of the given polynomial:
d) The only possible rational zeros of the given polynomial are 1 and -1, hence the
Horner scheme gives
1 0 -1 1 0 -1 I x=1
1 1 0 1 1 10 x =-1
1 0 0 1 10 x =-1
1 -1 1 10.
This shows that
The last factor x 2 - x + 1 has no real, therefore no rational zeros, but rather
its zeros are conjugate complex numbers. This means that the formula (2.13)
is the factorization of the form (2.8).
e) The possible rational zeros of P(x) are ±1,±5. The Horner scheme gives
1 0 6 -5 9 -10 4 -5 I x = 1
1 1 7 2 11 1 5 10 x=1
1 2 9 11 22 23 128.
This shows that x = 1 is a zero of first order, and not of second order, since
the last number in the third row is 28. We leave to the reader to check that the
numbers x = -1, x = 5 and x = -5 are not the zeros of the given polynomial,
hence it has no other rational zeros. Moreover, the polynomial P(x) has no
other real zeros, and the factorization of the form (2.8) is
P(x) = x7+6x5-5x4+9x3-10x2+4x-5
x+l x+3
c) d)
x 3 - 2x 2 + X - 2 ' x4 - 5x 2 + 4'
x2 + 1 2X2 - 4x + 3
e) f)
(x-l)3' x4 - 6x 3 + 13x 2 - 12x + 4
Solutions.
a) The zeros of the polynomial x 2 - 1 (the denominator of the given rational func-
tion) are x = 1 and x = -1. Using Theorem 2.33, it follows that it can be
decomposed as the sum of following partial fractions
2 A B
- - = -x -
x 2 -1 -1
+- -.
x+1
78 CHAPTER 2.
From this equality we get the constants A and B in the following way. First,
we add the last two fractions
A B A(x + 1) + B(x - 1)
--+--=
x -1 x +1 x 2 -1
.
Since the denominator of the obtained fraction is the same as that of the given
rational function, their numerators should be also the same, hence
2 = x(A + B) + (A - B).
2 1 -1
-2-- = - - + --.
x -1 x-I x+l
b) Let us notice first that the degree of the numerator is greater than that of the
denominator, so let us do the following transformations:
x 3 - 2x - 35 x3 - 2x 2 - 15x + 2x 2 + 13x - 35
x 2 - 2x -15 x 2 - 2x -15
2X2 + 13x - 35 17 x - 5
x+ 2 =x+2+ 2 •
x - 2x - 15 x - 2x - 15
17x - 5
x 2 - 2x - 15
Further on we have
17x-5 A B A(x+3)+B(x-5)
x 2 - 2x - 15 = -x---5 + -x-+-3 = -----'---x--=-2--'-2-x------'---1-5-----'- ,
which gives us the system of linear equations
A + B = 17, 3A - 5B = -5.
So we obtain A = 10 and B = 7 and finally
x 3 - 2x - 35 2 10 7
=
----::--------::-----------:'CC' X + + -- + --.
x 2 - 2x - 15 x - 5 x + 3
FUNCTIONS 79
x+1
-A- + ---:---
Bx+C
x3 - 2x 2 + X - 2 X - 22 x +1
Ax 2 + A + Bx 2 - 2Bx + Cx - 2C
(x - 2)(X2 + 1)
or
x +1 (A + B)x 2 + (C - 2B)x + A - 2C
x 2x + X - 2
3 - 2 x 3 - 2x 2 + X - 2
This gives us a system of linear equations
x+3 ABC D
--,-----:-- = - -
X4 - 5x 2 + 4 x-I
+- - +--
x +1 x - 2
+ --.
x +2
Adding together the last fractions gives us
Cx 3 + 2Cx 2 - Cx - 2C + Dx 3 - 2Dx2 - Dx + 2D
+ X4 - 5x 2 + 4
(A + B + C + D)x3 + (A - B + 2C - 2D)x 2
X4 - 5x 2 + 4
-(4A + 4B + C + D)x - 4A + 4B - 2C + 2D
+ ~--------~~-------------
X4 - 5x 2 +4
Hence
A + B + C + D = 0, A- B + 2C - 2D = 0,
-(4A + 4B + C + D) = 1, -4A + 4B - 2C + 2D = 3,
80 CHAPTER 2.
which gives
A = -2/3, B = 1/3, C = 5/12, D = -1/12.
So we obtained the decomposition
x +3 -2 1 5 1
-x4:---5-x-=-2-+-4 = 3(x -1) + 3(x + 1) + 12(x - 2) - -12-(x-+ 2)"
x2+1 ABC
-- + + -,-----
(x - 1)3 x-I (x-l)2 (x-l)3
Ax 2 - 2Ax + A + Bx - B + C
(x - 1)3
which gives the system of linear equations
x2 + 1 1 2 2
- - - = -x-I
(x-l)3
- + (x-l)2 +(x-l)3·
---
2X2 - 4x + 3 ABC D
---,--:--::--.,-,:--=-----,-.,-----:- = - - + + -- + ---
x 4 -6x 3 +13x 2 -12x+4 x-I (x-l)2 x-2 (x-2)2
A + C = 0, -5A + B - 4C + D = 2,
8A - 4B + 5C - 2D = -4, -4A + 4B - 2C + D = 3,
2X2 - 4x + 3 2 1 - 2 3
--------- = -- + + - - + -,------
4
x -6x 3 +13x 2 -I2x+4 x-I (x-I)2 x-2 (x-2)2
FUNCTIONS 81
1 2
c) d)
X4 + l' X4 + x 2 + l'
Solutions.
3 1 x-2
a) ---
x3 + 1 x+l x 2 - X + l'
4x 2 + 4x + 26 1 x+2 3x +4
b) ------
(2x - 4)(X2 + 1)2 x-2 x2+1 (x 2 +1)2'
1 V2x+2 V2x-2
c)
X4 +1 4(x 2 + xV2 + 1) 2
4(x -xV2+1)'
d) 2 x-I. x +1
x4 + x2 + 1 x2 +X + 1
x2 - X + 1
Example 2.37. Prove that the function f( x) = X1 / 2, x 2: 0, is an irrational
function. 3
Solution. Assume that there exist polynomials P and Q such that
1/2 _ P(x)
x -Q(x),x2:0.
Then
1 P(x)
y'x xQ(x)' x> 0,
hence
lim _1_ = lim P(x) = O.
x--++oo y'x x--++oo xQ(x)
So the degree of P(x) is less than the degree of x . Q(x) and the degree of P is ~
than the degree of Q. Thus
·
11m r::: l' P(x) K
X~+OO
yX = 1m
X----++OCl
Q( X ) = ,
3In the proof, we use the notion of the limit of a function at +00, given in Definition 4.4.
Chapter 3
Sequences
3.1 Introduction
3.1.1 Basic notions
A sequence is a function a : N -+ R. It is usual to write
Definition 3.1. A real numbed! is the limit of a sequence (an)nEN if for every
E > 0 there exists a natural number no = no( E), such that for every n > no it holds
I an -£ 1< E, i.e.,
• tends to minus infinity if for every real number M > 0 there exists a natural
number no = no(M), such that for every n > no it holds an < -M (then we
lim an = -=).
write n->()()
83
84 CHAPTER 3.
Theorem 3.5. A sequence converges iff it is bounded and has exactly one accumu-
lation point.
Theorem 3.6. If (an)nEN and (bn)nEN are convergent sequences, then it holds
• n---+oo
lim (an· bn) = n---+oo
lim an· n--+oo
lim bn;
lim an
· a n n-+oo
• 11m - = - - -
n-+oo bn lim bn '
n-+oo
where bn f 0 for every n E N and lim bn f O.
n-+oo
Theorem 3.7.
• Assume (an)nEN and (bn)nEN are convergent sequences, with the property that
there exists a natural number no such that for every n > no it holds an ~ bn .
Then
lim an ~ n--+oo
n--+oo
lim bn-
• If for the sequences (an)nEN, (bn)nEN and (Cn)nEN there exists a number no
such that for every natural number n > no it holds
an ~ bn ~ Cn,
(n--+oo
lim an = n--+oo
lim Cn = £) :::} (lim
n--+oo
bn = f).
Definition 3.8. A sequence (fn)nEN is a Cauchy sequence if it satisfies the fol-
lowing condition
(VE > 0) (3no E N) (Vm, n E N) m, n > no :::} Ifm - fnl < E. (3.2)
(Vf > 0) (3no E N) (Vn,p EN) n > no :::} Ifn+p - fnl < f. (3.3)
lOne can prove that the axiom (R15) from Subsection 1.1 is equivalent with the last statement,
which explains the term "completeness of R" in (R15).
SEQUENCES 85
1 n-l
a) a n -_. b) b =-_.
n2 + 2'
- ,
n n
1 1
c) c - . d) en -- _ ".
n - ~n + 3' n.
1
e) dn = - a> O.
n Ol '
Solutions.
a) We have to show that for every given positive number e there exists a natural
number no = no (e) such that the following implication holds
(\:In E N) (n > no =? Ian - 01 < e), i.e., (\:In EN) (n > no =? ~n < e).
From the last relation it follows that for every n which has the property n > ~,
. 11 I 1.
It holds ;:; - 0 = ;:; < e. ThIs means that we can take no = [1] + 1, wheree
~
[xl is the greatest integer part of x defined in Example 1.47.
Remark. It is sufficient to consider only the case when e > 0 is "small" (say when
o < e < 1). Namely, such a restriction does not loose on generality, because if there
exists no = no(c) such that for every n E N, n > no, it holds Ian - £1 < e (see
relation (3.1)), then
b) For every given e > 0 there exists a natural number no = no( e) such that
n-l
(\:In E N)(n > no =? Ian - 01 < e), i.e., (\:In E N)(n > no =? -2-- < e).
n +2
n- 1 n -1 n2 1
--<---n --<-<e
n2 + 2 n2 + 2 - n2 +2 n '
and the case a), we obtain that we can choose no such that no = [~] + 1.
86 CHAPTER 3.
c) For given c > 0 we shall find a natural number no = no(c) such that for every
n > no it holds
1~I<c.
From the implications
I ~-ol
nOi =~
nOi <c, hence
1
- < nOi
c
or
1
\IE < n.
So we can take no = [ ~] + 1.
If a is a positive irrational number, then there exists a rational number f3 such
that it holds 0 < f3 < a and n f3 < n Oi . Hence
Example 3.11. Show by Definition 3.1 that the following sequences have a limit
which is equal to one.
a) -+ 1 .
a n -_ n b) bn -- nl(i
yu, a > O·, c) en = y'ii.
n+2'
Solutions.
SEQUENCES 87
If we suppose that a > 1, namely a = 1 +b, b > 0, then for every n EN, n > 1,
there exists a real number c( n) > 0 such that it holds
Thus lim
n--+oo
yra = 1 for any a > O.
n(n - 1)
n = (1 + d(n)t = 1 + nd(n) + 2 d2 (n) + ... + dn(n),
we obtain the estimation
n>
n(n - 1) d2 (n),
2 wherefrom we have d(n) < J~
n l'
88 CHAPTER 3.
Iyrn - 11 = d( n) < J 2
n -1
< e, provided that n > [~e + 1] .
For given e choose no = [:2 + 1] + 1.
Example 3.12. Prove the following limits:
0, Iql < 1;
r qn = { 1, q = 1; b) lim nbqn = 0, Iql < 1, bE R;
a) n~~ +=, q > 1;
n .... =
an na
c) lim -, = 0, a E R; d) lim -, = 0, a E R.
n---+oo n. n-+oo n.
Solutions.
a) First of all, let us consider the case when Iql < 1. We can write then
Iql = 1 h'
1+ h > 0.
Then, for given e > 0, using the Bernoulli inequality, the following estimation
holds
1 1 1 1
Iqn - 01 -- (1 + h)n <- - - < - < e'
1 + nh - nh for every n > he'
So we can take no(e) = [hI]e + 1. Thus it follows for Iql < 1 that lim qn
~=
= 0.
If q = 1, then it is obvious that lim qn = l.
n .... =
If q > 1, then we have
qn =~,
rn
°< .,. < 1, hence lim qn
n-+oo
= +=,
because lim
n .... =.,.n = 0.
Remark. In the case q :::; -1, the sequence qn does not converge. If in particular
q = -1, then it has two accumulation points 1 and -1.
because
n
n > 2k ==?- n > 2k - 2 ==?- 2n > n + 2k - 2 ==?- n - k + 1 > 2'
If we take h > 0, such that Iql = ~h < 1, then from the previous estimations
1+
we have
Inbqn _ 01 __ n bl q In __ (1 +
nb
h)n
n b2_
<_ k k! -
nkhk -
2kk! b-k
--y;;;-n <
2 k k'. b-k
--y;;;-n 1 < c:,
for every
n> [k_b1Wl
V2kkf +1,
where b1 E Q such that b < b1 < k.
c) If it holds that lal < 1, then from a) and Example 3.10 d) it follows that
an
lim -, = O.
n~oo n.
If lal > 1, then it holds
I
an
n! -
1_ -;J
lanl _ lal lal lal lal lal lal
- T . 2 . 3' .. k . k + 1 .,. -:;;'
where kEN is chosen sufficiently big in order to satisfy the following estima-
tions
~ >1
k -
and ~
k +1
< l.
Therefore we can write
lim M. I I )n-k =
( _a_ O.
n->oo k+1
From the last two relations and Theorem 3.7 it holds that lim an, exists and
n ...... oo n.
is equal to zero.
d) Let us suppose that a > 0 and let the natural numbers k and n satisfy the
following inequalities
n n
k > a and n > 2k, hence n - k + 1 > 2' ... , n > 2'
90 CHAPTER 3.
1 nk 2k
<
(n - k)! (%r - (n - k)!'
.----
it follows that for every E > 0, there exists a natural number no = no(E) such
that it holds
Inal
2k 2k
n! < (n - k)! < n _ k < E,
Exercise 3.13. Show by definition that the following sequences converge to zero.
a) fn = (_l)n 1995
n 1995 ; b) fn = y'n;
c) fn = 1995 n . d) fn = n 1995
f •
n! ' n.
Exercise 3.14. Prove the following.
a) n-->oo
lim n 5 = +00; b) li.~ (~r = +00;
c)
nf
lim - ' = +00;
n-->oo 12 n
d) lim (20 -
n-+oc> n
n) = -00;
e) n-->oo
lim n 1 / k = +00, kEN; f) lim
n-->oo ifrJ = +00.
2n 5 - 3n 2 + 1 b) lim 3n4 + 2n 2 + 1
a) lim
n-->oo n 5 + 3n + 2 ; n-->oo n3 + 1
e . (n2 + 2n + 3 -
11m n3 + 5 ) .
f) lim(y'n+2-y'n);
) n-->oo n+ 1 n 2 + 2n + 1 ' n-->oo
g) lim(v'n 2 +2n-n);
Vn+T
n-->oo h) li.~ Vn+2 + y'n+3'
Solutions. We shall repeatedly use Theorem 3.6.
SEQUENCES 91
2n 5 - 3n 2 + 1
5 _ _ ( 3 1)
. n 2 - n3 + n5 _ n-+oo
lim 2 - - + -
n3 n5
(3 1)
a) lim = hm ( 3 2) - ( 3 2 ) = 2.
n-+oo n 5 + 3n + 2 n-+oo n5 1 + - + - lim 1 + 4" + 5"
n4 n 5 n-+oo n n
. 3n4 + 2n 2 + 1
11m . n 4 (3 + 22 + -;)
b) = hm n n
n3 + 1 1) = +00.
n 3 1 +-3
n-+oo n-+oo (
2 n 2(8+-+-
3 1) 3
8+-+- 1
c) lim 8n + 3n + 1 lim n n 2 = lim..!:... n n2 = o. 8 = O.
n3 + 1 1 ) n-+oo n 1
n3 1 + - 3 1 +-3
n-+oo n-+oo (
n n
· (2n+1)3-8n3
11m 1·8n3 +12n 2 +6n+I-8n 3
d) = 1m = 12.
n-+oo 2 n +1 n-+oo n2 + 1
. (n2 + 2n + 3 -
11m n3 + 5 ) 1. n 3 + 3n 2 + 5n + 3 - n 3 - 5
e) = 1m
n-+oo n+ 1 n 2 + 2n + 1 n-+oo (n + 1)2
2 n2 ( 3+ 5 - - 2- )
lim 3n + 5n - 2 = lim n n 2 = 3.
(n + 1)2 1) 2
n 2 1 +-
n-+oo n-+oo (
lim
n-+oo (v'n+2 + yin) -- 0.
yin. VI + ~
n_~,;n . (~r.3)
1
h) -
v'ii+l
,!i.'1t, vn+2 + v'n+3 = lim
V1+ ;; + V1+ ;; 2
92 CHAPTER 3.
1+2+3+"'+n 12 + 22 + 3 2 + ... + n 2
a) b)
n2 n3
1 . 2 +2 . 3 + " . + n( n + 1) . 1 + 3 + 5 + ... + 2n - 1 2n +1
c) d) ---
n3 n+1 2
1 3 5 2n - 1 1 +a + ... + an
e) -
2
+ -22 + -23 + " . + -2-
n
'' f)
l+b+,,·+bn '
o < lal, Ibl < 1.
Solutions.
. 1·2+2·3+3·4+,,·+n(n+1)
hm - - - - - - - - - - ' - - - - - - ' -
n~oo n3
= lim 1(1+1)+2(2+1)+3(3+1)+"'+n(n+1)
n--+oo n3
. 12 + 22 + 3 2 + ... + n 2 + 1 + 2 + 3 + ... +n
hm - - - - - - - - -
n--+oo n
. ( 1 + 3 + 5 + ... + 2n - 1 2n + 1)
11m ---
n->oo n +1 2
lim
n->oo
(~_ 2n +
n +1 2
1) = lim -3n -
n->oo 2(n + 1)
1= _ ~. 2
SEQUENCES 93
1 3 5 2n - 1
e) Putting fn = "2 + 22 + 23 + ... + ~' (using Example 1.24) we have
fn f _ fn = ~ + (~ _ ~) + ... + (2n - 1 _ 2n - 3) _ 2n - 1
2 n 2 2 22 22 2n 2n 2n+1
1
1 (1 1 1 ) 2n - 1 1 1 1- 2n"=1 2n - 1
"2 + "2 + 22 + ... + 2n- 1 - 2n+1 ="2 + "2 . 1 - 2n+1 .
1- -
2
Thus we obtain
lim fn
n->oo
. (
hm 1+2 1 - - -2n
n->oo 2n- 1 2
(
--n- 1) 1)
· -1- - 2 1·1m -n
3 - 11m 1·1m -1 = 3.
+ n->oo
n->oo 2n- 2 n->oo 2n 2n
f) In the numerator and the denominator we have geometric sums and therefore we
can write
1 __ an +1
. 1 + a + ... + an . 1_ a 1 - b . 1 - an+1 1- b
hm = hm hm ----,----,-
n->oo 1 + b + ... + bn n->oo 1 - bn+1 1 - a n--+oo 1 - bn+1 1- a
1- b
a) Xn = V16 + ~2; b) Xn = ( 16 + 1
2n r 1 3
/
;
e) Xn = Jn 2 + n - Jn 2 - n; f) Xn = fin + 1 - fin - 3.
Answers.
1
a) 4. b) 2y!2. c) 1. d) 1. e) 1. f) O.
94 CHAPTER 3.
V'3-3 . yI64 - 1
a) Xn = 1+ \11995' b) Xn = V'8 _ 1 '
1 2 y1n5 + V'5 .
c) Xn = 1 _ V'3 - 1 - \1'9; d) Xn = 5 yr:;;'i + ytSn'
e) Xn= n4+4n
n + 4n +!' f) Xn = C f;
9: 5
Answers.
1 1
a) -1. b) 2. c) d) -.
2 3
1
e) 4' f) O. g) O. h) O.
Exercise 3.19. Determine the limits of the following sequences.
1 - 2 + 3 - ... + (2n - 1) - 2n
a) Xn = v'n2+1
b) Yn = 12 + 32 + ... + (2n - 1 )2
n3 ;
1 (1 1 1)
c) Zn = Vii J2 + V4 + V4 + v'6 + ... + ffn + V2n + 2 .
Answers.
a) -1. b) ±
3'
c) J2
2 .
Example 3.20. Determine the limit of the following sequences.
a)
1 1 1.
Xn = ~ + 3.5 + ... + (2n -1). (2n + 1)'
c) Zn ~ (1 - D ( D {- 1- n( n :+ 1) ) ;
23 - 1 33 -1 n3 - 1
d) w - - - . - - ... - -
n - 23 + 1 33 +1 n3 + 1.
SEQUENCES 95
Solutions.
a) In this case we have
lim Yn
n--+oo
= lim
n--+oo
(1 - ~)
22
(1 - ~)
32
... (1 - n 2 2..)
= lim (~. 2·4 . 3·5 ... (n - 1)(n + 1)) = lim ~. n + 1 = ~.
n--+oo 22 32 42 n2 n--+oo 2 n 2
c) Since it holds
1 _ .. 1 = (j - 1)(j + 2) .
J(J + 1) j(j + 1) , J = 2,3, ... ,n,
2
we have
lim
n--+oo
Zn lim (1 -
n--+oo
~) (1 - ~) ... (1 _ n(n 1+
3 6 1)
)
2
j3 -1 (j - 1) (j2 + j + 1) (j - 1) (j2 + j + 1)
j3 + 1 (j + 1) (P - j + 1) (j + 1)((j - 1)2 + (j - 1) + 1)'
we obtain
23 1 33
- -1 n3 - 1
lim Wn = lim - - . - - ... - -
n--+oo n--+oo 23 + 1 33 +1 n3 +1
. (1 . 7 2· 13 3· 21 4· 34 (n - 1) ( n 2 + n + 1) )
= l~~ 3·3· ~ . 5·13 . 6·21 ... (n + 1)((n - 1)2 + (n - 1) + 1)
. 2 n2 + n + 1 2
= hm -. =-
n--+oo 3 n(n + 1) 3
96 CHAPTER 3.
tIn
J=2
(1 - ~2) = ir (1 - ~2) .
J
In
J=2 J
The logarithmic function is a continuous one and therefore we have
lim
n ..... oo
L
j=2
n
In (1) 1 -""72
J
= In lim
n ..... oo
II
n
j=2
(
1 -""72
J
1) 1= In -.
2
vn 2 + 1 + vn 2 + 2 + ... + vn 2 + n '.
a) . (1
11m 1 1 )
n ..... oo
an
b) lim
n-+oo (1 + a)(l + a 2) ... (1 + an) , a> 0;
and
n l'
~=1= 1m ~,
·n
11m
v n2 + n n ..... oo v n2 + 1
n ..... oo
+
. (1
11m
n ..... oo vn 2 +1 vn 1+ 2 + ... + vn21+ n)
2
= 1.
SEQUENCES 97
thus
an
lim = O.
n-+oo (1 + a)(1 + a 2 ) ••• (1 + an)
If a = 1, then
·
11m 1 . l'1m - 1 = O.
n-+oo 2 . 2 ... 2 n-+oo 2n
'-v---'
n-terms
Therefore we can write for all a >0
an
lim = O.
n-+oo (1 + a)(1 + a 2 ) .• • (1 + an)
c) Denoting the general term of the given sequence by h n , we have
I)-l/n
( 3 -:;; , n = 1,3,5, ... ;
{
hn =
(3 - :;;1) l/n
, n = 2,4,6, ....
3- l / n < h n < 3 l / n .
lim 3- l / n
n~oo
= nlim
...... oo
3l / n = 1
'
we have lim h
n-+oo n
= l.
lim an
n--+oo
lim lanl = lal.
= a:::} n--+oo
Is the opposite statement true'?
98 CHAPTER 3.
The opposite is not always true. For instance, the sequence given by
_(n+l)(-I)n, nEN,
an - n
a) lim j2n = j2
n---+(X)
=> n---+oo
lim fn = f;
b) lim
n--+oo
P =F
n
lim in = f.
=> n--+(X)
Answers.
a) No. For example, the sequence fn = (_I)n, n E N, does not converge, but
the stationary sequence f? = ((-I)n)2 = 1, n EN, converges to 1.
b) Yes.
Exercise 3.25. Show that there exist sequences (an)nEN and (bn)nEN such that
lim an = +00 and n--+oo
n--+oo
lim bn = +00, but
b) a n = -+n,
1
n
1
bn = - - 2n;
n
lim (an
n-+oo
+ bn ) = lim
n--+oo
(~-
n n) = -00.
c) an = -+2n,
1
n
1
bn = -n - n; lim (an
n--+oo
lim (~+
+ bn ) = n--+CXl n n) = +00.
d) an = (_I)n + n, bn = -n; the sequence (an + bn)nEN neither converges nor
diverges to +00 or to -00.
Exercise 3.26. Show that there exist sequences (an)nEN and (bn)nEN such that
lim an = +00 and lim bn = +00, but
n--+oo n--+oo
SEQUENCES 99
. an . an
11m
a ) n~oo -b --
n
o·, b) lim abn = A, A> 0;
n .... oo
c ) 11m -
n-+oo bn
= +00;
n
b) an = 2n, bn = n; lim an = 2.
bn
n .... oo
Exercise 3.27. Show that there exist sequences (an)nEN and (bn)nEN such that
lim an = 0 and lim bn = +00, but
n--+
00 n--+oo
a) nlim
.... oo
an . bn = 0; b) nlim
.... oo
an . bn = A, A ~ 0; c) lim an . bn = +00'
n--+oo '
1
b) an = -, bn = 3n; lim (an' bn ) = 3.
n n .... oo
1
c) an = -, bn = n2; lim (an' bn) = +00.
n n .... oo
( _l)n
d) an = - - , bn = n 2 ; clearly, the sequence (an' bn)nEN neither converges nor
n2
diverges to +00 or to -00.
Exercise 3.28 .
• Assume that the sequences (an)nEN and (bn)nEN satisfy either lim an = +00
n .... oo
and lim bn
n--+oo
= +00, or lim an
n-+oo
= -00 and lim bn
n-+oo
= -00. Prove then
• Assume that the sequences (an)nEN and (bn)nEN satisfy either lim an =
n--->oo
-00
and lim bn = +00, or lim an = +00 and lim bn = -00. Prove then
n---+oo n--+oo n--+oo
lim anbn =
n--->oo
-00.
Exercise 3.29. Show that there exist sequences (an)nEN and (bn)nEN such that
lim an = 0 and lim bn = 0, but
n--+ 00 n--+oo
. an . an
11m . an
a ) 11m - = 0; b) -b -1· c ) 11m - = +00·'
n--+oo bn n--+oo -
n , n-+oo bn
a) a = 1+1
- + -1
+ ...1
+ _.
n 4 2 9 n '
b) b
n
=sin-
2
1 sin 2
+-22
sin n
+ ... +_.
2n '
l' 2' ,
c) en = c;~ 2· + c;~ 3· + ... + n .c~::·I);
1 1 1
e) en =I+-+-+···+-.
ln2In3 Inn
Solutions.
a) Using Definition 3.8, we shall show that for every c > 0, we can find no = no(c)
such that for every n 2 no and for every pEN it holds la n+p - ani < c. First
we have
Ian
+p
- an I = 11 + 4~ + ... + ~ + 1 + ...
n 2 (n+lF
+ 1 - 1 - ~4 -
(n+pF
... - ~
n2
I
1 1 1
=
(n+lF + (n+2F + ... +---
(n+p)2
1 1 1
<
n(n+l)
+ (n+l)(n+2) + ... +(n+p-l)(n+p)
---
11 1 1 1 1 1 1
= - - - - + - - - - - + - - - - - + ... + ---
n n+l n+l n+2 n+2 n+3 n+p-l n+p
1 1 1
= ---- <-.
n n +p n
SEQUENCES 101
1 1 1 1 1
< 2n+1 + ... + 2n+p S 2n+1--I = 2n <e
1- -
2
for every n > [~~~e] + 1. This means that the sequence (bn)nEN is Cauchy's.
c) The sequence (Cn)nEN is Cauchy's, because for given e > 0 it holds
1 1 1
< (n + I)(n + 2) + (n + 2)(n + 3) + ... + -'--(n-+-p-)(-n + p + 1)
1 1 1
~----,- - < <e
(n+I) (n+p+I) (n+I) ,
d) In this case we shall show that the sequence (dn)nEN is not Cauchy's. This means
that there exists e > 0, such that for every no, there exist n > no and pEN
satisfying Idn+p - dn I > e.
Let us take e = 1/4. From
1 1 . 1 P
Id n+ -dnl = - - + - - + ... + - - > - - ,
p n+I n+2 n+p n+p
it follows that for n = p we have
1 1
Idn+p - dnl = Id2n - dnl 2: 2" > 4' for every n E N.
So the sequence (dn )nEN is not Cauchy's and does not converge.
102 CHAPTER 3.
e) The sequence (en)nEN diverges, because from the inequality In x < x, which holds
for x > 1, it follows that for c = 1/3 it holds
1 1 1 p P
Ie n+p -el-
n -In(n+1)
+
In(n+2)
+ ... +
In(n+p)
>
In(n+p)
>--.
n+p
For p = n we obtain
1 1
Ien+p - en I > "2 > 3"'
Therefore the sequence (en)nEN is not Cauchy's, hence it diverges.
an = (1 + ~r, (3.4)
a) is monotonically increasing;
b) is bounded from above with the number 3.
Solution.
a) Let us show that the sequence given by relation (3.4) is monotonically increasing
in two ways .
• First method. We shall consider the quotient ~ and show that it is less
an+l
than l.
an ( 1 + ;:;
1) n ( n + 1)
-n- n
1 1 n +1
an+l - (1 _l_)n+l = n+2 . n+2 - (n(n+2))n' n+2'
+n+1 n+1 n+1 (n+1)2
SEQUENCES 103
• Second method. Next we consider the difference an+! - an and show that
it is positive. We have
1 )n+1 ( l)n
an+! - an = ( 1 + n + 1 - 1 + :;;:
t ((
j=O
n -: 1 )
J
1 . _ ( n ) ~)
(n+l)J j nj
+ _1
(n+l)n+1
n 1{(
L"7j" 1)
1· 1 - - - . (1 - -2)
- ... (1 -j-
-l-)
j=l J . n+l n+l n+l
- (1 - ~)
n
. (1 - ~)
n
... (1 - ~)
n
}+ 1
(n+l)n+1
> 0,
n(n - 1) = ~ (1 _ ~) < ~,
2n 2 2 n 2
n(n - l)(n -
3! . n 3
2) = ~3! (1 _~)n (1 _~)n < ~3! '
we obtain
1 1 1 1 1 1
(1+~f < 1+1+-+-+···+-<1+1+-+-+···+-
2! 3! n! 2 22 2n - 1
1+ 1 - (~r < 1+ 2 ~ 3.
1--
2
r ~ (1 ~r 2,
Note that since (an)nEN is monotonically increasing, it also holds
an = (1 + ~ + = n EN.
Remark. From Theorem 3.32 it follows that the given sequence converges. Its limit
is the irrational number e (see Example 3.36).
Example 3.34. Determine the following limits.
a) lim
n--+oo
(1 + -n1r n ; b) If ;
lim(I--
n--+oon
. en + 3) 3n+2 .
r
c) hm - 2 - , d) lim (n+lf
-- ;
n
n---+oo n- 1
n-->oo
1 n2
--1
2
e) lim (n2 - f) lim (n---)
n-->oo n2 + 1 n-->oo n +1
In Vn+T - In yin.
g) lim , h) lim n . (In Vn+T -In yin).
n-->oo n n-->oo
Solutions.
b) lim (1 - -
1) = li..~ ( n1 )
n
n =. (
1
1) n 1
. lim - - 1
n-->oo 1 + __
1
n-->oo n -- hm 1 + --1 n _ 1
n- 1 n-->oo n-
1 1
-·1 = -
e e
¥.~
2n + 3 3n+2
c) n---+oo
lim ( -2n- ) = n---+oo
lim (1 + -2)
n . lim (1 + -)
3 3n
2n
n-+oo
3 2
= n--+oo
lim
(
1+ -2n
1 )
3
2n/3) 9/2
( lim
n-->oo
(1 + _1
2n
) .1 = e 9/ 2
3
SEQUENCES 105
( 1 +-l)n
d) lim (~)n _. (n (1 + ~))n
n-->oo n - 1 - J!...~ ( 7)
n 1--
lim n
n-->oo ( 1- - 1) n
n n
J~~ (1 + ~r
2
lim (1_1)(-n)(-l) =e •
n---+oo -
n
The last example can be done also as follows.
n+l)n
lim ( - -
n-->oo n - 1
= lim ( 1 + - - )n-l . lim ( 1 + - -
n-->oo n- 1 n-->oo n- 1
2 2)
. (
= hm 1 + - -
2) 2(n-l}/2
=
(. (
hm 1 + - -
2) (n-l}/2) 2
= e2 •
n-->oo n - 1 n-->oo n - 1
n2)
e lim
) "_00
n2-1
(n, +
-
1)
n2
= lim (
"_00
(1- 21 -)
(1+;!,
n
r 2 = e- 2 •
·
f) n--+oo
n- 1
hm (n -+)1
n2
.
= n--+oo
hm (
l)n)n
(( 1--
n
1 +-
n
1 r . -2 n
hm (e ) =
= n--+oo o.
h) · n· (lnJnTI -lny'n) = hm
hm . 1
-In (n+l)n
- - = 1/2.
n--+oo n--+oo 2 n
Example 3.35. Prove the following estimations.
Eo JT < k. k!'
k 1 1
0< e - kEN. (3.5)
J=k+l
~
L..J ~(
., 1 -
J.
1)(1 - -n2) ... (1 -
-
n
j-l)
- - .
n
106 CHAPTER 3.
+ (k+1)(k+2) k ) ( 1 -k+1)
1 ... (k+n)· (1-;;: -n- .. · ( 1 -n-1)}
-n- .
Thus we obtain
t
j=k+l
(~)~=
) nJ
t
j=k+l
n(n-1)(n-2) ... (n-j+1).~
j! nj
1( k+1
< k!
1 + (k+1)2
1 + ... + (k+1)n-k
1) < k!'
1 k+1'1_~
1 1
k+1
So it holds
Ln (n)
. 1
j<
1
k.k'· (3.7)
j=k+l ) n .
From relations (3.6) and (3.7) we obtain for 0 < k < n :
1 = k -.
1
lim
n_oo L )~
k
J=O
( )
~
nJ L).!
J=O
(3.9)
~ (1 - ~) (1 _~) ... (1 _j : 1) ,
it follows that for fixed j E N it holds
lim (
n_oo
~)~
) nJ
~, '
= ).
SEQUENCES 107
· ~
11m 1 = 1·1m (1 + -1 + -1 + ... + -1 ) = e.
L.J -:-
n--+(X).J'.
3=0 n--+(X) l'. .2' • n'
Example 3.36. Prove that the real number e is not rational.
Solution. Let us suppose the opposite, i.e. that e is a rational number. Then it
can be written as e =~, where p, q E N, 2 < ~ < 3 and lcd(p, q) = 1. As
q q
usual, lcd(p, q) stands for the largest common divisor of the numbers p and q. If in
relation (3.5) we put k = q, then we obtain
1 1 1) <_.1
O<e- ( 1+1+-+-+···+-
2! 3! q! q. q!'
or
o < q [e q! - q' + 3i
( q! + q! + 2i q' + ... + q')]
qi < 1.
Taking e = ~ , we have
q
q! q! q!
p(q - I)!, q! and q'. + q'. + -2! + -3! + ... + -q!
are integers and therefore the expression
Remark. e ~ 2.718281828459045.
. 1
Example 3.37. Prove that hm
n~oo
nCi =
vn!
o.
Solution. Using mathematical induction, let us show first
For n = 1 the inequality is correct. Let us suppose that it is correct for n = k, i.e.,
(1 + ~f'
(3.12)
So the relation (3.11) is correct. Therefore for given s > 0 one can find no such that
it holds
1
1 I
( 1 )n+l
an 1 +; ( 1 )n+l n+1
a n +l 1 + _1_) n+2 = 1 + -n-:-(n-+-2-) n+2
(
n+1
> ( 1+ n +1 ) n +1
.--=
n 3 + 4n 2 + 4n + 1
>1.
n(n+2) n+2 n 3 +4n 2 +4n
SEQUENCES 109
This means that an > an+1, so the sequence (an)nEN is monotonically de-
creasing.
b) The sequence (an)nEN is monotonically decreasing and bounded from below (for
example with zero), and therefore from Theorem 3.32 it follows that this is a
convergent sequence and its limit can be ordered as
lim
n-+oo
(1+-n1)n+l = n-+oo
l ) n . ( 1+- =e.
lim (1+-
n
1)
n
l)n
( 1+; (l)n+l
<e< 1+;
In (1 +~)
n
< ~
n
and In (1 +~) >
n
_1_.
n+1
So we can write
-1- <In (1+-
1 ) <-.
1
n+ 1 n n
d) Using the inequality proved in c), we obtain the following inequalities for n > 1.
In (1 +~) <
1
n
< In (1 + _1_)
n-1
,
In(1+ n :1) < n+1 < In
1
(1 +~) ;
In C2: 1)
n <
1
2n
< In(~).
2n -1
Summing the previous inequalities, we get
L2n In (1
1 + k) = In
2n
II (1 + ~) = In (n + 1 . n + 2 ... 2n + 1)
k=n k=n k n n +1 2n'
110 CHAPTER 3.
we obtain
In (2n n+ 1) < f k=n
~ < In (~) .
k n - 1
The logarithmic function is continuous on the interval (0, +00) and therefore
we have
lim In
n--+oo
(2n n
+ 1) = lim In (~) = In2.
n--+oo n - 1
Using Theorem 3.7 we obtain finally
2n 1
lim
n--+oo
E -k = In2.
k=n
an - a n +l = In (1 + ~) - _1_ > 0,
n n+1
an
n 1
E--lnn>Eln 1+- -Inn
n ( 1)
k=l k k=l k
In (2 . ~ . ~ ... n + 1 . ~) = In n + 1 > 0 .
2 3 n n n
From Theorem 3.32 it follows that the sequence (an)nEN converges and has a limit
lim an =: T' The limit T is called Euler's constant.
n--+oo
r
b) --<In(l+r)<r, rEQ, r>-l.
l+r
Solutions.
SEQUENCES 111
a) The proof of the left-hand side inequality is essentially the same to the proof of
relation (3,11) in Example 3,37, So let us prove the right-hand side inequality,
To that end, using the mathematical induction and the fact that (1 + ~) > 2 n
(n;lf
for n > 1, it follows
n!<
Further on we have
(n+1)n (l)n
( ~)n = e (!.?:)n, - 2 = e (!.?:)n, 1 + ~ < e (!.?:)n
2 2 e (%f 2 e 2
In (1 + ~)q + In (1 + _1_)
q+1
+ ' " + In (1 + q+p-1
1 )
1 I I p
< -+--+'''+
q q+1 q+p-1
<-,
q
Thus
In(l +r) < r, (3,13)
Also, from Example 3,38 c) it follows
P
In(l + r) > -
1
- +-
IIp
+'" + -q+p > -q+p = - - = --,
q r
(3,14)
q+1 q+2 E+1 r+1
q
If we take -1 < r < 0 and r E Q, then we can put r = -rl, where 0 < rl < L
Denoting by r2 = _r_l-, it follows that r2 E Q and r2 > 0, From relations
1 - rl
(3,13) and (3,14) we obtain
r2
- - < In(1 + r2) < r2'
1 + r2
rl < In ( 1 + -r-
l ) < -rl
-,
1 - rl 1 - rl
Then it follows
1+ 1-
-r- = -
-rl
1+
- < -In (
r
- - = In(l -
rl )
rl 1- rl
rl) = In(l + r) < -rl = r,
and these relations immediately give the inequalities in b),
112 CHAPTER 3.
1 (2) a
b) In+! = 2 a + In , ft = 2' 0 < a ::::; 1;
e) In+!=ln-sinln, 0::::;ft<1r.
Solutions.
a) Using mathematical induction, we shall prove that this sequence is monotonically
increasing and bounded from above.
It is obvious that it holds h = Je+..fi > ft· Assume In> In-I. Then we
have
Je + In> Je + In-l =? In+! > In·
Next we show that for all n E N it holds In < ..fi + 1. Firstly we have
ft = ..fi < ..fi + 1. If we suppose In = ..fi < ..fi + 1 for some natural number
n, then it follows
I; = e + In-l lim
=? n---+oo I; = c + lim In-l
n---+oo
=? £2 = e + £,
and it has the form
£ _ 1± v'I+Tc
1~ - 2 .
All terms of our sequence are positive and therefore the number
R
q =
1 - ../1 + 4c <0
2
can not be the searched limit. Hence the limit of the sequence (fn)nEN is
£ = £2 = 1 + v'I+Tc
2 .
SEQUENCES 113
b) We shall show that the sequence (fn)nEN is monotonically increasing and bounded
from above. It holds that in > 0 for every n E Nand
2)) a 1 (a)2 a
i2 = 21 ( a + (a4 = 2 + 2 2 > 2 = It.
lim
n--+-oo
in = £ = lim
n--+CX)
in-I'
The limit £ can be ordered from
1
£= 2 (a + £2) =} £1,2 = 2 ± J4=4a
So the limit of (fn )nEN IS
. 2 .
£=£2 = 1-~.
In this case we could not accept the number £1 = 1 +vr=-a > 1, (for 0 < a <
1) as the limit of (fn)nEN, because we have shown in < 1, for every n E N.
in = 2n!tl
-
and lim
n->oo
in = O.
b b
2in+l = in + in =} 2in+1 - 2Vb = in - 2Vb + in
we can conclude that fn > 0 for n > 1. Using the last inequality we get
fn+1 - fn = 'l(b
2 fn - fn
) =
b-F:' < 0,
'12 . ~
fn+! = ~ (fn + YJ .
Thus £= ~ (£ + D> O. Since fn > 0 for all n E N, we get finally
£= 0.
• Case 3. If we have b > 0 and f1 < 0, analogously it can be shown that
the sequence is monotonically increasing and it holds that fn < 0 for
every n E N. This means that the considered sequence converges with
the limit x = -0.
Remark. In the Cases 2 and 3, the sequence is not decreasing (resp. increasing),
but it is rather eventually decreasing (resp. eventually increasing); i.e., the sequence
becomes monotone starting from an index no. This situation happens, for instance,
when in Case 2 it is assumed 0 < II < 0.
d) In Example 1.33 we have shown that the arithmetic mean is not smaller than
the geometric mean of any finitely many positive numbers. Thus
b
(m - l)fn + jm-1
> m/ jm-1 . _b_ = Vb
fn+! = m n
Vn f~-l
fn+! - fn =m 1( b
f:::- 1 - fn
) b-f:::
= mf:::- 1 s:; o.
From the last relation it also follows that the sequence is monotonically de-
creasing and therefore it converges. The function sin x is continuous and the
limit i can be ordered from the equation
i=i-sini, (3.15)
Equation (3.15) has two solutions on the interval [0,7r], namely i1 = 0 and
i2 = 7r. From the inequalities fn :::; it < 7r it follows that the limit is i = i1 = O.
Exercise 3.42. Prove that the sequences given by the following recurrence formulas
converge and determine their limits.
1
c) cn +! 1 - c~, C1 = 2' d) dn+! = dn (1 - dn), 0< d1 < 1.
Answers.
a) 2. b) 5. c)VS- I . d) O.
Example 3.43. Prove that the sequences (fn)nEN and (gn)nEN given by
Solution .
• First method. It is obvious that fn > 0 and gn > 0 for every n E N. Also it
holds that fn+! ;::: gn+! for every n E N (comparison between the arithmetic
and the harmonic mean - see Example 1.33). Therefore we have
f n+1 - f n = fn +
2
gn _ f
n
= gn - fn
2
<0
-, for every n> 2,n E N.
The sequences (fn)nEN and (9n)nEN are bounded from below, resp. above,
because
fn+! 2': 9n+l 2': ... 2': 92 and 9n+!:::; fn+! :::; ... :::; h,
for every n E N. So these two sequences converge. Let us denote
f = lim fn
n~oo
and 9 = n--+oo
lim 9n·
Then we have
lim fn+l
n-+oo
= ~2 (lim
n-+oo
fn + lim 9n )
n-+oo
=;. f =f+
29 =;. f = 9.
From the equality
2fn9n
9n+l - fn + 9n
- ,
it follows
fn+19n+! = fn9n = fn-19n-l = ... = 1t91'
and we obtain
it follows
1 n+l
= fn
2
+ f191
2fn
=~
2
(I +
n
f191)
fn .
Theorell1 3.45. Any subsequence of a convergent sequence converges and they both
have the same limit.
SEQUENCES 117
A subsequence (fnJEN of a sequence (fn)nEN which has the property fnk = £ for
some number £ and for all kEN is called stationary subsequence. Clearly,
then £ is an accumulation point of the sequence (fn)nEN.
Definition 3.46.
I = nlim
..... oo
(inf{ am 1m;?: n}).
s= nlim
..... oo
(sup{fml m;?: n}).
Each of the last two limits either exist in the (usual) sense of Definition 3.1, or is
+00 or -00 (see Definition 3.2). In any of these cases we shall write
Let us note that the limes inferior and the limes superior of a sequence are resp.
the smallest and the largest accumulation point of that sequence.
Solution. If the set MI = {nl fn = £} is infinite, then its elements can be ordered
into a monotonically increasing sequence which diverges to +00. So we can put
fnk = £ for kEN and nl < nz < ... < nk < ... -+ +00. This means that for every
mEN there exist nk E M I , such that it holds nk > m. Then, for arbitrary e > 0,
we have
Ifnk - £1 = 1£ - £1 < e.
We obtain that from condition (ap1) it follows that £ is an accumulation point for
the sequence (fn)nEN.
If for every e > 0, the set (£ - e,£ + e) n Unl n E N} is infinite, then the set
Mz = {nl Ifn - £1 < e} can be ordered into a monotonically increasing sequence
nl < nz < ... which diverges to +00. This means that, for every mEN, there exists
kEN, such that nk E M z and nk > m. So for arbitrary e > we have °
Ifnk - £1 < e.
We obtain that from condition (ap2) it follows that £ is an accumulation point for
the sequence (fn)nEN.
118 CHAPTER 3.
Solution. Let us suppose that £ is an accumulation point for the sequence (fn)nEN
and that the condition (apl) is not fulfilled (this means that the sequence (fn)nEN
has no stationary subsequence (fnk )nEN with the property (fnk hEN = £, for every
kEN). We have to show that then condition (ap2) is fulfilled.
Let us suppose that this is not true, namely that there exists an 6 > 0 such
that the set (£ - 6, £ + 6) n Un I n E N} is finite. If n1 is the smallest natural
number such that n > n1 => fn f. £, then the supposition means that the finite set
(£ - 6, £ + 6) n Unl n E N} is either empty, or there exists a natural number n2 > n1
such that for every n > n2 => fn (j. (£ - 6, £ + 6). Hence this means that £ is not an
accumulation point for the sequence (fn)nEN, a contradiction.
Remark. The statements in Examples 3.47 and 3.48 can be together expressed as
follows.
A real number £ is an accumulation point for the sequence (fn)nEN if and only if
either the sequence (fn)nEN has a stationary subsequence whose every element is
equal to £, or every interval containing £ has an infinite number of terms of the
sequence (fn)nEN (or both).
Solutions.
3k
g3k = 3k + 1 sin(27rk) = 0
converges to 0 and this means that 0 is an accumulation point for the sequence
(gn)nEN. Further on, we have
·
11m +
3k 1 . (k
= l'1m -k--sm 2 7r + -27r) V3
=-.
k-->oo 3 + 2
93k+1
k-->oo 3 2
SEQUENCES 119
3k+2 . (
g3k+2 = 3k + 3 sm 2k1r
41r)
+ :3
it follows that - V; is the third accumulation point for the sequence (gn)nEN·
As another example of a sequence with three accumulation points, one can
n1r
take the sequence with the general term h n = cos 2' n EN.
c) We shall use the sequence whose accumulation point is every positive rational
number given by the following scheme
Example 3.51. Assume that I = liminf an is a real number. Prove that then I
n-+oo
is the smallest accumulation point of the sequence (an)nEN.
From the definition of the sequence (fn)nEN, it follows that for every n E N there
exists a number mn = mn(c) 2: no satisfying In ::; a mn ::; In + c. The numbers
mn can be chosen so that the sequence (mn)nEN becomes monotonically increasing.
Then for every mn it holds
I - c ::; a mn ::; I + c.
So I is an accumulation point of the sequence (an)nEN; we have yet to show that it
is also its smallest accumulation point. From the inequalities
and since c > 0 is arbitrary, it follows that I is the smallest accumulation point for
the sequence (an)nEN.
Remarks.
1. If the sequence (an )nEN is not bounded from below, it is obvious that
liminf an = -00.
n--+oo
liminf
n--+oo
an = +00.
2. In the previous example it was shown that the point I satisfying I = liminf an
n--+oo
is the smallest accumulation point of the sequence (an)nEN. So we can prove
that in this case for every c > 0 it holds that
3. Similarly as in Example 3.51, it can be shown that the point S satisfying that
S = lim sup an is the biggest accumulation point of the sequence (an)nEN. So
n--+oo
we can show that in this case for every c > 0 it holds that
Example 3.52. Determine lim sup fn and liminf fn for the following sequences.
n-+oo n-+oo
1
a) fn = [( _1)n -I)n + -, n E N;
n
b) fn = a + bn(-l)n
1 + n(-l)n, a < b, n E N·,
n2 • 21m
c) fn = -1--2 sm-, n E N;
+n 3
d) fn = 1 + (_I)n+! + 3(_I)n(n-l)/2, nEN.
Solutions.
from f2k+! = [( _1)2k+! - 1)· (2k + 1) + - 21+k1 ,it follows lim f2k =
k--+oo
-00.
Thus it holds
lim sup fn = 0 and liminf fn = -00.
n--+oo n--+oo
b)
a + b· 2k
hk= 1. nl , k=I,2, ... , lim f2k = b=limsupfn;
k-+oo n-+oo
b
a+--
f2k-l = 2k 11, k = 1,2, ... , lim f2k-l = a = liminf fn.
1 + __ k--+oo n--+oo
2k -1
y'3(3k - 2)2 . y'3.
c) f3k-2 = 2(1 + (3k _ 2)2)' k = 1,2, ... , t:'I~,f3k-2 = 2 = h~~p fn;
_ -y'3(3k - 1)2 . -y'3. .
hk-l - 2(1 + (3k _ 1)2)' k = 1,2, ... , 1:.~ hk-l = -2- = h~~f fn;
(3k)2 .
hk = 1 + (3k)2 ·0, k = 1,2, ... , kI:.~ f3k = O.
d) f4k = 1 + (_1)4k+! + 3(_1)4k(4k-l)/2 = 3,
14k+! = 5 = limsupfn,
n--+oo
f4k+2 = -3 = liminf
n--+oo
fn and !4k+3 = -l.
Exercise 3.53. Prove that a necessary and sufficient condition for a sequence (fn)nEN
to converge is
lim sup fn = liminf in = lim in = i. (3.16)
n-+oo n-+oo n-+oo
Answer. The statement immediately follows from Theorem 3.5, which claims that
a sequence converges iff it is both bounded and has exactly one accumulation point.
122 CHAPTER 3.
Example 3.56. Let us suppose that (an)nEN and (bn)nEN are bounded sequences
and satisfy the following condition:
lim inf an ::; lim inf bn and lim sup an ::; lim sup bn .
n-too n-too n-+oo n--+oo
and suppose the opposite, i.e., a > b. This means that there exists a real number
d > 0 such that d = a-b.
Since a is the smallest accumulation point of the sequence (an)nEN, then there
exists nl E N such that it holds
d
an > a - 3 for all n > nl'
Since b is the smallest accumulation point of the sequence (bn)nEN, then for infinitely
many indices n E N it holds
d
bn < b + 3'
Hence for infinitely many indices n E N it holds
d d
bn < b + 3 < a - 3 < an, hence bn < an-
Answers.
b) inf{xnl n E N} = liminf
n-+oo Xn = 0, Sup{xnl n E N} = limsupxn = 2.
n-+oo
Example 3.58. If (fn)nEN is a sequence of positive numbers, prove then the fol-
lowing relations.
1 1
a) limsupfn = -liminf(-fn); b) lim inf -1 = lim sup fn
n-+oo n---+oo
n---+oo n n---+oo
Solutions.
a) Let us denote
limsupfn = L, L E R. (3.17)
n-+oo
Then for every c > 0, there are
liminf(-fn)
n-+oo = -L.
If, however, lim sup fn = 0, then for every c > 0, there are
n-+oo
124 CHAPTER 3.
•
. fi mte
III
. Iy many terms -I
1 such t h at -1 > -,
1
n fn c
• at most finitely many terms fI such that II < ~.
n n c
So we have
1 __ 1
liminf -I =
n---+oo n
+00 - limsupfn
n---+oo
Example 3.59. If (In)nEN and (gn)nEN are two sequences of real numbers, then
the following inequalities hold.
lim inf fn
n-+oo
+ lim sup gn
n-+oo
a) liminf fn + liminf gn :::; liminf(Jn + gn) < {
+ lim inf gn
n---+oo n-+oo n---+oo
lim sup fn
n-+oo n---+oo
Solutions.
a) Let us denote by
f = liminf fn,
n-+oo
9 = liminf gn
n-+oo
and £ = liminf(Jn
n-+oo
+ gn);
F=limsupfn, G=limsupgn and L=limsup(Jn+gn)'
n---+oo n-+oo n-+oo
1. First we shall prove that f +g :::; £. If we suppose the opposite, i.e., £ < f +g,
then it holds d := (J + g) - £ > O. For every c > 0, there are
Since L is the greatest accumulation point of the sequence (in + gn)nEN, there
are
Determine the smallest and the largest accumulation point for every sequence and
determine
limsup(fn
n->oo
+ gn), liminf(fn
n->oo
+ 9n), limsup(fn . 9n),
n->oo
liminf(fn
n->oo
. 9n).
Answers.
a) liminf fn = -1,
n->oo
liminf
n->oo
9n = -1, liminf(fn
n->oo
+ 9n) = -1,
limsupfn = 1,
n->oo
limsupgn = 1,
n->oo
limsup(fn
n->oo
+ gn) = l.
1 3
b) liminf fn = 1,
n->oo
liminf
n->oo
gn = -2' liminf(fn
n->oo
. 9n) = 2"
7 9
lim sup fn = 3,
n->oo
limsuP9n
n-+oo
= 2' limsup(fn . 9n) =
n->oo
2'
SEQUENCES 127
Example 3.61. Two sequences (Xn)nEN and (Yn)nEN are given and the first one
converges, i.e., there exists an x, such that x = lim Xn . Then it holds
n->oo
a) liminf(xn
n-+CX)
+ Yn) = n-+oo
lim Xn + liminf
n-+oo
Yn;
Solutions.
lim inf Xn
n--+oo
+ n-+oo
lim inf Yn:S: lim inf(xn
n-+oo
+ Yn):S: lim sup Xn
n-+oo
+ n--+oo
lim inf Yn
lim Xn = lim
n-+oo
inf Xn = lim sup Xn
n--+oo n->oo
Solution. We shall prove only the first relation. Since i := inf {xn I n E N} is the
largest lower bound of the last set, it holds that i :s: Xn for every n E N. Therefore
we have
lim (inf{xml m ::::: n}) = liminf
i:S: n--+oo n--+oo
Xn .
Example 3.63. Let us suppose that the sequence (fn)nEN has two accumulation
points a and b and assume lim inf fn = -00 and lim sup fn = +00. Prove that then
n--+(X) n--+oo
there exist subsequences (nl)IEN, (nkhEN, (np)PEN and (nq)qEN of the set N such
that
lim fn!
1-+00
-00, lim fnk = a, k--+oo
lim fnp
p--+ 00
=b and lim fnq
g--+(X)
= +00.
Solution. We shall construct only two subsequences of (fn)nEN converging to a
and +00 respectively. First put
M = {n E NI fn = a}.
128 CHAPTER 3.
If the set M is infinite, then it can be written in a unique way as an increasing se-
quence (nk)kEN which diverges to +00. Clearly, (fnk)kEN is a stationary subsequence
of (fn)nEN which converges to a.
If, however, the set M is finite or empty, then there exists an no E N such that
('tin E N) n > no ::::? fn i- a.
Put n1 := no + 1 and C1 := IfnI - al/2. Since a is an accumulation point of (fn)nEN,
there exists a natural number n2 > n1 such that
fn2 E (a-Cl,a+cl).
Putting C2 := Ifn2 - al/2, there exists a term f n3 in the interval (a - C2, a + C2) ,
etc. Continuing this procedure ad infinitum, we obtain a subsequence (fnk) kEN of
the sequence (fn)nEN which converges to the point a.
A subsequence (fnq)qEN which diverges to +00 can be obtained from the fact
that the sequence (fn)nEN is not bounded from above. Namely, denote by nl the
smallest natural number such that fnl > 1. Next, put n2 for the smallest integer
greater than fnl + nl' Continuing in this manner we get a monotonically increasing
sequence (nq )qEN which diverges to infinity. This sequence is the set of indices of a
monotonically increasing subsequence (fnq)qEN which diverges to +00.
Exercise 3.64. If a sequence (Xn)nEN is bounded from below, then there exists
lim (inf{xml m ~ n}).
n ..... oo
Prove.
Example 3.65. Let (fn)nEN be a sequence and denote by
f1 + h + ... + fn
F n := , n E N, (3.23)
n
its sequence of arithmetic means. Prove then the following implication.
lim fn
n~oo
=f ::::? lim Fn
n--+-oo
= f.
Solution. From the condition nlim
..... oo
fn = f, it follows that
1
< -(If1 - fl + 112 - fl + ... + Ifno-1 - fl + Ifno - fl + ... + Ifn - fl)
n
A n - no A
< - + --- . - < - + -,
C C
n n 2 n 2
SEQUENCES 129
Gn := n-+oo
lim ylglg2'" gn (3.24)
lim gn = 9
n---+oo
=> n--+oo
lim G n = g,
Since it holds
1
InGn = -(lngl + Ing2 + ... + lngn),
n
we can apply the previous example and obtain
lim In G n
n--+oo
= In g, hence finally lim.Jgl ... gn
n---+oo
= g.
Remark. The opposite does not necessarily hold. Find an example!
Example 3.67. Prove that if a sequence (gn)nEN has the property
lim fn = n--+oo
n-+oo
lim (gn - gn-l) = 9
and let us calculate the sequence of arithmetic means of the sequence (fn)nEN. We
have
Fn = h + h + ... + fn = gl - go + g2 - gl + g3 - g2 + ... + gn - gn-l = gn.
n n n
From Example 3.65, it follows that
· -gn = l'1m 1'n
11m L'
= g.
n-+oon n--+oo
130 CHAPTER 3.
Example 3.68. Prove that if a sequence (hn)nEN of positive numbers has the prop-
erty
lim hhn
n---+oo n-l
= h, then lim
n---+oo
~ = h.
G n = \/gl··· gn = rj _
hl
ho
... --
hn
hn-
=
1
nhh
V nn
nfh;.
lim V Il n
n---+oo
= h.
Example 3.69. Determine the limits of the following sequences, given with their
general terms.
a) fn = W. b) gn =
\j(kn)!
nL.
kEN;
n '
Xn
n!
nn
(n - I)!
n(n - l)n-l = (n: 1 1 r- 1
1 )n-l.
Xn-l
(n - l)n-l ( 1+--
n-1
So we have lim ~ = ~e
n---+oo Xn-l
and from the previous example it follows
W 1
lim - - =-.
n-+oo n e
. (kn)!.
b ) In thIS case we put Yn = -k- and obtam
n n
(kn)!
Yn nkn (n - 1)kn kn· (kn - 1) ... (k( n - 1) + 1)
Yn-l (k(n - I))! n kn · (n-1)k
(n - 1 )k(n-l)
4
c) lim h n = -.
n ..... oo e
Example 3.70. Let the sequence (Fn)nEN be the sequence of arithmetic means of a
sequence (fn )nEN. Prove that then it holds
liminf fn :::; liminf Fn :::; lim sup Fn :::; lim sup fn.
n~oo n~oo n---+oo n-+oo
Let us denote by f = lim sup fn and assume that f ~ O. Then for every c > 0,
n ...... oo
there exists no = no (c) such that for every n > no it holds fn < f +c. So for n ~ no
we have
fl + h + ... + fno + ... + fn
Fn
n
Exercise 3.71. Let us suppose that (Pn)nEN is a sequence of positive numbers such
n
that nlim
...... oo
L Pk = +00, and let (an)nEN be a sequence of real numbers. Put
k=l
(Fn is called the sequence of generalized arithmetic means of the sequence (an)nEN')
Prove then the following.
lim inf an :::; lim inf Fn :::; lim sup Fn :::; lim sup an.
n---+oo n---+oo n---+oo n--+oo
132 CHAPTER 3,
Thus if the sequence (an)nEN converges, then its sequence of generalized arithmetic
means converges to the same limit,
' -
l 1m Qn l' Qn - Qn-l ,
= 1m
n-+oo P n n-+oo P n - P n- l
Solution. Using the conditions on the sequence Pn , we can construct the following
two sequences
If there exists lim an = £, then from the previous exercise it follows that the se-
n-+oo
quence of generalized arithmetic means converges to the same limit, i,e"
' Qn
I1m 0
n-+oo -p =.c
n
Example 3.73. Prove that the statement opposite to the one from the Stolz theorem
is not necessarily true, In other words, even if the limit
' Qn - Qn-l
l 1m
n-+oo-=-'------=:-----
P n - P n- l
might not exist,
, , I'1m Qn - Qn-l
However, t h e Ilmlt p d oes not eXIst,
' b ecause
n---;(XJ Pn - n-l
, ' f Qn
I lmln p - Qn-l
p = I'lmln ' fsln- J3
' mf = --,
n---;(XJ n - n-l n---;oo 3 2
,
IImsup Qn - Qn-l
p p = I'lmsupsm J3
- = -,
,mf
Example 3,74. Determine the following limits by using the Stolz theorem,
1 1
k 1+-+ .. , + -
1k + 2k + ' , , + n kEN i b) 9n = V2 Vn,
a) fn = nk+'
n '
1k + 2k + ' , , + n k _n kEN,
e) Yn = nk - k + 1'
Solutions.
, Qn - Qn-l
I1m nk
= lim ---:---,-------,--
Pn - Pn - l
n---;(XJ n---;(XJ nk+ l - (n - l)k+1
nk 1
= lim --,-----0-----,--,----:---0-------,----0--0--
n---;oo n k+l - n k+l + (k + l)nk -'" - (_l)k+l k+1
we obtain that , Qn
I1m I' f 1
n---;oo 1m n = -+
-P = n---;oo k I'
n
1 1 1
b) If we put Qn = 1 + V2 + J3 + '" + Vn and Pn = n, n E N, then we have
1
Vn- -- 0,
, 9n -- I'1m -Qn -- I'1m Qn - Qn-l -- -
I1m
n---;(XJ n---;(XJ Pn n---+(XJ Pn - Pn - l 1
2P
lim h n = p + 1
c) n---;oo
134 CHAPTER 3.
. Qn - Qn-l
11m = lim In n - In( n - 1) = lim In ( 1 + ~)
n-->oo Pn - Pn - l n-->oo n l / k - (n -l)1/k n-->oo ( ( l)l/k)
nl / k 1- 1- -
n
< E,n;, n -1
1
n'" (1- (1-;;) 1 l/k)
r 1 1 k-l ( 1 j/k
n~~ n _ 1 . n(l/k)-l L
)=0
1 - -)
n
= O.
Thus from Theorem 3.7 it follows that lim Qn - Qn-l exists and
n-->oo Pn - Pn - l
Example 3.75. Let two sequences (an)nEN and (bn)nEN be given and let us define
the sequence (Cn)nEN by
Cn =
a1b n + aZbn - l + ... + anbl , n = 1,2, ....
n
lim an = 0 and Ibnl:::; B for every n E N, then it holds
a) If n-+oo lim Cn = O.
n-+oo
b) If n-+oo
lim an = a and lim bn = b,
n-+oo
then it holds lim Cn = a . b.
n-+oo
Prove.
Solutions.
a) From the implication
lim an
n-+oo
= o:::} n-+oo
lim lanl = 0,
and the Stolz theorem, it follows
lim
n-+oo
t lakl =
~n k=l lim lanl
n-+oo
= o.
SEQUENCES 135
Since the sequence (bn)nEN is bounded, i.e., Ibnl ~ B for every n E N, we have
xlbn+···+xnbl bn+···+bl
~-~~+a· =fn+9n.
n n
From a) it follows that
lim fn = l.
n--+<XJ 9n
Then we write
fn rv 9n as n -+ 00.
Definition 3.77. Assume that (9n)nEN is a sequence with positive terms. We say
that
r in 0
n~~ gn = ,
In particular, if the sequences (fn)nEN and (gn)nEN diverge to +(X) and satisfy
the condition
fn = o(gn) as n -+ (X) -¢:=} lim fn = 0
n-+oo 9n '
then we say that the sequence (gn)nEN diverges faster to infinity then the sequence
(fn)nEN and we write
fn -< gn, as n -+ (X).
a) fn = n 4 + 3n 2 + 2 and gn = n4 ;
n
b) fn = ~ and 9n =-,
e
thus
n! '" nne- n J27rn as n -+ (X). (3.25)
This implies
Solutions.
Inn Inn
-
n a <n-
1/ q •
In(n+l)-lnn
-,----'---:--;-----:-;- =
In(I+~)n
n <
q(n+1)(q-1)/q ( l)n
In 1 + - ,
(n + 1)1/q - n 1/ q n((n + 1)1/q - n 1/ q) - n n
it follows
. In(n+l)-lnn
11m = O.
n-+oo (n + 1)1/q - n 1/ q
The last inequality we obtained from the following identity
which implies
1
yin +1 - y'n:::: q1(n + l)q-1
· -In1n/ = 0, . In n
11m an d t h·IS imp1·1es 11m - - = O.
n q
n---+oo na
n--+oo
Remark. Using this example, we can form a (partial) scale of growths for sequences
diverging to infinity as follows.
Inn --< n a --< n b --< qn --< n! --< nn, 0 < a < b, q> 1.
Example 3.80. If (fn)nEN is a given sequence such that fn > 1 and lim fn =
n-+oo
+00, then it holds qin >- fn' for q > 1.
138 CHAPTER 3.
Solution. Since Un] :s; in < Un] + 1 and qfn ~ qUnl, then
in
-qfn < "'---;-:-:-
[in] +1 nk +1
qUnl qnk
.
T h e sequence gIven by - +-
nk- 1 IS
. a su b sequence 0 f t h e sequence gIven
. by n + 1
qnk qn
From the limit
lim n+1 =0
n->= qn
it follows that
.
11m nk 1
--- =
+ 0 I.e.,
.
hm - j =0.
qfn
n--+CX) qnk ' n->= n
From this example we can form another scale of the growth of sequences which
diverge to infinity,
nG
na -< qn -< qq -< ... , a> 0, q> 1.
G
b
because lim ~ = 0, lim an = +00, we obtain
n-+oo an n--+oo
. qb n
hm - =0.
n-+oo qan
From this example we can again form the following scale of growth for the sequences
which diverge to infinity
qfo -< qn -< qn2 -< ... , q > 1.
Example 3.82. Ii (fn)nEN is a given sequence such that in >1 and lim in
n->=
= +00,
prove then
In in -< in.
Solution.
In in In(Un]+l) In(nk+1)
--< = .
in [in] nk
. In(nk + 1) . . In(n + 1)
The sequence gIven by IS a subsequence of the sequence gIVen by .
nk n
From Example 3. 79 e) it follows that
Example 3.83. If the sequences (fn)nEN and (fn)nEN satisfy fn > 0, 9n > 0,
prove then
for some K > 0, M > 0 and n > no, for some no E N. Thus it holds for n > no
lim an + bn = lim ( an + bn ) = O.
n---+oo fn + 9n n---+oo fn + 9n fn + 9n
So we obtain o(fn) + 0(9n) = o(fn +9n).
140 CHAPTER 3.
Exercise 3.84. If the sequences (fn)nEN and (9n)nEN satisfy fn > 0 and 9n > 0,
n E N, prove then
a) G(fn) + O(9n) = G(fn + 9n); b) G(fn)' G(9n) = G(fn . 9n);
Solution. The limit lim fn = 1, means that for every c > 0, there exists no = no(c)
n-+oo 9n
-
such that it holds
I~: 11 < c, for every n > no·
Thus we have
-c < fn - 9n < c, .
meamng l'1m fn - 9n 0
=, hence fn - 9n = 0 (9n ).
9n n-+oo 9n
Exercise 3.86. Prove that fn rv 9n as n --t 00, if the sequences (fn)nEN and (9n)nEN
are 9iven as follows.
) _ 2n 3 + 3n 2 + n +1 _ ~ .
a fn - 3n 2 + 1 ,9n - 3 n ,
n2 •
b) fn=I+2+3+"'+n, 9n= 2'
c) fn = ifr0 + n + 2, 9n = nifii;
2Pn P+1
d) fn=lP+3 P + ... +(2n+l)P, 9n= p+l' pEQ.
Exercise 3.87.
a) Does a "fastest" sequence (9n)nEN exist, i.e., such that
lim fn = 0, for every (fn)nEN?
n-+oo 9n
Answers.
a) No, because it always holds fn -< f::.
Limits of functions
4.1 Limits
4.1.1 Basic notions
For the definition of the limit of a function at a point, we need the notion of the
accumulation point of a set, see Definition 1.76 c).
lim f(x) = L.
x--+xo, xEA
Note that we do not need the function f to be defined in the point Xo. Even if
Xo E A, the value of f at Xo is irrelevant.
Using logical symbols, Definition 4.1 can be expressed as follows.
lim f(x) = L
x--+xo,xEA
lim f(x n ) = L.
n-+oo
141
142 CHAPTER 4.
If in Definition 4.1 we take only the values x E A greater (resp. smaller) than Xo,
we get the definition of the right-hand limit (resp. left-hand limit) of a function
f: A ~ R at the point Xo. The right-hand limit of f at Xo is denoted by
lim f(x),
x--+xo+, xEA
lim f(x).
X--+xo - , xEA
for the limit, right-hand limit and the left-hand limit of the function f at the point
Xo·
Definition 4.4 .
• Assume the domain A of the function f : A ~ R contains the interval (a, +(0)
for some a E R. The number L is the limit of f at +00 if for every c > 0
there exists a number T > a, T = T(c), such that for every x > T it holds
1f(x) - L 1< c. Then we write
lim f(x)
x--++oo
= L. (4.2)
In Definition 4.4 x went in the first case to plus infinity through increasing
positive values, while in the second case it went to minus infinity through decreasing
negative values.
LIMITS OF FUNCTIONS 143
Definition 4.5. Assume the domain A of the function f contains the interval (xo, b)
and assume that for every T > 0, there exists a 8 > 0, 8 = 8(T), such that for every
x E A and x E (xo, Xo + 8) it holds f( x) > T. Then we say that the function f
tends to plus infinity when x --t xo+, and write
lim f(x) = -00, lim f(x) = +00 and lim f(x) = -00.
+
x---+xo X---+XO - x--+xo-
Relation of the limit of a function with the basic operations and inequalities gives
the following statement.
Theorem 4.6. Assume the functions f and 9 are defined on a set A c R and let
Xo be an accumulation point of A. Moreover, assume that the following two limits
exist:
lim f(x) = L and lim g(x) = K.
X---+XO, rEA X---+XO, rEA
Then it holds
• lim
x--+xo, xEA
(J(x) ± g(x)) = L ± K;
• lim
x---+xo,xEA
(J(x)· g(x)) = L· K;
lim f(x) L
• x--->xo,XEAg(x) K'
where we additionally assume that there exists a 8 > 0 such that g( x) =1= 0, for
all x in the set (xo - 8, Xo + 8) n A and, moreover, K =1= 0;
The last equalities and inequalities remain true if the point Xo is replaced with one
of the symbols +00 or -00.
x2 - 4
lim = 2.
x-->2 x 2 - 2x
Solutions.
• First, let us use Definition 4.1. For arbitrary c > 0 and x E (1,3), x =1= 2, we
can write
4 _ 21 = IX + 2 _ 21 = Ix - 21.
1xX22 -- 2x x x
144 CHAPTER 4.
Therefore, we have
(Vc > 0) (30 = c) (Vx E (1,3)) 0 < Ix - 21 < o::::::? IxX22 _ 2x4 - 2I < c .
• In order to apply Definition 4.2, let us consider a sequence (Xn)nEN such that
for every n E N, 1 < Xn < 3, Xn #- 2 and
lim Xn
n--+oo
= 2.
Then we have
X2 - 4 . _
11m Xn _+ 2 - n--+oo
lim Xn + 2
lim f(xn) = E.~ X2n n_ 2x n n--+oo X - --1'-'----- = 2.
n~oo
n n~~ Xn
7r
Example 4.8. Show that the function f(x) = sin -, x E R \ {O}, has no limit at
x
the point Xo = O.
.
So IutlOn. Let us cons]'der two sequences gIven. b
y Xn I= -d an Yn = 2 , n =
n 4n + 1
lim Xn = n-+oo
1,2, ... , which have the same limit 0, i.e., n---too lim Yn = O. But it holds
7r
Therefore from Definition 4.2 it follows that there is no lim sin -.
x---+o x
Remark. Neither the limit lim f(x) nor lim f(x) exists.
x--+o+ x--+o-
Exercise 4.9. Prove that lim f( x) does not exist for the following functions.
x--+o
lim (J(x)
X---+XO
+ g(x)) and lim (J(x) . g(x)) ?
x--+xo
lim
X---+O
(~+
X
(-~))
X
= limO = O.
x--+o
LIMITS OF FUNCTIONS 145
. 1
Example 4.11. Show that hm (
" .... 2 X - 2
)2 = +00.
Solution. Let T > 0 be given. Then from the relations
1
(x-2)2>T, (x-2?<!..
T
1 _ 8 1
o < Ix - 21 < v'T - :::}
(x-2)2>T.
2x 3 - 3x 2 - X - 2 + 12x6 + 3x3 .
x8
a) lim , b) lim
" .... 2 x-2 " .... 0 x 7 + 4x 6 + x 5 + x 3 '
3x 2 - 13x - 10
c) lim . d) lim ex +3 _ 3x +1) .
x .... 52x 2 -7x-15' " .... 0 x2- X x3 - X '
rI m
x3 - 8 x 7 + 12x 5 - 13x 4 + 5x 2 + 4x - 9
e) --' f) lim ;
" .... 2 x 2 - 4' x .... l x 5 - 4X4 + 3x 3 - 2x 2 + X + 1
. x 5 - 3x 4 + 5x 3 - 7x 2 + 4
g) hm - - - - - - - - - -
" .... 2 x 5 - 5x 4 + 8x 3 + x 2 - 12x + 4
Solutions.
l' (2
· 2X3_3x2_X-2 1· (x-2)(2x2+x+l)
a ) 11m
" .... 2 x - 2
= 1m
x .... 2 X - 2
= 1m 2x
" .... 2
+ x + 1) = II.
Pn(x) = (x - XO)Qn-I(X),
146 CHAPTER 4.
where the coefficients of the polynomial Qn-l (x) can be ordered by using the
Horner scheme (see Subsection 2.2.1). For the polynomial in the numerator it
holds
1 0 12 -13 0 5 4 -91 x = 1
1 1 13 0 0 5 91 o.
So we have
1 -4 3 -2 1 11 x = 1
1 -3 0 -2 -11 0,
and it follows
So we obtain
. x 7 + 12x 5 - 13x 4 + 5x 2 + 4x - 9
hm - - - - - - - - - - - - -
x--+l x 5 - 4X4 + 3x 3 - 2x 2 + X + 1
1 -3 5 -7 0 41 x = 2
1 -1 3 -1 -21 O.
Therefore, it holds
x5 - 3x 4 + 5x 3 - 7x 2 + 4 = (x - 2)(X4 - x 3 + 3x 2 - X - 2).
Also, we have
1 -5 8 1 -12 41 x =2
1 -3 2 5 -21 o.
So, we can write
Therefore it holds
a) lim x - 3x + 2 3x + 2
2
b) lim x 2 -
x ...... 2 x 2 -4 X""" +00 x2 - 4 ;
c) lim x - 3x + 2
2 X7 + 5x6 - 7x 2 + 5
4X4 -
d) lim '
x ...... -2 (x + 2)2 ; x ...... l x6 - 3x + x + 2x - 1
5 3
Answers.
1 7
a) 4 b) 1. c) + 00, d)
4
'
b) l1m
x ...... l
(m ------
1 - xm
n) '
1 - xn
m,nEN;
x
m+l - xn+l + xn - mx + m - 1
"
m n E N'
,
c) xlim
...... l (x 1)2
' (l+mx)n-(1+nx)m
d) l1m N
' m,n E ,
x ......O x2
Solutions.
b) Assume m > n > 3. If we introduce t = x-I (t ---+ 0 when x ---+ 1), then we
obtain
'
l 1m (m
- - - - - = lim
1 - xm
X""" 1 1 - xn t ...... o 1 - (1
n) (m+ t)m - -----:---:--
1 - (1
n)
+ t)n
. + n m( m2 - 1) ) t + t 2 ( -m (n : + n (m
( -m n( n 2- 1) + ...
hm -'----------;---,--------=------'-----::-~___:_'-----'':__--'--_:_--'--.!..--
3) r: 3) )
t-+O t(t m - l + mtm- 2 + ... + m)(tn- l + nt n- 2 + ... + n)
. t n- l (-m+n(mr: n )) +···+nt m- l
+hm-,-----'--------'---,-:'--'--------
t-+O t(t m - l + mt m- 2 + ... + m)(t n- l + nt n- 2 + ... + n)
x m+ l _ x n+ l + xn - mx + m - 1
lim
x-+l (1)2
X -
Here 0(t 2) stands for a function ¢>(t) with the property lim ¢>(2t )
t-+O t
o (see
Definition 4.54).
a) x--->l
lim (-1
3 + ~1)
-x-
; X
3
xm -1
lim~, m,nEN;
b) x-+l x
Answers.
b) m n(n-1)···(n-m-1)
a) 1. c)
n 1·2···m
Example 4.16. Prove that lim y'X = y'a, a> 0, and lim y'X = 0, n E N.
x-+a x-+O+
LIMITS OF FUNCTIONS 149
Solution. Let us first suppose that a > O. Then for arbitrary e > 0, from
Ivx- ytal = Ix - al Ix - al
< - n-1 < e,
V'x n- 1 + V'xn- 2a + ... + V'a n- 1 V'a -
e) lim
x--+8
v'8+X -
3[;;
4 f) lim y'X -
x--->a+
+~va
'/x2-a~ ;a>O.
yX - 2 '.
Solutions.
· v'x2+x+l-x-l
d) 11m l' (v'x2+x+I-x-l)(v'x2+x+l+x+1)
= 1m -'-------:~=;;======o-----___:-----'--
x--->o X x--->o x( v'x 2 + X + 1 + x + 1)
-x -1
= lim = - - = -1/2.
x--->ox(v'x 2 +x+1+x+1) 1+1
150 CHAPTER 4,
f) lim
x---+a+
VX - va + ~ =
..jx 2 _ a2
lim
x---+a+
(vx - va + ----:vr=;x;=-=a=;;:)
..jx 2 - a2 ..jx 2 - a2
lim ( x-a + 1 )
x---+a+ (VX + va)..jx 2 - a2 vx:ta"
E~+ (VX ~ va' J: ~: + ..jx1+ a) ..;k, =
a)
, ..jx
I1m + 2 - ijx + 20 '
x---+7 ~x+9-2 '
b) lim \1f+X
x - 1 n E Z \ {O};
x---+o
C)
, 11
I1m + ax' xV'1 + bx - 1 , m,n E Z \ {O}, a, bE R;
x---+o
d) lim
'11 + P(x)-1 , where P(x) = alX + a2x2 +, .. + anx n ~ 0, mE Z \ {C
x->o X
Solutions.
, ..;x+2-3 x-7
+ I'1m -----.:...:..
3 - ijx + 20
3
lim 1 1
X _ 7
x->7
x->7 X 7 - 27 112
, ~x+9-2
I1m-----= - 6 1- 27
x---+7 X - 7 32
b) If we change the variables t = 11 + x-I, then from Example 4,16 it follows
that lim 11 + x = 1, and we obtain that t - t 0 when x - t 0, So we have
x---+o
t 1
x->O
n~ 1 = lim
lim y 1 +X x - t---+O (1 +t t)n _ 1 = lim
t---+O
nt + ( n) t + ' , , + tn
2 2 n
LIMITS OF FUNCTIONS 151
c) We assume a, b i- 0, the other cases are left to the reader. Using b), we obtain
. \11
11m + ax· 'V'1 + bx - 1 1.
= 1m
yIf+Tx( ytr:taX - 1) + yIf+Tx - 1
x~o X x~o X
lim 'V'1
x--->o + bx . a . lim ytr:taX - 1 + b . 1·1m ----:----
'V'1 + bx - 1 a + -.
b
x--->O ax x--->O bx = -n m
d) We assume that mEN, the case -m E N is left to the reader.
hm
. r/1 + P(x) - 1
=hm
. r/1 + P(x) - 1 P(x)
._-
x--->O X x--->O P(x) X
\!l+P(x)-l 1
lim
x---.o
P( )
x . lim (al
x~o
+ a2 x + ... + anx n- )
. 1 a1
= al· hm =-.
x--->O r/p + p(x))m-l + r/(l + p(x))m-2 + ... + 1 m
lim ( 1 -
t--->o
v'1=t . 1 - ~ ...
t t
1- vT=t)
t
= ~. ~ ... ~
2 3 n
1
,
n.
.
c) lim
Vl6 + x + x + 4 ; . 1- Vx
d) hm 1 {/X;
x--+-5 yt9 + 2x + 1 x-+1 - X
f) 2yIC a b
nc . g) - --.
m n
152 CHAPTER 4.
.
11m anx n + an_Ix n-I+ . . . + ao an
x-++oo bmx m + bm_Ix m- I + ... + bo n=m;
bm '
0, n < m,
whenever an =I 0 and bm =I O.
Solution. The given expression can be written as
1+ -
an-l
-X
-1 + · · ·+-aoX-n (4.5)
an n-m an an
-·x
bm 1+ bm-I
- - x -1 + · · ·+-bX
o -m
bm bm
Let us suppose that n > m. Since,
·
11m (an-l
--x- 1 + ... + -aoX - n) = 0
x-++oo an an
and
·
11m (b rn -
- - X -1
1
+ ... + -bboX -m) 0
=,
x-++oo brn m
there exist positive numbers Xl and X 2 such that the following inequalities hold.
and
1+ -
an-l
- x -1 + · · ·+-aoX-n
xn- m an an
1+-
bm-l
-X
-1 + · · ·+-boX -m
bm bm
1- l-an-1
-x -1
+ ... + -aox -nl 1 - -1 n-m
1-
> xn-m. an an > xn-m _ _
2 = _x_
I
1 + bm-l x-I
bm
+ ... + ~x-m
b
bm
1 + -1
2
3
LIMITS OF FUNCTIONS 153
Since lim x m -
X---++CXJ
n = +00, we have
.
11m anx n + an_IX n-l + . . . + ao = (s an )
n - . 00
x->+oo bmx m + bm_ 1 x m- 1 + ... + bo g bm .
.
11m anx n + an-IX n-1 + . . . + ao an
--.
x->+oo bmx + bm_ 1 x - + ... + bo
m m 1 bm
.
11m anx n + an-IX n-l + . . . + ao
x->+oo bnx n + bn_1x n- 1 + ... + bo
1 + -an-1 -1 + + aO -n
-X "'-x
an lim xn-m. lim an an = an . 0 . 1 = O.
bm x->+oo x->+oo bm- 1 -1 bo -m bm
l+--x
bm
+"'+-x
bm
a)
.
11m 5X3 + 3x 2 + 2x + 5 5x 3 + 3x 2 + 2x + 5
. b) lim
x->+oo 5x 3 + x 2 + X + 3 ' x->+oo 5x 4 + x 2 + X + 3
) .
11m 5X4 + 3x 2 + 2x + 5
. d) lim v'x+T
Jx + Jx +,;x;
C
x->+oo 5x 3 + x 2 + X + 3 ' x->+oo
e) lim (~-
x--++oo
v'?"TI) ; f) lim (J16x 2 + x-I - 4x) ;
x-++oo
g) lim (\lx3
x---++oo
+ 3x 2 - JX2 - 2x) ; h) lim X (Jx2
x---++oo
+ 2x - 2JX2 + X + x) ;
i) lim (;;!(x
x-++oo
+ al)(x + a2)" . (x + an) - x) .
Solutions.
3 -
x 3 ( 5+ -+ 2 +-
5)
5x 3 + 3x 2 + 2x + 5 l' X x2 x3 =1
lim 5x 3 + x 2 + X + 3 =
a) x->+oo 1m ( 1 1 3)
x->+oo x3 5 + _ + _ + _
.
X x2 x3
3
( 5 + -; 2 5)
b) lim 5x 3 + 3x 2 + 2x +5 x3 + ;2 + ;J
= O.
x-++oo lim
x->+oo 4 ( 5 + _1 + _1 + _3 )
X x2 x3 X4
154 CHAPTER 4.
3
( 5 + ;; 2 5)
.
11m + 3x z + 2x + 5 X4 + ;2 +~
C) +00.
5X4
=
x-++oo 5x + x Z + x + 3
3 lim
X-++OO 3( 5 + _1 + _1 + _3)
x X XZ x3
Vx·R
Vx+ Vi ~ ~ ~
=l.
g) x!i~oo (V/ X 3 + 3x z - v'x z - 2x) = x!i~ (V/ X 3 + 3x z - x)+ x!i~oo (x - v'x z - 2x)
3x Z . 2x
lim + hm = 2.
x-++oo \/(x3 + 3x2)2 + xV/x 3 + 3xz + xZ x-++oo x + v'x z - 2x
2x (v'xz + 2x - x-I)
h) lim x (v'xz
x-++oo
+ 2x - 2v'x z + x + x) = lim x
x-++oo v'X Z + 2x + x + 2.Jx2+x
X Z+ X
_2x2 1
- lim
- x-++oo (v'X Z + 2x + x + 2v'X Z + x) (v'x Z + 2x + x + 1) 4
i) Taking x =~, where t --+ 0+ when x --+ +00, we obtain
t
\l1+P(t)-1
+ al)(x + az)··· (x + an) -
n
y/(x x= t
where
P(t) = (al +az+·· ·+an)t+(alaZ+ala3+·· ·+an_lan )t2+·· ·+alaZ·· ·antn.
From Example 4.18 d) it follows
lim (n/r-(x-+-a-d-(x-+-a-z-).-.-.(-x-+-an-) - x)
x-++oo V
lim
\II + P(t) - 1
=
al + a2 + ... + an .
t-+O+ t n
LIMITS OF FUNCTIONS 155
c
)
lim
x--++oo
( X41 +
+x
2x
3
3
- x) .
'
d) ·
11m
x--++oo
-
(x2- +
2x + 1
x 3 + 4x 2 -
1 - 2x 2
2) .'
3x 6 -1
(Vx4 + 2x 2 - 1- Vx4 - 1) ;
+
e) lim + 2; f) lim 2x 2 -
x--++oo v'x 12 + 2 x4 x--++oo
1
e) 3. f) 2. g) 2· h) J2
8 .
smx
Example 4.23. Knowing that lim - - = 1, determine the following limits.
x--+O x
a)
r1m--;
tan x
b)
r1m-.--,
smax
001 a, bE R;
x--+O X x--+O sm bx
cos 2x 3 - 1
c) lim ( . / . - ~) ; d) lim ;
x--+O sm x SIn x sin x x--+O sin6 2x
e) lim
x2
. f)
r1m tan(a + x) tan(a - x) - tan 2 a
.
x--+o VI + x sin x - vcos x' x--+O x2 '
. cos(a+2x)-2cos(a+x)+cosa
g) 11m 2 .
x--+o X
Slnax
, a'---
' SIn ax l'1m a
b) lIm-,-- = , ax
x-+O sm bx x-+O b, sm bx b
bx
) r ( 2 __ 1_) _r _2_S1_'n_x.,..--.,..-2_S1:-'n-;;x_co_s_x_
c x~ sin 2x sin x sin 2 x - x~ sin 2x sin 2 x
1 - cos x 2 sin 2 ( x /2)
' 2 1 - cos x
l1m, ,
,
= lIm,
x2
,
r 4(x/2)2 1
-
x-+o SIn 2x SIn x x-+o SIn 2x sm x = x~ sin2x sinx 2
2X2 2x x
( sin X3) 2 )
'
d) l1m cos 2x 3 - 1 l' -2 sin 2 x3 l' ~ 1
= 1m = -21m (
x-+o sin6 2x x-+o sin6 2x x-+o 26 (si~:x) 6 32
lim VI + x sin x +
x-+o 1 - cos x sm x
ycosx = _2_ = ~
1 3'
- -2- + - - -+1
x x 2
' cos(a+2x)-2cos(a+x)+cosa
g ) l 1m
x--->o x2
lim
x--->o X
~((cos(a + 2x) - cos(a + x)) - (cos(a + x) - cos a))
lim -1 ( -2 sm
,x ' (a + -
- 'sm 3X) + 2 sm, x , ( a + -X))
- 'sm
x2
x--->o 2 2 2 2
, x
-2sm- 3
ETa x 2 2 (sin(a+ ; ) -sin(a+~))
,x
-2 sm 'x
-, 2 sm -, cos ( a + x )
lim 2 2 = - cos a,
x--->o x2
LIMITS OF FUNCTIONS 157
1 1 ) 2 cos a
d) 2" e) - f --.
a sin 3 a
Example 4.25. Determine
. sin 47rX b) lim smx - sina
a ) 11m. ;
x-+1 SIn 57rX x---+a x- a
· cot x - cot a
11m k d) 1· tan 3 x - 3 tan x.
c)
....L
, aT 7r;
COS X + 6"
x---+a X - a 1m
x-+tr /3
(tr)'
. . 27r
e) hm xsm-;
X---++CXl X
f) lim 3x 2 (cos
x-++oo
~X - cos ~)
X
.
Solutions.
a) After changing of variables t = x-I, t --+ 0 when x --+ 1, we obtain
. x-a x+a
sin x - sin a 2 sm - - - cos - - -
b) lim = lim 2 2 = cosa.
x---+a X - a x---+a X - a
·
11m 2Jr l' sin t
x sin - = 1m 2Jr-- = 2Jr.
"'-->+00 x t-->O+ t
f) lim 3x 2 (cos
"'-->+00
~x - cos~)
x
= lim 3x 2 (2 sin ~ sin ~)
"'-->+00 x x
. 1 . 2
sm- sm-
lim 3·2· 2 _ _x . __ x = 12.
"'-->+00 1 2
x x
.
11m 2 cos 2 X + cos x-I d) lim cos(x + Jr) - cos(3x + Jr)
c) .
X-->7l" /3 2 cos 2 X - 3 cos x + 1 ' X-->7l" (x - Jr) 2
Answers.
1 1
a) 2Jr b) c) - 3; d) 4.
3
Example 4.27. Determine the following limits.
Solutions.
x
. y'lTXsin x + Vcos x 4
hm . =-.
1 - cos x sm x 3
+--
x-->O
----=--
x2 X
LIMITS OF FUNCTIONS 159
c
r y'COS(iX -
) "'~ x 2
y'cOSbX
= "'~
r (y'COS(iX -
x2
1
-
y'cOSbX -
x2
1) = ~(b
2 n
2
_ a2 )
m
•
·
SInce 1cos Vx+T2 +ft -< 1 for all x E R ,we h ave
1
1",.!!f)sin
. .
Vx+T - sin ft)l::; "' .....lim
+00
12 sin 2( VxTI
1
+ ft) 1. 1 0 T
. hus
hm (sm Vx+T - sin y
X~+OO IX)w= 0.
.
11m {lsin x - ,ysin x
c) . d) lim (cos VxTI - cos ft) .
cos 2 X
"' ..... 7r/2 ' x-++oo
Answers.
1 1
a) 4V3. b)
12 c) 24 d) O.
1 t
7r - 4 arctan 1 + x = lim ~ t 1
1m x
"'1·..... 0 t .....o cot -4--
. 7r-t . 7r t. 1 1
tsm--sm-
J2. v'2
lim
t ..... o . (7r
sm
4
- -7r-- - t)4 = limo t ..... . t
sm-
= 2.
4 4 4
160 CHAPTER 4.
a) lim
arcsIn x
, b)
r
Im--
arctan 2x
x-+o 3x x-+o x
Answers.
1
a) - b) 2.
3
Exercise 4.31. Let us suppose that the following inequalities
are satisfied on the interval (a,{3) \ {xo}, Xo E (a,{3). Additionally, if there exists a
number L such that
lim gl(X) = X-+Xo
X-+XO
lim g2(X) = L,
a) lim
x-++oo
(1 + ~)X
X
= e; b) lim(l+x)l/x=e.
x-+o
Solutions.
a) Let x take the values of the sequence (xkhEN with the property lim Xk = +00.
k-++oo
Then for every monotonically increasing sequence (nkhEN such that nk E N
and nk -+ 00 when k -+ +00, it holds nk :::; Xk :::; nk + 1 and
111 111
--<-<- hence 1 + - - < 1 + - < 1 +-.
nk + 1 - Xk - nk' nk + 1 - Xk - nk
Wherefrom we have
1 )nk
(1+- - < ( 1+-l)Xk
< ( 1+-l)nk+l
nk + 1 - Xk - nk '
or
b) If x --t 0+, then for t = ~ we have t --t +00 and from a) we obtain
x
lim (1
x-+o-
+ x)1fx = (1 _~)-.
s
lim (_S_)'
lim
.-++00 s - 1
.-++00
=
lim (1
x----+o-
+ X)l/x = lim (1
X~O+
+ X)l/x = e,
it follows that
lim(l
x-+o
+ X)l/X = e.
a) lim aX = a Xo , a > 0 , 0
x E R, b) lim Inx = Inxo, Xo > 0;
X~Xo x ....... xQ
c) lim (u(x))v(x)
x ......... xQ
= ab , Xo E R.
In c)! we suppose that u( x) > 0 and that there exist a > 0 and b such that
Solution.
and therefore, for given £ > 0, there exists no E N such that for a > 1 it holds
1 - ~ < a-l/no < aI/n0 < 1 + ~.
aXo aXo
Note that we used the fact that the function aX is monotonically increasing on
1 1 1
the whole R. If we suppose that Ix - xol < -, then from - - < x - Xo < -,
no no no
it follows
1 - ~ < a-l/no < a X- XO < aI/no < 1 + ~,
aXO aXO
wherefrom we have
£ £
< a X- XO _ 1 <
aXO aXO
162 CHAPTER 4.
1
So for 0:= - and Ix - xol < 0, we have
no
laX - axol = axolax-xo -11 < e.
Thus for a > 1, it follows lim aX = aXO for any Xo E R. In the case when
i,
x-+xo
· a = 1·1m
11m X
-1= -1 = a X
o.
x-xo x-xo bx lfo
b) We proved the following inequalities (see Example 3.38 c))
- e < __
~-1
1_ < In (1 - ~) < In (1 +~) < ~ < e.
~ ~ ~
(4.6)
Xo
If we suppose that Ix - Xo I < - , then we have
no
1 x - Xo 1
--<--<-. (4.7)
no Xo no
From relations (4.6) and (4.7) we obtain
under the condition Ix - xol < 0 with 0:= xo. So we get for Xo > 0
no
lim Inx = Inxo.
X---IoXo
}~o (u(x))v(x) = }i.~o exp (v(x) In u(x)) = exp Ui.~o v(x) . }i.~o In u(x))
e blna = abo
LIMITS OF FUNCTIONS 163
then we have
lim u(xt(x) = exp (lim (u(x) - 1) v(x)) , (4.8)
X-+Xo X-+Xo
Solution. We have
lim u( x )v(x) = lim (1 + (u(x) _ 1))l/(U(X)-l)) (u(x)-l)v(x)
X--+Xo X-Io
c
Example 4.35. Determine the following limits.
a) lim - -
x-->+oo c+
1
x-I r .
'
b) lim - X +-3 r+ 2 .
x-->+oo 2x + 2 '
c) lim( JI+X
x-->a
- x)2fx; d) lim( cos x
x-->a
t cot 2 x;
e)
. C1++
hm
x-->a
tanx) 1/ sin 3 x
.
sm x
f)
. (1 + tan X) 1/ sin x
hm
x-->a1+ sin x
Solutions. We shall repeatedly use Example 4.34.
x+I
a) In this case we put u(x) = - - and v(x) = x, hence
x-I
(u(x) - 1) v(x) = ( ; ~~ - 1) x = x 2~ 1.
Thus we obtain
.
11m -+-
(X 1) x = exp ( 1.1m -2X)
- = e 2.
x-->+oo x-I x-->+oo x-I
2X+3)X+2
lim ( - -
x-->+oo 2x + 2
= exp ( x-->+oo
lim - - = e1 / 2 = .;e.
X+2)
2x + 2
Also, this example can be done as follows.
2X + 3)X+2
lim ( - - = x-->+oo
lim
(( 1 )2X+2) 1/2 . lim ( 1 + -1)
1 +- - - =.;e.
x-->+oo 2x + 2 2x + 2 x-->+oo 2x + 2
164 CHAPTER 4.
2
c) If u(x) = vr+x - x and v(x) = -, then
x
Thus we have
(u(x) - 1) v(x) = (v'1+x - (x + 1)) ~.
x
From the formula (4.8) we get
lim(v'l+x - X)2/X --
x---+-o
~•
e
1 + tan x 1
e) If we denote by u(x) = . and v(x) = -'-3-' then we have
1 + smx sm x
(ux ( ) -_ tanx-sinx . _
() -1 ) vx 1 _ tanx(l-cosx)
-- ._-
1
1 + sinx sin3 x 1 + sin x sin3 x
2 . 2 X
sm "2 1
cosx(l+sinx)' sin 2 x'
Therefore we have
1 + tan x 1/ sm
. 3x ( 2 sm
. 2 -X )
lim = exp lim . 2 = ve·
x-a (1 + sinx) x-o cosx(l + smx) sin x
2
f) !~ (1 + t~n X) 1/ sin x
° 1 + smx = 1.
Exercise 4.36. Determine
a lim
X
--
)X ; b) lim (X2 + 2)X2_1
x +1
(
) X_+OO X_+OO x2 - 2 ;
Answers.
1
a) - b) e 4 • c) e. d) e.
e
Exercise 4.37. Prove that
a) lim log a x = log a Xo, a > 0, Xo > 0; lim arccos x = arccosxo, Ixol < 1;
b) X---+Xo
X---+Xo
1 7r
c) lim arctan - = -2;
x ..... o+ x
d) lim arctan x =
x ..... -oo
_!!:..2
Example 4.38. Determine the following limits.
. In(1
c ) 11m
+ 4x + x 2) + In(1 - 4x + x 2) . d) lim V4+X - 2 - sinx.
X"'" 0 2x 2 ' o
x .....In(1 + x) ,
. In cos 4x
e) 11m .
x ..... o In cos 3x '
f) lim x 2 ln (sin
x ..... oo
(!!:.2 - !!:.))
x
;
In(1 + 3 X ) • . In(1 +3 X )
b)
x ..... +oo
x +-
· x 1og3 -
11m 5 = -11- . 1·1m 1n ( 1 + - -
+ 3 n 3 x ..... +oo
x + 3 = -1n-3. X
2) x 2
c) lim In(1 + 4x + x 2)2x
x..... o
+2In(1 - 4x + x 2) = lim In(1 - 14x2 + X4)1/(2x2)
x ..... o
=
. .;4+X-2 -hm--
( x---+o
hm . Sinx) ·hm
. ( In(1+x )I/X)-l_ ~
--4.
x x---+o X x-+o
2
1r2
f) lim x 2 ln (sin (~2 - ~))
x
= ~2 X--++OO
lim In (1 _ sin2 ~)X =
X--++OO X 2
eX -1
a) lim--; b) lim(l+x)b- 1 , bER;
x--+o x x--+o x
. 1 - cosb X d) lim e
X2
- (cos x)2v'3
c) hm 2 ,b E R;
x--+o x x-+o X -
Solutions.
b) For b = 0 it is trivial that the given limit is equal to 0 = b. For b =1= 0, we start
with the following transformation
e bln(l+x) _ 1 bln(1 + x)
Since lim b ( ) = 1 and lim = b, we obtain finally
x--+o In 1 + x x--+o x
c) For b = °the limit is zero. For b -I- 0, similarly as in b), we can write
l-cosbx l_ebln (cosx) bln(cosx) 1 b
lim = lim . lim = 1· b· - = -.
x--+O x2 x--+o b In( cos x) x--+o x2 2 2
aX _ x a XX _ aa aa X
_ a xa
a) Iim--- b) Iim--- c) l i m - - -
x--+a x- a x_a X - a x--+a aX - x
a
Solutions.
X - a)a
aX _ x a x-a 1 ( 1 + - - -1
Iim--- aa lim a - _ lim a . aa-1
x-+a x - a x_a X - a x_a X- a
a
aa In a - a· aa-1 = aa In(aje).
XX _ aa XX _ x a x a _aa
x-a
-x-a
- + x-a
--
X_ a
l'1m
ealnx (e(x-a)lnx - 1)
· x
11m X
x_a X - a = x-a ( X - a ) 1n X . 1n x a 1
= ana.
Iim xa - aa _. aa-1((1+ x - a )a - )
x_a X _
a
- hm
x-+a x-aa 1 = aa ,
a
aa X _ a xa . aaX-x" - 1 . x. . at - 1 . a"
c) Iim--- a
x--+a aX - x
= x_a
hm . lIma
aX - x a x_a
= hm---a
t_D t
a =a Ina,
a) lim (2
x-+o+
+ v'x); b) lim (2
x---+o-
+ v=I);
1
l'1m -; . 1
c) d) hm -;
x-+o- X x-+o+ X
g) lim arcsin(x
x---.+o-
+ 1); h) lim arcsin( x
x-+o+
+ 1);
1 1
k) lim I) lim
+ e2- x + e 2 -1 x
1 ,
x-+2- x2 x-+2+ x2
Answers.
a) 2. b) 2. c) -00. d) +00.
e) O. f) 1. g) 7r /2. h) Senseless.
i) O. j) +00. k) O. I) 1/4.
Solutions. For the convenience of the reader, we rewrite the definition of the
hyperbolic functions.
sinh x = eX - e- X
eX + e- X sinh x eX - e- X
cosh x = 2 and tanh x = --- = - - -
2 cosh x eX + e- X
. h -x 2x 1
· -
a ) 11m sm- -
x l'1m e
= x-+o
X
- e l'1m e -x . -
= x-+o e -- - = 1.
x-+o x 2x 2x
. h2 -X 1
2sm ( sm-
. h X) 2
1
b) lim cosh x - 1 = x-+o
lim 2 = - lim ~ -
x-+o x2 2 x-+o _ 2
2
1 .
11m 1
.
b) EW-1 +e1/x' x-+o+ 1 + e1 / x '
c) lim
x~-oo
(1 + -111)X x lim
x-++oo (1 + -111)X;
x
d) x.!!~(Jx2+5x-Jx2+2x+1), lim
X~+OO
(J x2+ 5x - J x2+ 2x + 1) .
Answers.
7r 7r 3 3
a) 2' 2 b) 1, O. C) e -1 , e. d) - 2' 2
Example 4.44. Show for a> 1, k > 0 :
xk
a) lim - = 0; b) lim loga x = O.
x-++oo aX x-++oo xk
Solutions.
a) Using Example 3.12 b), we have
lim -
nk
= 0 hence lim (n + l)k
n~oo an n-+oo an = O.
This means that there exists no E N such that, for given c > 0, it holds
(n + l)k
-'-----'-- < c, for every n > no.
an
If we denote n = [x], then we have n::; x < n +1 and
xk (n + l)k
o < -aX < an
< c, for n > no.
lim
x--+o+
(x"" sin.!..) = O.
X
b) In bX) , b > 1.
lim In( x In b) In ( -x-
X--+O+ In-
b
Solutions.
a) lim ((x
x-++oo
+ 3) In(x + 3) - 2(x + 2) In(x + 2) + (x + 1) In(x + 1))
. (x + 3)x+3( X + 1)x+l .
X+3)X+2 (x + 3) )
(( :.;;---z
=
"-+~
hm In
( (x + 2)'«") )
=
"-+~hm In +
(X-+-2)'+' (x+2)
-
~ 0.
x+1
(-x-
~b ~b-~-
X)b
In _ ) (
b) lim In(xlnb)ln = lim In(xlnb) ·In 1 + x
X--+O+ X--+O+ In _
b b
In(x/b)/lnb2
. In ( x In b) In b2 1 In b2) 1 b2
1 + ---x
11m (
x . n = n .
x--->O+ In - In-
b b
provided that both limits in (4.9) when x -+ +00 (resp. when x -+ -(0) exist.
d) The function f is defined on the interval [1, +00). The graph of f has no vertical
asymptotes, because
lim
x-+1+
(Vx+1 - Vx=1) = Vi.
It has a horizontal asymptote y = 0 when x -t +00, because
lim
x-++oo
(Vx+1 - Vx=1) = O.
e) The function f is defined on the interval [0,4J. Its graph has no asymptotes.
f) The function f is defined on the interval (-00, OJ. It has no vertical asymptotes,
because
.
11m . x 7r
arcsIne =-.
x-+o- 2
However, from
lim arcsin eX = 0,
x---+--oo
Solutions.
a) The natural domain of the function f is the set R \ {I, -I}. Its graph has
vertical asymptotes x = 1 and x = -1, because
.
11m 1
- - - =-00 and lim _1_2 = +00.
x-+1+ 1 - x2 x-+-1+ 1- x
(Note that lim _1_2 = +00, lim _1_2 = -00.) Its graph has a
x-+1- 1 - X x-+-1-1- x
horizontal asymptote y = 0 when x -t +00 and when x - t -00, because
c) The graph of f has two horizontal asymptotes, namely y = -7r/2 when x ----t -CXJ
and y = 7r /2 when x ----t +CXJ.
d) The graph of f has two slanted asymptotes, namely y = -x when x ----t -CXJ and
y = x when x ----t +CXJ.
Note that
lim In(l + eX) = +CXJ,
x---++oo
hence it has no horizontal asymptote when x ----t +CXJ. However, it has a slanted
asymptote y = x when x ----t +CXJ, because
a) f(x) = x 2 e x ; b) f(x) = e- x2 ;
c) f(x) = e1 / x ; d) f(x)=e- 1 / x2 ;
d) horizontal asymptote y = 1 both when x ----t -CXJ and x ----t +CXJ, because
lim e- 1/ x2
x---+-oo
=1 and lim e- 1 / x2
x---++oo
-
-,
1
f) vertical asymptote x = 0 and a slanted asymptote y = x-I when x ----t -CXJ and
also when x --t +CXJ.
174 CHAPTER 4.
In particular, if g(x) == 1, then f(x) = 0(1) as x - Xo. This means that f tends
to zero as x - Xo.
Definition 4.55. We say that a function f is big oh of the function 9 as x
approaches Xo if there exists a constant K > 0 such that
In3
c) ( X+l)X rve2, x-+oo; d) In(1 + 3X ) rv -1-In(1 + 2X ), x - +00.
x-I n2
LIMITS OF FUNCTIONS 175
Solutions.
eX -1
a) From the equality lim -.-- = 1/2, by Definition 4.53 it follows
x-+o SIn 2x
Xl.
e -1", "2sm2x, x --t 0.
sinx, x E Q; X, x E Q;
f(x) = { and g(x) = {
0, x E R \ Q, 0, x E R \ Q,
and by Definition 4.53 this means that f(x) '" g(x), when x --t 0.
Remark. Note that in this case the limit lim f((X)) does not exist.
x-+o g x
. sin 3x + 2 arctan 2x + 3x 2
11m In cos x
a) . b) lim 2;
x-+o In(l + 2x + sin 2 x) + xeX ' x-+o tan x
c) lim x (In (1
X-++OO
+ :.)
2
- In :.) ;
2
d) ~~(1 - x 3 )cot X.
Solutions.
b) Since
2ln cos x = In(l - sin 2 x) '" - sin 2 x '" _x 2 , tan x 2 '" x2 when x -+ 0,
we have
lim In cos x = -1 lim x 2 = _ ~.
x-+o tan x 2 2 x-+o x 2 2
c) lim x (In (1
x-++oo
+ :.)
2 - In:') = lim x In (1
2 x-++oo
+ 3.)
X
= lim x
X~+OO
(3.x + (3.))
x
=
0 lim (2
x~+oo
+0 (1~)) = 2.
X
In(l - x 3 ) _x 3
lim(l - x 3 )cot x = l, where £ = lim = lim - - = 0,
x-+o x-+o tan x x-+O x
it follows
lim(l - x 3 )cot x -- I .
x--+o
7r 1 1 7r
e) arctan x '" 2' x -+ +00; f)
-cos m X '" (7r
2 -x
)m' X -+ 2.
Exercise 4.60. Determine the numerical constants a and b such that f(x) '" ax b,
when x -+ Xo, if
Answers.
1 1
a) a=l , b=-
8 '3 b = -2'
b) a = Vv,
21
c) a -- -4' b -- 4 . d) a = 12, b = 2.
Solutions.
c) hm
. (1 + sin xcos ax) cot
3
X
Solutions.
x
a) From the following expression e x /(x+1) -1 = --+o(x), x -+ 0, and Example
x+1
4.34 we can write
lim ( 2e x /(x+1)
x--+o
- 1) (x2+1)/x = exp (
lim(
2 e x /(x+1)
x--+o
- 1) .x-+-
2
X
1)
exp ( hm2
x_a
--+o(x) ) .x-+-
. (X
x +1 x
2 1)
exp
' 2(X2-+-1+ (x 2+ l)O(X))) = e2.
(1x_a
1m
x +1 x
178 CHAPTER 4.
a2x 2
a2x2
In ( 1- - + o(x 2)) _ _ + o(x 2)
. In cos ax
hm
x-+o In cos bx
l'
= 1m
x-+o In
(
1_ b2
2
2
+ o(x 2)
)
+ = lim -~}"-;2;----
x-+o b x + o(x 2)
--2-
a2
b2 •
(
1 + sin x cos ax ) 3 cos ax - cos bx cos 3 x
-----~-1 ·cot x= .--
1 + sin x cos bx 1 + sin x cos bx sin 2 x
b22
(- -a 2
- x 2 +o(x) 2) (1-2"+o(x
X2 2) )3
1+ (x + o(x)) . (1 _b2;2 + O(X2))' (X2 + o(x 2))
b2 - a2 x 2 + o(x 2)
2
x 2 + o(x 2)
Then we have
(.
exp hmcot 3 x·
x-+o
(1 + sin. x cos axb -1 ) ) =exp((b
1 + sm x cos x
2 -a 2 )/2).
Solutions.
a) Since it holds 11995x + xcosxl ::; Clxl, x E R, (for example for x 2: lone can
take C = 1996), it follows that the given statement is true.
b) The statement is true, because for x 2: 1 it holds
1
Iy' X2 + 3 - x I < 3 . -,
x
x:2: 1.
x2
a) l-cosx=2+0(x),x~0; b) aX -1 = xlna + o(xlna), x ~ 0;
Solutions. We shall prove only parts a) and d), the others are left to the reader.
a) Put 9 = O(J) and h = O(g). Then by Definition 4.55 there exist constants
KI > 0 and K2 > 0 such that in some neighbourhood U of Xo it holds for
x # Xo
Ig(x)1 ::; Kd(x) and Ih(x)1 ::; K 2g(x)
Then on the set U \ {xo} it holds
d) Put 9 = 0(J) and h = O(J). Then by Definitions 4.54 and 4.55 there exist a
function 1>, a constant K > 0 and a neighbourhood U of Xo such that
Then the sum of the functions 9 and h on the set U \ {xo} can be written as
for KI := 1 + K.
180 CHAPTER 4.
Exercise 4.66. Show that if the functions f and 9 have the properties f( x) =J. 0
and g( x) =J. 0 for x =I- Xo, then as x --t Xo it holds
Example 4.67. Let us suppose that f(x) rv h(x) and g(x) rv gl(X), when x --t Xo.
If there exists lim h ((x ))' then there exists lim f(( x)) satisfying
X-Xo gl X x-xo9 X
Prove.
1 + O(Jl(X))
lim f(x) = lim f1(X) + 0(J1(X)) = lim f1(X) . lim f1(X) = lim f1(X)
x-xo g( x) x-xo gl (x) + O(gl (x)) x-xo gl (x) x-xo 1 + ---,O(g=--l,:. .x:..c..))
.,( x-xogl (x)"
gl(X)
Chapter 5
Continuity
If the point Xo E A is an accumulation point of the set A, then the following two,
mutually equivalent, definitions can also be used.
Definition 5.4 .
• Assume that the domain of a function f contains an interval (a, xo]. Then the
function f : A ----+ R is continuous from the left side at the point Xo if
181
182 CHAPTER 5.
• Assume that the domain of a function f contains an interval [xo, b). Then the
function f : A --+ R is continuous from the right side at the point Xo if
In view of this theorem, one can classify the discontinuities of a function f defined
on an open interval. Namely, if a function f : (a, b) c R --+ R is discontinuous at
a point Xo E ( a, b), then there are three possibilities.
• If the limit lim f(x) exists and equals to some number L
X--+Xo
:f- f(xo), then f has
a removable discontinuity at the point Xo;
exist, but are unequal, Ll :f- L 2, then f has a first order discontinuity at
the point Xo;
• if at least one of the two one sided limits in (5.2) does not exist, then f has a
second order discontinuity at the point Xo.
A function f : A C R --+ R is continuous on a set B C A if it is continuous at
every point from B. It is important to note that all basic elementary functions are
continuous on their natural domains.
If the functions f : Al C R --+ Rand 9 : A2 C R --+ R are continuous at a point
Xo E Al n A2 (resp. on a set B C Al n A 2), then the following functions are also
continuous at the point Xo E Al n A2 (resp. on the set B C Al n A 2) :
• f + 9 (sum of f and g),
• f· 9 (product of f and g),
Of course, for the quotient £ this holds only when g(xo) :f- 0 (resp. when g(x) :f- 0
9
for every x E B).
Assume that a function 9 : A C R --+ R is continuous at a point Xo E A and that
a function f: B C g(A) --+ R is continuous at the point g(xo). Then the composite
function h = fog is also continuous at the point Xo. Shortly we say that the
• composition of continuous functions is continuous.
CONTINUITY 183
a) f(x) = 2x + 1, x E Rj b) f(x) = x 2, X E Rj
e) f(x) = arctan x, x E R.
Solutions.
a) Fix Xo E R. Let c > 0 be given. From the inequality
and assume that the last expression is less than c. Then choosing
Remark. The number 8 from Definition 5.1 (hence also in the last example) is
not uniquely determined, Namely, if for given c, we find a 8 such that for every
x E (xo - 8, Xo + 8), it holds If(x) - f(xo)1 < c, then for every other 81, 0 < 81 < 8,
it holds
Ix - xol < 81 =? If(x) - f(xo)1 < c.
Again, it is important that for given c > 0 a 8 > 0 exists.
c) Fix first Xo > O. Let c > 0 be given. From the relations
Ix - xol Ix - xol
1v'X - VxOI = ..;x + JXo <JXo,
184 CHAPTER 5.
Hence we can determine b:= c..jXO, which then gives the implication (5.1).
The case Xo = 0 is left to the reader.
Ix-xol b
2 < -3 .
(
~ + 1Xo
2
)
+ ~4 Vay6 X
-(i6
4
x - Xo bi bi
< < < c,
1 + xXo 2 i:
1 + Xo + UIXO - 1 + Xo - blXo
2
. 1 + x5
prOVIded that bi < C.
1 + cXo
Similarly, if x < Xo, i.e., b2 := -(x - xo) > 0, then
b2
Iarctan x - arctan xol ~ 8 < c, (5.3)
1 + Xo - 2XO
2
1 +x 2
provided that again 82 < 0 c.
1 +cxo
Hence, putting
8 := ~ . min { 1 + x5 c, Ixol }
2 1 + cXo 2
(the latter in order to keep the intervals (xo - 8j, Xo + 8j ), j = 1,2, away from
the point 0) we get the implication in (5.1).
CONTINUITY 185
Exercise 5.7. Check the continuity of the following functions at an arbitrary point
Xo from their domains.
Exercise 5.B. Check the continuity of the following functions at an arbitrary point
Xo from their domains.
Exercise 5.9. Check the continuity of the following functions at an arbitrary point
Xo from their domains.
2 - 2 cosx
e) f(x) = x cot x, x E R \ {hi k E Z}; f) f(x) = ? ,x E R \ {O}.
x
Example 5.10. Using Definition 5.1, check the continuity of the following functions
at an arbitrary point Xo from their domains.
x # 0; 0, x E 1:= R \ Q;
a) f(x) = { 1:1' b) D(x) = { 1,
x E Q.
-1, x = 0;
Here Q and I stand for the (mutually disjoint) sets of rational and irrational num-
bers, which together make the set of real numbers R.
Remark. The function in b), called Dirichlet's function, was already defined in
Example 2.20.
Solutions.
186 CHAPTER 5.
a) From the definition of the absolute value, it follows that f(x) = 1 if x > 0, and
f( x) = -1 if x :S O. Let us show first that f is continuous at every point
Xo i= O. Firstly, if Ix - xol < ~Ixol, then x and Xo are of the same sign. Hence
the function f is only continuous from the left at 0, but it is not continuous
at the point O. In fact, f has a first order discontinuity at o.
b) We claim that Dirichlet's function is discontinuous at every real point Xo, rational
or irrational. We shall prove this for Xo E Q. (The other case, namely Xo E I,
is handled in a completely analogous way.)
In order to prove the discontinuity of the function D at xo, we must show
that there exists an c > 0 such that for every b > 0 there exists an x E R (x
depending on b) with the property
Let us put c := 1/2. Then for every b > 0 there exists an irrational number X5
such that IX5 - xol < b. However, then it holds
-I, x E Q;
f(x) = { 1, x E R \ Q.
Example 5.13. Assume that the function f: (a, b) -+ R has the following property
at a point Xo E (a, b).
c) ("18> 0) (3c > 0) ("Ix E (a, b» Ix - xol < 8 => If(x) - f(xo)1 < c.
What can one say about the continuity of f at xo?
Solutions. We shall find three functions that will satisfy the stated implications in
a), b) and c) respectively, but neither of them will turn out to be continuous at the
point Xo. This means that the order of the quantifiers as well as the direction of the
implication in Definition 5.1 are essential.
a) Let us define the function f by (see Figure 5.1).
X, x::; 1;
f(x)= {
x+1, x>1.
lY
J
lY
2 / x
0
'(
Figure 5.1. Figure 5.2.
Clearly, this function is continuous on the set R \ {I}, while it has a first order
discontinuity at the point Xo = 1. Let us show, however, that f does have the
stated property at the point Xo = 1. To that end, for c > 1 we put 8 := c - 1.
Then for every x E R it holds
For c ::; 1, we put simply 8 := c. Then the set of points x > 1 that satisfy the
inequality If(x) - f(l)1 < c is empty, hence the implication (5.4) is true. For
x < 1 it holds
If(x) - f(1)1 < c -¢:=} Ix - 11 < c.
188 CHAPTER 5.
(If(x)- f(O)1 = Ix 2 -11 < x 2 +1 < c) =?- (Ix-Ol = Vlxl2 + 1 - 1 < ~ < b).
f(xo) .
b) (Vx E (a, b)) Ix - xol < b =?- f(x) < -2-' promded that f(xo) < O.
Solution. We shall prove only the case a), the other one being analogous. By the
definition of the continuity of f at Xo, for c := f(;o) , there exists a b > 0 such that
the following implication holds.
f(xo)
(Vx E (a, b)) Ix - xol < b =?- If(x) - f(xo)1 < -2-.
Hence for all x E (a, b), such that Xo - b < x < Xo + b, it holds
(_f(;o) < f(x) _ f(xo) < f(;o)) =?- (f(;o) < f(x) < 3f~xo)).
CONTINUITY 189
1, x = 0; 1, x = 0;
c) f(x) =
{ ,in (~) , x
xi- 0;
d) f(x) =
{ "in (~), x
xi- 0;
C, x = 0; 0, x = o.
In c), C stands for some real number. We suggest to the reader to prove the conti-
nuity of these four functions on the set R \ {O}.
Solutions.
a) Let us show that f is continuous at the point O. Using the limit lim smx = 1,
! x-a X
see Example 4.23, we have
11m x ! = 1·1m I-
· f() x I = 1·1m -
sin- sin-
x = 1 and
x_o+ ! x_o+ X x_o+ X
I I
I . •
· f()
I1m x = 1·1m - x = 1·1m -
sm- - sm
-- x = l.
x_o- x_o- x x-o--x
Since also f(O) = 1, in view of Theorem 5.5, f is continuous at o.
b) Since . .
. smx. smx
hm
x_o+
f( x) = lim -,-, = hm -
x_o+ x x_o+ X
= 1 and
· f()
11m x = 1·1m - x = 1·1m -
sin- sin-
x =-1
x--+o- x--+o- Ixl x--+o- -x '
the function f has a first order discontinuity at the point O.
c) Let us prove first that f has no limit at O. If Xn = _1_, n E N, then lim Xn = 0,
n7r n--+oo
2
and if x~ = n E N, then also lim x~ = O. However, it holds
4n + 1 1f
f )'
n-oo
Example 5.16. Find the points of discontinuities and their kind for the following
functions.
I, x> 0;
a) sgn x:= { 0, x = 0; b) f(x) = sgn 2 x, x E R.
-1, x < 0;
Solutions.
a) Since
lim sgnx = 1 and lim sgnx = -1,
X-+O+ x--+o-
'y
2 -
lr
'Y
I
:It
o
-
2 3
:It
0 1
-1
I
Figure 5.3. Figure 5.4.
Example 5.17. Find the points of discontinuities and their order for the following
functions.
lim (ax
x-+k-
+ b[x]) = ak + b(k - 1) = (a + b)k - b.
lim x[x]
x-+o-
= x-+o-
lim X· lim [x] = O· (-1) = O.
x-+o-
lim x[x]
x--+k-
= x-+k-
lim X· lim [x] = k· (k -1) = P
x-+k-
- k.
0, x = 0; 0, x = 0;
0, x = O.
192 CHAPTER 5.
Solutions.
lim
x--+O+
[~]
X
= t--++oo
lim (t - r(t)) = +00.
Analogously it holds
lim
x--+O-
[~]
X
= -00.
at the points i, k E Z.
k-l
·
I1m
x--+1/k+
X· [ -1 ]
X
= l'1m ( -1
~--+o+ k
+€) . [ 1 ]
1/ k + €
= _.
1 I'1m [-k- ]
k ~--+o+ 1 + k€ -k-'
c) Let us check first the continuity of f at O. We start from the following two
sequences that converge to zero.
It holds
lim f(x n ) = +00
n~oo
and lim f{x~) = -00.
n~oo
Thus f has no limit or one sided limits at 0, hence f has a second order
discontinuity at O.
lim
x~l/v'k+
[~].
x
sgn (sin( ~))
X
= lim
e~O+
(~+£r
1
.~+n (~I+£r)
lim [ (1
e--->O+
k ] .sgn (sm. (1 + £VkF
+ £VkF k) = (k - 1) . sgn( sm k) .
and
lim
x~l/v'k-
[~].
x
sgn (sin( ~))
X
= lim
e~O+
(~-£r
1
. sgn (,in I I
(Vk-£
r)
lim [ kVk ]. sgn (sin k ) = k· sgn(sin k).
e~O+ (1-£ kF (1-£ Vkk)2
1
Since the left- and the right-hand side limits of f at Vk are different, the
1
function f has first order discontinuities at the points Vk' kEN.
Example 5.19. Determine the constant C, if possible, in order to obtain (at least
one sided) continuity of the functions given below at the given point xo.
{ Ixl-x x # 0;
a) f(x) = x2 ' xo = 0;
C, x = 0,
{ Ix + II x # -1;
b) f(x) = ~' Xo = -1;
C, x = -1,
c) f(X)~{ CXP(x+D· x # 0;
xo = 0;
C, x = 0,
x
# 0;
= 0,
Xo = 0;
e) f(x) = { xlnx
2 x # 0;
,
Xo = O.
C, x = 0;
194 CHAPTER 5.
Solutions.
0, x> 0;
2
f(x) = { , x < 0;
x
C, x = o.
Hence
lim f(x) =
x--+o-
+00 and lim f(x) = 0,
x--+o+
b) Since f is equal to
-I x< -1;
f(x) = { 1,' x> -1;
C, x = -1,
it has a first order discontinuity at x = -1. For C = -1, f is continuous from
the left side, while for C = 1 it is continuous from the right side.
Exercise 5.20. Determine the constant C, if possible, in order to make the follow-
ing functions continuous at the given point Xo.
{ x-2 x i= 2;
a) f(x)= x 2 -4' Xo = 2;
C, x = 2,
f(x) ={ Cx 3 + 1, x::::: 0;
b) Xo = 0;
x, x < 0,
{ ,;n2x, x <~.
- 2' 7r
c) f(x) = Xo = 2'
+ 1),
7r
C(x x>-
2'
d) f(x) ~{ 1
In(x _1)'
x> 1;
Xo = 1.
C+1, x :S 1,
Answers.
Exercise 5.21. Determine the constants a and b, if possible, in order to make the
r-'
following functions f continuous on R.
x<
- 2·,
x i= 2, x i= -2;
{ 24'
1
b) f(x) = x:,
x = 2;
b, x = -2.
Answers.
a) a = 1, b = -2.
196 CHAPTER 5.
b) The function has second order discontinuities at the points Xl = 2 and X2 = -2,
and therefore we can not order the constants a and b to make f continuous on
R.
Example 5.22. Determine the constants a, band c, if possible, in order to obtain
continuity of the functions given below on their domains.
{ 2x, Ixl ~ 1;
a) f(x)= x 2 +ax+b, Ixl > 1;
1 1
x x +1 x r/: {-1,0,1};
1 1'
c) f(x) = x -1 x
a, x = -1;
b, x = 0;
I
c, x = 1;
(~)
~
x cos
-7r 37r]
sm x X E [2'2 \ {0,7r};
d) f(x)
a, x = 0;
b, x = 7r.
Solutions.
a) Since it holds
lim f(x) = lim 2x = 2, lim f(x) = lim(x 2 +ax+b)=1+a+b,
x-+l- x-+l- X"'" 1+ X"'" I+
lim f(x) = lim (x 2 +ax+b) = 1-a+b, lim f(x) = lim 2x = -2,
x-+-l- x-+-l- x ..... -1+ x ..... -1+
the function f is continuous on R only if
= 1 + a + band - 2 = 1 - a + b.
2
Hence we must choose a = 2, b = -1.
c) It holds
lim f(x) = +00, limf(x) = -1, limf(x) = O.
x~-l- X--+O x--+l
lim f(x)
x--+o
=1 and lim f(x)
X--+1r
= ~,
2
Example 5.23. Examine the continuity of the compositions of the functions fog
and 9 0 f on R if
a) f(x) = sgnx, g(x)=1+x 2 ;
2, x < 0;
g(J(x)) = 1 + (sgnx)2 = { 1, x = 0;
2, x> 0,
has a removable discontinuity at Xo = O.
x :S 0;
g(J(x)) = sgn(x(l + sgnx)) = { ~: x> o.
Both functions have first order discontinuities at the point x = 0, while the
function fog has the same kind discontinuity at the point x = -1.
Exercise 5.24 Prove that a monotone function can have only first order disconti-
nuities.
Example 5.25. If the function f : A C R --t R is continuous on the compact set
K C A, then its range,
f(K) := {y E RI (3x E K) y = f(x)},
is also a compact set.
Solution. The set f(K) can be either finite or infinite. If f(K) is finite, then it is
both bounded and closed, i.e., by Definition 1.77 f(K) is then compact.
If f(K) has infinitely many elements, then for the compactness of f(K) it is
enough to prove that every infinite subset S C f(K) has an accumulation point
that belongs to f(K) (see Example 1.94). To that end, let us put
T:= {x E KI (3y E S) y = f(x)}.
Since f is a function, the last set is also infinite; being a subset of the compact set
K, T has an accumulation point Xo that belongs to K. We shall show that f(xo)
is an accumulation point of the set S. To that end, let (Xn)nEN be any sequence
from T that converges to Xo, such that for every n E N it holds Xn i- Xo. Since f is
continuous at Xo, using Definition 5.3, we have
lim f(x n ) = f(xo),
n ..... oo
c) f(x) = nlim
..... oo
y'3 + x 4n , X E R; d) f(x) = nlim
..... oo
(1 - sin 2n x), x E R;
e) f (x) = lim
n ..... oo
(x.
2 + 2 sm x
)4'
n
x E R.
Solutions.
a) It holds
0, xE[O,I);
lim xn = {
n ..... oo
1, x = 1;
+00, x> 1.
Hence the function f is equal to
1/2, o :s x < 1;
f(x) = { 1~3, x = 1;
x> 1,
and the function f has a discontinuity of first order at the point x = 1.
CONTINUITY 199
b) We have f(x) = sgnx, hence by Example 5.16 a) the function f has a discon-
tinuity of first order at the point x = 0.
f(x) = lim
n--->oo
\1'3 + x4n = 1.
For Ixl > 1 it holds
f(x) = n-+oo
lim Y/3+ x4n = n-+oo
limx 4 'Ql+ x4n
3 =X4.
d) The function f has removable discontinuities at the points (2k + 1)7r /2, k E Z,
since it equals to
7r 57r
12 sinxl > 1 ~ (3k E Z) "6 < Ix - hi < 6'
Hence the limit function f is
m
R(x) = { ~, X=-,
n
mEZ, nEN, lcd(m,n)=l;
0, x E R \ Q.
Compare a) to Example 5.10 b). In c), lcd( m, n) stands for the largest common
m
divisor of the numbers m and n. When lcd( m, n) = 1, the fraction - is irreducible.
n
Solutions.
m! x = m! E=
q
1 ·2 .. · (q - l)(q + 1)··· m· p,
hence m! x is an even number. This implies
(Vx E R \ Q) D(x) = O.
Hence for no x E R does the limit lim D(x) exist, which means that D
X--+Xo
lim rn
n~oo
= Xo and lim in
n~oo
= Xo.
Then it holds
lim f(rn)
n~oo
= xo·1 = Xo and lim f(i n) = Xo· 0 = O.
n~oo
This means that f has a second order discontinuity at every real point different
of zero.
Putting the results from (5.5) and (5.6) together, it follows that for every
sequence of real numbers the same remains true. But this means that R is
continuous at every irrational number.
Assume now that Xo is a rational number of the form Xo = m, where
n
lcd(m, n) = 1. By the definition of the function R, it holds R(xo) = .!.. Let us
n
put now
mk+l
rk kEN.
:= ~~
nk
This sequence of rational numbers converges to Xo as k -+ 00; however, it holds
0, x = OJ
f(x) = { 1/x, x E Q \ {O}j
x, x E 1:= R \ Q.
Solutions.
a) We have to prove that f is both an injection,
(VXl' X2 E R) Xl f:. X2 ::::} f(Xl) f:. f(X2),
and a surjection,
(Vy E R) (3x E R) y = f(x).
To that end, let us put
(Q\{O})UI=R\{O}.
Thus f is also a bijection on the last set.
Finally, since
(Vx E R) f(x) =0 ~ x =0
it follows that f is a bijection on the whole R.
b) We shall prove the continuity of f at the point x' = 1 by using Definition 5.3.
Let (xkhEN be an arbitrary sequence of positive real numbers converging to
x' = 1 j it is no restriction to assume Xk f:. 0 for every kEN. Our task is to
show
lim f(Xk) = 1 (= f(1)). (5.9)
k--+oo
lim Xk = l. (5.10)
k--+oo, kEAj
Let us define the number i as the following infimum:
i := inf{lxkll k E Ad.
Clearly, either i = 0, or, as we claim, i is positive. Namely, if i = 0, then there
exists a subsequence (Xk,)IEN of the sequence (xkhEAl which converges to O.
(Observe that each Xk, is nonzero.) But the existence of the last subsequence
is in contradiction with (5.10). Hence i > O.
By the definition of f, it holds f(Xk) =~, k E AI, and using the inequality
Xk
i > 0, it follows that
IXk -11
If(xk) - 11
IXkl
(5.11)
< (inf{lxkll k E Adtl ·Ixk -11 = i-I ·Ixk -11.
For given £ > 0, we choose kl E N such that
(Vk E At) k> kl =} (Ixk - 11 < i· E).
This implies
Further on, since f(Xk) = Xk for every k E A 2 , for given £ > 0 there exists
k2 E N such that
(Vk E A2) k> k2 =} (If(xk) - f(I)1 = IXk - 11 < E). (5.13)
Thus from (5.12) and (5.13) it follows that the following implication holds.
Hence the limit lim f(x) is certainly not 0 (in fact, it does not exist), thus
x-+O, xEQ
in view of Theorem 5.5, f can not be continuous at O.
c) We shall prove that f is discontinuous in each point of the set R \ {-1, 0,1}.
Let first Xo E Q \ {-1, 0,1}. Then for every kEN there exists an irrational
number Xk, Xk i: Xo, in the interval (xo - 1 ,Xo
-k + -k1 ), thus lim Xk = Xo. But
k-+oo
then
lim f(Xk) = lim Xk = Xo
k-+oo k-+oo
i: ~
Xo
= f(xo),
(Vc > 0) (3b > 0) (VXI,X2 E X) IXI - X2! < 8 =} If(xd - f(x2)1 < c. (5.15)
The following two theorems give two important properties of a continuous func-
tion on a compact set, resp. on a closed bounded interval (which is thus also a
compact set).
CONTINUITY 205
In other words, there exist two numbers Xm and XM in K such that for every x E K
it holds
f(xm) ::::; f(x) ::::; f(XM).
Theorem 5.32. Assume that a function f : A c R - t R is continuous on an
interval [a, bJ c A, and assume} for instance} that f(a) < f(b). Then for every y
such that f(a) < y < f(b), there exists an x E (a, b) such that f(x) = y.
Solutions.
a) Of course, the uniform continuity of f follows at once from Theorem 5.30, since f
is continuous on the compact set [3,6J. We shall, however, show that directly.
For every pair Xl, X2 E [3,6J it follows
for given E > 0 we can choose b := E/2. Then the following implication holds
1
Example 5.34. Prove that the function f(x) = -, x> 0, is
X
Solutions.
206 CHAPTER ,,).
a) Let 5 > 0 be given. For every pair XI, X2 from the interval (c, 1) it holds
b) In order to prove that f is not uniformly continuous on the open interval (0, 1),
1 1
we shall use the sequences Xn = -, n E N, and x~ = - - , n E N. Obviously
n n+1
both sequences converge to 0, i.e.,
lim Xn
n---+(X)
= n--+oo
lim x~ = O.
However, it holds
IXn - Xn'1_
- n( n
1+ 1,< 8.
Finally, let us prove the continuity of the function f at every point Xo E (0, 1].
For given 5 > 0 let us choose
Example 5.35. Discuss the uniform continuity of the following functions on their
domains.
a) f(x)=lnx, xE(O,l); b) f(x)=x·sinx, xE[O,+oo);
c) f (x) ={ . ;;-,
x . sm 1 x E (0, 1r) ;
d) f(x)
1
= eX . cos -, x E (0,1).
0, x = 0; x
Solutions.
a) We shall prove that the function f(x) = Inx, x E (0,1), is continuous at every
point Xo E (0,1), but is not uniformly continuous on the open interval (0,1).
Fix Xo E (0,1) and let c E (0,1) be given. The inequality
Put c := 1/2. Then for every 0 > °there exists an n = n5 E N such that
e-1
IX n - x~1 = en+! < o.
For such n it holds, however,
b) We shall prove that the given function f is not uniformly continuous on the
interval [0, +00).
. 1 ( n7r
1(-l)nsm-1
n
+ -n1) (2
> -. -1) (n7r + -)
7r n
1 > 2 > c.
n
We used the inequality
. 2
sIn X > -x,
7r °< X < "2.
7r
c) Let us observe first that If(x)1 ::; Ixi for all x E [0,7r).
°
Let c > be given; we shall assume that °< 6 < 7r (this is no restriction). If
xI, X2 E [0, ~], then it holds for 151 := 6/2 :
2
IXI - x21 < 151 '* If(xd - f(X2)1 < Xl + X2 < 6, (5.17)
6
If Xl, X2 E [2,7r], then
- x21
< IXI - x21 + 2X2 IXI2XIX2 ::; IXI - x21
1
+ -IXI
Xl
- x21·
Thus If(xd - f(X2)1 < IXI - x21 (1 + D.This shows that choosing 152 :=
2
_6_ the following implication holds
2+6
(VXI' X2 E [c,7r)) IXI - x21 < 152 '* If(xd - f(X2)1 < 6. (5.18)
Putting relations (5.17) and (5.18) together, we obtain that for given 6 > °
we can choose 15 as
2
6 6 }
15 := min {
2' 2 + c '
°
d) Since 1- (0,1), the given function is continuous in each point of its domain. We
shall prove next that it is not uniformly continuous on (0,1).
1 1 .
Let Xn = - - and x~ = (
2mf 2n + 1) 71" for n E N, hence It holds
lim
n~oo
Xn = n--+oo
lim x~ = 0.
IX n - x~1 = 1 < 8.
If(x n ) - f(x~) I /ex p C~71") cos 2n71" - exp C2n ! 1 )71" ) cos(2n + 1)71"/
1, x = 0;
Fl(X) =<
Sinx
x
°< x < 1;
sin 1, x = 1.
Clearly, FI is continuous on the closed interval [0, 1], hence it is uniformly continuous
there. This also gives the uniform continuity of f on the interval (0,1). Analogously,
the function F2 given by
- sin 1, x = -1;
SlllX
F2(X) =< x
-1 < x < 0;
-1, x = 0.
is uniformly continuous on the closed interval [-1,0]' and equals to f on (-1, 0).
Hence f is uniformly continuous on (-1,0).
210 CHAPTER 5.
However, we shall prove next that f is not uniformly continuous on the union
1 - 1
Let Xn = --- and x~ = - - for n E N. These two sequences are from A and
n+l n+l
both tend to zero as n --+ 00. For given 8> 0, there exits n = no such that
If(xn) - f(x~)1
Isin (~)I_lsin (~)I
1 1
n+l n+l
2(n+l) I
1
. lll-- 1
S
n+l
~2(n+l). (2- .n+l
7r
-1)
- 4 =-.
7r
Hence for (say) E = 1, it holds If(xn) - f(x~)1 > E, proving that f is not uniformly
continuous on the set (-I,O)U(O,I).
Solution.
a) Let us prove first that a bounded, monotone and continuous function on the open
interval ( a, b) has a right-hand limit at a. Assume, say, that f is monotonically
increasing on (a, b). Since f is bounded on (a, b), it follows that the range of
the function f,
is bounded from below. By Example 1.50 a), it follows that the set B has an
infimum, which we shall denote by L. Let us show that
lim Xn a.
n->oo
L':= n-+oo
lim f(xn). (5.20)
L, x = a;
F(x) := { f(x), a < x < b; (5.22)
K, x =b.
b) In the same way as in a), one proves that there exists the
lim f(x)
x-+a+
= L.
Let us assume, say, that f is monotonically increasing. Then from the bound-
edness of f it follows that the set
Let 6 > °be given. Then there exists a number T > a, depending on 6, such
that
6
Vx >T =? IF(x) - KI < 4. (5.23)
In view of a), F is uniformly continuous on the set [a, TJ, hence f is uniformly
continuous on (a, T]. This means that there exists a 5 > such that °
(VXI,X2 E (a,T]) (lxI - x21 < {j =? If(XI) - f(X2)1 < ~). (5.24)
We have still to prove that f is uniformly continuous on the set [T, +00) as
well. To that end, from (5.23) it follows that for Xl > T and X2 > T it holds
If(xt} - f(X2)1 < If(XI) - K +K - f(X2)1
6 6 6 (5.25)
::; If(xt} - KI + IK - f(X2)1 ::; 4 + 4 = 2·
Hence IXI - x21 < {j =? If(XI) - f(X2)1 < 6/2, and from relations (5.24) and
(5.25) it follows that f is uniformly continuous on the set [T, +00).
Remark .
• The function F from (5.22), continuous on [a,b] and equal to f on (a,b), is
called the continuous extension of f from ( a, b) to [a, b] .
• The assumption on monotonicity of f in Example 5.37 is essential. Namely,
the function
g(x) = sin(l/x), x E (0,1)
is bounded and continuous on (0,1), but can not be continuously extended to
[0,1] (see Example 5.15 c)).
Exercise 5.38. Show that the function f is uniformly continuous on the set X
given below.
a) f(x) = sinx, X = R; b) f(x) = cosx, X = R;
Answers. In a), b) and c), the functions are uniformly continuous on their do-
mains, while in d), f is not uniformly continuous, but only continuous.
Chapter 6
Derivatives
6.1 Introduction
6.1.1 Basic notions
Definition 6.1. Let f be a real function defined on an open interval (a, b) and let
Xo E (a, b). Then the following limit
Besides the notation f'(xo) for the first derivative of the function f at Xo one also
uses the notation ix(xo).
Occasionally, instead of f'(xo) we shall write f~(xo) to emphasize the variable x
in which the derivative is found.
The number h in (6.1) is called the increment of the independent variable
x at the point Xo, while the difference f(xo + h) - f(xo) is called the increment of
the dependent variable at the point Xo. Thus relation (6.1) can be interpreted
as the limit of the quotient of the increments of the dependent and the independent
variable, when the latter tends to zero.
One can also define the one sided derivative of a function at some point Xo
from its domain as the one sided limit of the quotient f(xo + h~ - f(x o). The right-
hand side and the left-hand side derivative of f at Xo are denoted by f~(xo), and
f~ (xo). In the next table let us give the first derivatives of the most commonly used
elementary functions, together with the largest subsets of R on which they exist.
1. (xn)' nx n - 1, nEZ, x E R.
213
214 CHAPTER 6.
3. (sinx)' cosx, x E R.
4. (cosx), = -sinx, x E R.
+ l)il k E Z}.
1
5. (tan x)' , x E R \ {(2k
cos 2 X
-1
6. (cot x)' = -'-2-'
SIn x
x E R \ {hi k E Z}.
1
8. (loga x)' = -~-,
X· na
a> 0, ai-I, x> 0, (lnx)'
x
,x> O.
1
9. (arcsin x)' = .Jf=X2' Ixl < l.
-1
10. (arccos x)' = .Jf=X2' Ixl < 1.
1
11. (arctan x)' = 1 + x2 ' x E R.
-1
12. (arccot x)' x E R.
1 + x2 '
If the functions f and 9 are defined on an interval (a, b) and have first derivatives
at the point x E (a, b), then
f'(x)g(x) - f(x)g'(x)
( f(X))' provided that g(x) i- O.
g(x) g2(X)
DERIVATNES 215
Assume that the function 9 : (a, b) -+ (c, d) has a first derivative at Xo E (a, b)
and assume that the function f : (c, d) -+ R has a first derivative at the point
g( xo) E (c, d). Then the first derivative at Xo of the composite function
h = fog, h: ( a, b) -+ R,
exists and the so called chain rule holds:
Assume that two differentiable functions y = y(t) and x = x(t) of the same vari-
able ("parameter") t E (a, (3) are given. Then the function y = f(x), determined as
the mapping f that assigns to x = x(t) the number y = y(t) is called a parametric
function.
The first derivative of the parametric function y = f( x) at the point Xo is
given by
y'(xo) = y~(to) to E (a, (3) (6.6)
xWo) ,
where x(to) = Xo. Clearly, formula (6.6) has meaning only at those points to, where
x and y have derivatives and it holds x~(to) #- o.
216 CHAPTER 6.
Definition 6.2. If the increment b.y of the function f : (a, b) -+ R at the point
Xo E ( a, b) can be written in the form
b.y = f(xo + h) - f(xo) =D .h + r(h) . h,
for some number D (independent from h), and it holds
lim r(h) = 0,
h--+O
then for the function f we say that it is differentiable at the point Xo.
Assume a function f has a first derivative at a point Xo. If 0 ::; 0: < 7r is the angle
between the tangent line at the point Xo and the positive direction of the x-axis,
then it holds
tan 0: = f' (xo). (6.11 )
In other words, the slope of the tangent line of the graph f at some point is exactly
the value of the first derivative of f at that point.
lThis theorem is not true for functions of two or more variables.
DERIVATIVES 217
Solutions.
VI + I + h - y'2 r I _ _1_
/'(1) = l~ h = h~ v'2+7i" + y'2 - 2y'2'
v'f+h - v1 1
1'(0) = hlim
..... O
h = -2'
Geometrically, the last equality means that the vertical line x -1 is the
tangent line to the graph of f at the point (-1,0).
c) It holds
1 + h + (1 +h - 1) arcsin fl+h - 1
/,(1) = lim
h ..... O h
VI+h+l = 1 + 7r4·
218 CHAPTER 6.
Answers.
c) 1'(-3) = -3.
Example 6.6. Find by definition the first derivatives of the following functions.
a) f(x)=x 2 +2x, xER; b) f(x) = <Ix - 1, x E R;
1
= 30x- 1)2
f~(I) = lim
h-+O-
\Ih -h <10 = +00,
which means that the tangent line of the graph of f at x = 1 is parallel with
the y-axis.
DERIVATNES 219
1 (
--lnlim 1 + - -
h) (x+1)/h I 1
x +1 h--+O X +1 x+l
d) We have
f'(x) = lim cos(2(x + h)) - cos(2x) = lim -2sin(2x + h)· sinh = -2sin(2x).
h--+O h h--+O h
Exercise 6.7. Find by definition the first derivatives of the following functions. at
the point x E R.
a) f(x)={ll+x4, xER;
2e2x
e) f(x)=ln\4J , xE(O,7r/2);
1 + cos x
Solutions. We leave to the reader to find the natural domains of the given functions
and the largest subsets of R on which their first derivatives exist.
a) Let us put u(x) = 1 + X4. Then from the chain rule it follows that
f'(x)
1
= -3 (u(X)t 2/3u'(X) = \I 4x
3 (1 + x )2
3
' 3
4
x E R.
3
b) f'(x) = x . 1 1
6/1 + \/1 + ~1 + x4 \/(1 + ~1 + x4)2 . (V/f+X4)3' x E R.
220 CHAPTER 6.
3 3 tan 2 X 7r
C) f'(x)=32tanxln3·(2tan3x)'=61n3·32tanx_-, x-=l-(2k+1)-, kEZ.
COS 2 X 2
1
f'(x) = sm arccot eX (sin arccot eX)'
1
sm arccot eX cos arccot eX. (arccot eX)'
_eX e2x
cot(arccot (e X ))--2- = ---2-' X E R.
l+e x l+e x
e) Since it holds
1 2x 1 2 X X 1 X
f(x) = -In(e ) - -lncos - = - - -lncos-
4 4 2 2 2 2'
we obtain
f'(x) = ~ + ~tan~, X E (0,7r/2).
. Inx .
f) Smce loga X = -1-, for X > 7, a > 0 and a -=I- 1 (prove that!), It follows that
na
1
f'(x) n 31og 5 1og 7 x (log5 log7 x)'
1 1 1 1 1
-----_.---.-.
log5 log7 X log7 X x In 31n 5 In 7 x In 31n x In log7 x .
Answers.
1
a) f'(x) = coshx. b) f'(x) = sinhx. c) f'(x) = cosh2 X
-1 1
d) f'(x) = sinh2 x e) f'(x) = 'x2 +1 f) f'(x) =~
DERIVATIVES 221
1 2x
d) f(x) = -arctan--2 , x E (-1,1);
2 1- x
x ex
e) f(x)=ex+ee +ee , x E R;
c) f'{ x) = 2 sin cos cos cos 2x . sin cos cos 2x . sin cos 2x . sin 2x.
This first derivative can be found also by taking the logarithm of the function
y = XX and then differentiating the obtained expression.
In fact, from In y = x In x it follows
y'
- = In x
y
+ 1 =} y' = y(1n x + 1).
Putting now y = f (x) = XX in the last equality gives again
vx)
it follows
1'(x) =ev'X lnx (lnx + =xv'X.lnx+2
2VX x 2VX .
f '()
X = ecosxlnsinx ( - Sln
. x 1n sm
. x + -cos.-2 x) .
slnx
Example 6.12. Prove that a differentiable function f (a,b) ----7 R at the point
Xo E (a, b) is also continuous at that point.
But then for given c > 0there exists 0 := ~ min {2ID~ + 1,01} such that for Ihl < 0
it holds
c
If(xo + h) - f(xo)1 ::; (IDI + Ir(h)l)lhl < (21DI + 1)21DI + 1 = c.
Example 6.13. The function f is defined by f(x) = lxi, x E R. Check whether
. -Ihl-
h- 0 -_ ~
h = 1, f~(O) = lim Ihl- 0
I +, (0) = lIm
h-+O+ h-+O- h = h-h = -1.
Since these one sided derivatives are different, f has no first derivative at O.
Remark. This example shows that a continuous function at some point does not
necessarily have a first derivative at that point.
c) Once we have proved in b) that f has no first derivative at the point 0, it follows
at once from Theorem 6.3 that f is not differentiable at O. Still, we shall prove
that f is not differentiable at 0 without using that theorem.
To that end let us write the increment of f at the point x = 0 in the form
f(h) - f(O) = D· h + r(h)· h ~ Ihl = (D + r(h»h,
where h is the increment of the independent variable. We shall show that
lim r( h) does not exist, whatever value one chooses for the constant D. In
h-+O
fact, it holds
h>0 '*
r(h) = 1 - D
and
h<0 '* r(h) = -1 - D.
Since 1 - D "# -1 - D for every D, the limit at zero of the remainder r( h)
does not exist, giving us the non differentiability of f at O.
224 CHAPTER 6.
Example 6.14. Find the largest sets on which the first derivative of the function f
lS
(i) continuous;
(ii) differentiable;
. 1 1
e) f(X)={
x sIn-,
x
x i= 0; f) f(x) = {
2'
x sm;, x i= 0;
0, x = 0; 0, x = o·,
. 1
g) f(x) = {
x"" sIn-, x i= 0; were
h ex . a poSt't'we parame t er.
lS
x
0, x = 0,
Solutions.
b) (i) The function f is continuous on the set R \ {O}. Note that one can not discuss
the continuity of f at 0, since f is not defined there.
J'(x) =
X·
1
sgnx
sgnx =
x
x .!., i= o.
DERIVATNES 225
1
1'(x) = -, xi- 0
x
(ii) Since f(x) = I(x + 2)2(X - 1)31 = (x + 2)2(X - 1)3sgn(x - 1), x E R, the
first derivative of f forx i- 1 is
and, similarly, the right-hand side derivative at the point 1 is also o. This
means that 1'(1) = o.
(iii) The function l' is continuous at the whole set R. In particular, it holds
lim1'(x) = lim(x
x--+I x--+l
+ 2)(5x 2 - X - 4)lx -11 = 0 = 1'(1).
We suggest to the reader to prove this theorem, and also to find a coun-
terexample that will show that the opposite is not true in general.
f '() . -1 -
x = SIn 1
- COS - .
1
x x x
. 1
h sm - - 0 1
lim
h--+O
~, = h--+O
lim sin -h
f '( x ) = 2x sm
. -1 1
- cos -,
x x
while for x = 0 we have
•
h 2 sm 1
- - 0 1
1'(0) = h--+O
lim hh
•
= h--+O
lim h sin -h = 0, ( 6.12)
does not exist, the function f' has a second order discontinuity at the point O.
Remark. The function f' obtained as a derivative of some function f, can never
have first order discontinuities, although, as we just saw, it might have second order
discontinuities.
Example 6.15. Find the largest sets on which the first derivatives of the following
functions exist.
2- x x < 2;
a) f(x) = { (2 - x)(3 ~ x), 2 ~ x ~ 3;
-(3 - x), x> 3;
arctan x, Ixl ~ 1;
-
{ 7r Ixl- 1
b) f(x)- "4sgnx+ 2 ' Ixl > 1;
x3 + 1, x ~ 0;
c) f(x) = { e- 1jx + 1, x> 0;
X, x E Q
{
e) f(x) = 0, x E R \ Q.
Solutions.
a) From the limits
lim f(x)
x--+2-
= x--+2+
lim f(x) = 0 = f(2)
and
lim f(x)
x->3-
= x->3+
lim f(x) = 0 = f(3)
it follows that f is continuous at the points 2 and 3 respectively, thus it is
continuous on R. Let us see what happens with the first derivatives of f at
the points 2 and 3. We have
f~(3) = f~(3) = 1,
hence 1'(3) = 1. Thus the function I' (the first derivative of f) exists on the
whole R and it holds
-I, x ~ 2;
f'(x) = { 2x - 5, 2 < x < 3;
1, x::::: 3.
228 CHAPTER 6.
. 1 -1+h+1 1
hm -h arctan 1 - ( -1 + h)
h-.O+
-,
2
lim =-f(~-_l_+_h...:....) - f ( -1 )
f!..( -1)
h-.O- h
1r
-sgn(-l+h)+
I - 1 + hi - 1 1r
-(--) 1
lim 4 2 4 - __
h-.O- h 2'
hence f has no first derivative at -1. In a similar way we obtain
f~(l) = f~(l) = ~,
hence l' (1) exists and equals ~.
So we get
1
1 + x2 ' - l',
-1 < x <
1
1'(x) = { x < -1;
-2'
1
x>l.
2'
c) For later purposes, let us show first that
. e -I/x
11m --=0
x-.O+ xm
1
for every m = 0,1, ... Putting t:= - gives that t ----+ +00 when x ----+ 0+ .
x
Hence
-I/x
lim _e__ = lim tme- t = O. (6.13)
x-.O+ xm t-.+oo
(Note that the easiest way to prove the last equality is to apply m-times the
L'Hospital's rule, see Section 6.4.)
On the set R \ {O} the function f is continuous. From (6.13) for m = 0 it
follows that
lim f(x) = lim e- 1/x + 1 = 1,
x-.O+ x-.O+
DERIVATIVES 229
f~(O) = 0,
e) This function is continuous at the point 0, since it holds If(x)1 ::; Ixl for every
x E R. In other real points f is not continuous (one can use the method from
Example 5.10 b)).
Clearly, the only point where f might be differentiable is the point O. In fact,
we shall prove that f is not differentiable at O. Let us examine the quotient
does not exist. This means that the set on which f' exists is empty.
Exercise 6.16. Assume that the domain A of the function f is a symmetric neigh-
bourhood of zero. Then
230 CHAPTER 6.
Example 6.17. Find the first derivative of the inverse function f- 1 for the given
function f, where
Solutions.
• Second method. Since f'(x) = 2 and f'U- 1 (x)) = 2 from the formula
(6.3) it follows
U- 1 )'(x) = __. 1
1
2' x E R.
b) In this case we shall use the Second method from a). To that end, we have
hence the first derivative of the inverse function f- 1 to the given function f is
U- 1 )'(x) = 2.) f-l(x) = 2(x - 2), x> 2.
c) From the equation x = f(y) = y2 - 2y, it follows f'(y) = 2y - 2. Thus the first
derivative of the inverse function f- 1 to f in variable y is
1 1
U- 1 )'(y) = 2y _ 2 = -,
Since (y - 1)2 = X + 1, we get finally
U- 1 )'(x) = ~ ~, x>-1.
DERIVATNES 231
1 1
Jl - cos 2 ( arccos x) Vf=X2'
XE(-I,I).
(f-l)' = (arcsinhx), 1 1
cosh(f-l(x)) cosh(arcsinh x)
1 1
VI + sinh (arcsinh x)
2 ~.
1
f) (arccosh x)' = .../x 2 _ l' x>1.
Example 6.18. Find the domains and the first derivatives of the inverse functions
of the following functions.
x2
a) f(x)=x+lnx, x>O; b) f(x) = --2' X
l+x
< o.
Solutions.
a) For x> 0 it holds f'(x) = y~ = x + 1 > 0, hence there exists a unique function
x
f- 1 whose domain B is the range of f, and is inverse to f. Clearly, B = R;
unfortunately, it is impossible to find the analytic formula for f- 1 explicitly.
Still, we shall find its first derivative. Namely by relation (6.3) it follows:
1 1 X
(f-l )' (y) = y~ = x + 1 = x + 1 '
x
(f-l)'(y) = (1 + X2)2 x3
2x 2y2 '
2
where x = x(y) in the solution in x of the equation y = ~.
1+x
232 CHAPTER 6.
Example 6.19. Prove that there is but one differentiable function on R such that
y3 + 2y = x, y(3) = 1, (6.14)
Solution. The existence of the function y = y( x) follows from the Implicit function
theorem, which is somewhat out of the scope of this book. However, in this case we
can prove its existence in the following way.
The function
F(X,y)=y3+2y-x
x~ = 2 + 3y2 > O.
Notice that we observed x as a function of y. Now the function x = x(y) is
monotone for y E R. Hence the inverse function of x = x(y) exists and is also
differentiable on R. Then it holds (see (6.3))
I 1 __ 1 •
Yx = '7 - 2 + 3y2
y
Let us assume that there are two solutions of equation (6.14) and let us denote
them by Yl (x) and Yz (x). Then it holds
This implies
1
y~ = 2 + 3y z'
DERIVATIVES 233
Example 6.20. Find the first derivatives of the following functions y = y( x) given
implicitly.
a) x 2 + y2 = 4; b) 2x-3y+3=x 2 +2y-6x;
1 y'
c) It holds that r; + -- = 5, hence
2yx 2V'Y
y' = 10V'Y - ~.
d) Firstly we have
4x 3 + 8xy2 + 8x 2yy' - 3y 3 - 9xy2y' +3 = 0,
which implies
, -4x3 - 8xy2 + 3 y 3 - 3
y =
8x 2y - 9xy2
Solutions.
6t 2 - 6 3t 2 - 3 t i- -1.
y~ = 2t +2 - t +1 '
y' _ sin t
- (1 _ cos t)' t E R \ {2k1r1 k E Z}.
. 2
3 sm
y' = t cos t -1
-6 cos 2 t sin t - 2cot t' t E (0, 7r /2).
Example 6.22. Find the differentials of the following functions given below.
R+
sm-
x 2x
1'(x) =
x 2 cos - 1+x 2 '
X
it follows (see relation (6.7)) that the differential of f at the point x from the
domain of f is
df (x) = 1'( x) dx = ( R1 )
x2
sin -
1
cos-
X
2x
+ 1 +x2
d
X.
X
DERIVATNES 235
c) It holds
df(x) = (cosx + 1/ cos 2 x)(x 3 + 3'" + 3x) - (sinx + tgx)(3x 2 + 3"'ln3 + 3) dx.
(x3 + 3'" + 3x)2
Example 6.23. If f(x) = \ju 2 + v 2, where mEN, m > 1, while u = u(x) and
v = v(x) are differentiable functions of the variable x, find the differential df(x).
2
-( u 2 + v 2 )(1-m)/m( U du + v dv).
m
Example 6.24. Find the approximate value of the number A, whose exact value is
Solutions.
a) For the function f(x) = ,;x, x ~ 0, its first derivative is f'(x) = 2~' x> 0,
hence for x = 4, h = 0.0003, it follows from formula (6.8)
1 0.0003
J4.0003 ~ v'4 + 1A0.0003 = 2 + - - = 2.000075.
2v 4 4
where a > 0, n E N, while the number x satisfies Ixl ~ an (i.e., Ixl is much smaller
than an).
where the remainder r satisfies lim rex) = 0, see Definition 6.2 and Theorem 6.3.
x->o
Thus neglecting the addend x . rex) in (6.16), we obtain the approximate formula
(6.15), provided that Ixl is much smaller than an.
Exercise 6.26. Prove the following approximate formulas for "small" Ixl.
1 x
a) ~::::Jl+-; b) sin x ::::J x; c) cos X ::::J 1.
1- x 3
Example 6.27. Assume that the functions u = u(x) and v = vex) are n times
differentiable on an interval (a,b), where n E N. Then using the usual convention
u(Oj(x) := u(x), the following, so-called Leibniz formula, holds:
Solution. For n = 1 the relation (6.17) reduces to the well known formula of the
product of differentiable functions.
Assume that relation (6.17) is true for n = k; we have to show that then it is
true for n = k + 1. It holds for x E (a, b)
= ~ (k ~ 1)UU)(X)V(k+ 1 -j)(X).
3=0 J
Thus we obtained relation (6.17) for n = k + 1, once we assumed its correctness
for n = k. By the principle of the mathematical induction, it follows that (6.17) is
correct for all n EN.
The reader should note that we used the equality
G) + C~l) G:D, =
which is true for all kEN and 0 ~ j ~ k - 1, see Example 1.28 a).
Example 6.28. Assume that the function f is three times differentiable on its do-
main. Find g" and gill, if the function 9 is given by:
a) g(x) = f(x 2 ); b) g(x) = f(1/x);
g' (x) = - ~
x
f' ( 1/ x ), g" ( x) = 23 f' ( 1/ x)
x
+ ~ I" (1/ x ) ,
X
a- =
p~j)(O)
-., '
0 < j ~ n.
_ (6.18)
3 J.
238 CHAPTER 6.
1
e) f(x) = 1 _ x' Ixl < 1; f) f(x) = In(1 + x); Ixl < 1.
In particular, find f(j)(O) for j EN.
Solutions.
a) Since f'(x) = eX and f"(x) = eX for every x E R and using the mathematical
induction on j E N it follows that for all j EN, f(j) (x) = eX. Hence f(j) (0) =
1, for all j E N.
b) Similarly as in a), we have f(j) (x) = 2X Inj 2, for every x E Rand j EN. Thus
f(j)(O) = ln j 2, j EN.
f) The first and the second derivative of the function j(x) = In(I + x), Ixl < 1, are
1
a) f(x) =ry ... ", x ~ {I,2}; b) f(x) = sin 2 x, x E R;
+ cos 4 x, a + bx
c) f(x) = sin 4 x x E R; d) f(x) = In --b-' a2
a- x
- b2 x 2 > o.
Solutions.
b b
d) From f'(x) = - - + - - we get
a + bx a - bx
n (n-l)!b n
f (n)( x ) -_ (-I)n-l(n-l)!b + .
(a + bx)n (a - bx)n
Answers.
( -1)nn! b) f(n)(x) = -(n - I)!
a) f(n)(x) = (1 + x)n+1 (1 - x)n .
3 . 1.
c) f(x)=-smx--sm3x () 3n . ( 3 x + - .
3 . ( x + - --sm
fn(x)=-sm n1l') n1l')
4 4' 4 24 2
1 - cos 4x cos 3x 1
d) f(x)= 2 cos3x=-2--4'(cos7x+cosx),
n
3 cos ( 3x+T
f(n)(x)=T n1l') -4'1 ( 7ncos ( 7x+T
n1l') +cos (n1l'))
x+T .
e-l/X, x> 0;
{
f(x) = 0, x::; O.
f'(x) = ~e-l/X
x2 '
1 2) e -1/
f "( X ) -_ ( - - -3
x4 x
x
'
f ill (
X
) -
-
(
---+-
1
x6
6
x5
6) e -1/x .
X4
f(n)(x) = Q2n(1/x)e- 1 / X,
DERIVATIVES 241
where Q2n(X) is a polynomial of degree 2n such that Q2n(0) = o. Thus (see Section
6.4) we have
lim f{n)(x)
x-+o-
= x-+o-
lim 0 = O.
Thus the right and the left-hand side limits of f{n) at 0 are both equal to zero.
We still have to prove the existence of f{n)(o). Of course, In)(O) = 0, for all
n EN. Further on we have
r hQ2n(1/h)e-l/h
h2.rr+ 1 = O.
Hence by the principle of the mathematical induction it follows that f~n)(O) = 0 for
all n E N, which implies the existence of f{n) on R.
Remark. This example shows that there exist infinitely differentiable functions
whose Maclaurin's polynomial of any degree is identically equal to zero, even though
the function is not identically equal to zero.
Example 6.34. Find the equations of the tangent and perpendicular lines at the
given point T(xo, f(xo)) on the graph of the following functions.
a) The equation of a line with slope k passing through the point T(xo, Yo) has the
form
y-yo=k·(x-xo).
The geometric interpretation of the first derivative at the point Xo is the tan-
gent line of the graph of f at its point T(xo, Yo), i.e., k = f'(xo) (see relation
(6.11)) together with f(xo) = Yo).
242 CHAPTER 6.
f'(x) = 2~'
hence putting Xo = 4 gives us f' (4) = ~. The value of f at Xo = 4 is Yo =
f(4) = 2. Thus the equation of the sought tangent line has the form
1
y-2=4(x-4), or 4y-x=4.
The equation of the perpendicular line of the graph f at its point T(xo, Yo) is
of the form (see equation (6.10))
y - Yo = k1 . (x - xo),
where kl= - ~, provided, that the slope k( = f' (xo)) is nonzero. Hence
k1 = -4, which gives the equation of the perpendicular line at the point
T(4,2),
y + 4x = 18.
b) From the first derivative f'(x) = 2xex2 - 1 it follows that the slope k of the
tangent line is k = f'(I) = 2. Hence the equation of the sought tangent line
has the form
y -1 = k(x -I):::} y -1 = 2(x -1),
or finally
y - 2x +1 = O.
The equation of the perpendicular line of the graph f at its point T(I, I) is of
the form
2y + x - 3 = O.
c) From y' 2x 4 it follows that y'(O) o. Thus the tangent line has the
l+x
equation
y- 0 = 0 . (x - 0) hence y = O.
The sought tangent line is, in fact, the x-axis. Clearly, the perpendicular line
of the graph f at the point T(O,O) is the y-axis, whose equation is x = o.
d) The function f has for its natural domain the interval [-4,0), the absolute value
of the argument of the arcsin function must be less than or equal to one. It
is easy to show that f is continuous from the left-hand side at the point O.
However, f has no left sided derivative at the point 0, since
Iim t - 7r /2 1· z 1. z
= 1m = 1m = +00.
t_tr/2- 2(sint -1) z_O- 2(cosz -1) z_O- -4(sin 2 (z/2»)
DERIVATIVES 243
This shows that the sought tangent line is the y-axis, whose equation is x = 0.
7r
The equation of the sought perpendicular line at the point T(O, 2") is y = 0,
which is, in fact, the x-axis.
Example 6.35. Determine the parameter k so that the line y = kx + 1 becomes the
tangent line of the parabola
Solution. The given curve is the union of graphs of two functions, namely of
Let us denote by T(xo, Yo), Xo > 0, the common point of the tangent line and the
parabola. Then
1
Yo = ±2JXo and k = ± Fa'
This gives
1
Yo = kxo + 1 =} 2JXo = --Xo + l.
Fa
So we obtain Xo = 1 and its common point with the parabola and the graph of It is
T(1,2). The equation of the sought tangent line is y = x + l.
Notice that there is no tangent line to the graph of h, since
1
-2JXo = ---Xo + 1 =} JXo = -1 < 0,
Fa
a contradiction. In fact, at the point x = 0, the graph of the function h and also
the given parabola have a vertical tangent line.
Example 6.36. Find the tangent lines to the parabola f( x) = x 2 - 3x + 1 through
the point A(2, -2) and determine the common point of each tangent line and the
parabola.
Solution. Let us denote by (xo, f(xo)) the common point of the tangent line and
the given parabola. The slope of the tangent line is k = f'(xo) = 2xo - 3, while its
equation is
y - f(xo) = k(x - xo), or
-2 - (x~ - 3xo + 1) = (2xo - 3)(2 - xo).
This implies the quadratic equation
y +x = ° and y = 3x - 8.
244 CHAPTER 6.
The intersection of this line with the coordinate axes are the points A (_a_,
cos t
0) and
B (o,~)
smt
.Hence the area A~AOB of the triangle AOB satisfies
1 a b -~>ab.
A~AOB = "2 cos t sin t - 2 sin 2t -
• f(a) = f(b),
then there exists at least one number eE (a, b) such that 1'(0 = o.
Theorem 6.39. The Lagrange Theorem.
If a function f : [a, b] --+ R is
• continuous on a closed interval [a, b],
• differentiable on the open interval (a, b),
then there exists at least one number eE (a, b) such that
f'(e) = f(b) - f(a) (6.19)
b- a .
Geometrically, Theorem 6.38 (resp. 6.39) means that the tangent line at the point
eis parallel to x-axis (resp. to the line y - f(a) = f(b~ - f(a) (x - a) ).
DERIVATNES 245
Solution. The given function is polynomial, hence it is continuous and has first
derivatives at every point of these intervals. Since it holds f(O) = f(l) = f(2) = 0,
the function f satisfies the conditions of Rolle's theorem on all three intervals [0,1],
[1,2] and [0,2].
We have I'(x) = 3x 2 - 6x + 2, and
So the points in which the first derivative of the considered function is equal to zero
are
6 =1- 3
v'3 E [0,1], and ~2 = 1 + 3v'3 E [1,2].
On the interval [0,2] the function I' has two real zeros, ~1 and ~2'
Example 6.41. On the intervals (-1,1) and (1,2) find the points in which the
tangent lines of the graph of the function f(x) = (x 2 -l)(x - 2) are horizontal.
Solution. The considered function is continuous and has a first derivative at every
point of these two intervals and it holds that f(-l) = f(l) = f(2) = 0. Hence f
satisfies the conditions of Rolle's theorem on the intervals [-1,1] and [1,2], and also
on the interval [-1,2].
From 1'( x) = 3x 2 - 4x - 1 it follows that 1'( x) = for °
~1 = 2 - n v'7 E [-1,1], ~2 = 2 + v'7
n
E [1,2].
The tangent lines of the graph of the function f(x) = (x 2 - 1)(x - 2) in the
points 6 and 6 are horizontal (1'(6) = 1'(6) = 0).
Example 6.42. Check whether the following functions satisfy the conditions of
Rolle's theorem.
1 - ij;2
a) f(x) = 2 on the interval [-1,1];
d) f(x) = t ~i
i=1 z
sin(ix) on the interval [0, 7r/2].
246 CHAPTER 6.
Solutions.
a) No, because the function f has no first derivative at the point x = 0, as seen
from either of the following one-sided derivatives at 0:
lim f (0 + h) - f (0) ~ W 1
h-.O+ h I'1m 2
h-.O+
- 2h -2 = -00',
lim f(O
h-.O-
+ h) - f(O) =
h
lim
h-.O-
-w
2h
= +00.
b) No, because the function f has no first derivative at the point x = 1 (see Example
6.13).
has a solution on interval [0, 7r]. Namely, the first derivative of the considered
function is just the left side of the equation (6.20).
d) No, because it is not true that f(O) = f(7r/2). Namely, the value f(7r/2) depends
on n and on the coefficients ai, i = 1,2, ... , n.
with real coefficients has only real roots, then its derivatives, P~, P::, ... , p~n-l), have
real roots as well.
Solution. Let us suppose first that all roots are simple. Then from Rolle's theorem
it follows that there exist n - 1 real roots of the polynomial P~ (x). Every root
of the polynomial P~(x) is located between the two roots of Pn(x). Similarly we
obtain that the polynomial P::(x) has n - 2 real roots, and so on. Note that the
(n - l)-th derivative
p~n-1)(X) = ann! x + an-1(n - I)!
p~n)(x) = ann!
is a constant.
If a number Xo is a multiple real root of order m > 1 of a polynomial Pn (x),
then it is also a root of the polynomial's derivative, as follows immediately from the
representation
Pn(x) = an(x - xo)mQn_m(x).
DERIVATNES 247
Solution. The polynomial Rn(x) = (x Z - l)n has on the interval [-1,1] exactly
2n real roots, namely
then there exists at least one point c E (a, b) satisfying f'(c) = O. Prove.
Solution. First we shall assume that (a, b) is a finite interval and let us denote by
C the limits in relation (6.23). Then the function
2 dn 2
Hn(x) = (_l)ne X dxn (e- X ),
c) the equation,
dn
(1 + x2)n dxn ((1 + X2)-I) = 0,
Solutions.
obtain that the function f(n-l)(x) has n real roots including the point x = 0,
while f(n)(x) has n roots also, because f(n)(o) -=I- O. Therefore the polynomial
Solution. It is trivial that if a function is a constant on the interval [a, b], then its
derivative is zero on ( a, b).
So we have to prove the opposite, i.e., if 1'(x) = 0 for every x E (a, b), then it
holds that f(XI) = f(X2) for each xI, X2 E [a, b].
DERIVATNES 249
From Lagrange's theorem it follows that there exists a point eE (Xl, X2), hence
eE (a, b), such that
f(xt} - f(X2) = f'(O(XI - X2).
From the supposition f'(x) = 0 and the last relation we obtain f(xt} = f(X2) for
each pair Xl, X2 E [a, bj.
Example 6.48. If the derivatives of two functions FI (x) and F 2 ( x) are equal on
some interval, i. e., it holds
Example 6.49.
Solutions.
a) Assume that f'(x) > 0 for all X E (a, b), (resp. f'(x) :2: 0) for all X E (a, b). If
Xl, X2 are two points from the given interval and Xl < X2, then by Lagrange's
e
theorem there exists a point E (Xl, X2) such that
b) Analogous to a).
1 + v'I3 1- v'I3
we obtain 6 = ~ E(-1,1)ande2= c E(-l,l).
DERIVATNES 251
x-y x x-y
c) - - < I n - < - - , 0< y < x;
x y y
x
d) - - < In(1 + x) < x, x> 0;
l+x
e) eX> 1 + x, x E R;
a
h) -+1
n'"
< (1
( ) - -1),
n - 1 '" n'"
n E N, a> O.
Solutions.
a) Applying Lagrange's theorem to the function f(t) = sin at, which is continuous
and differentiable on any interval [y, x], we obtain that there exists a point
e E (y,x), such that it holds
sin ax - sin ay = a( x - y) cos ae, wherefrom
Ilnx-Inyl =-e
Ix - yl
and
1
-<-<-
x e
1 1
y'
we obtain the given inequality.
252 CHAPTER 6.
d) Using the function f(t) = In(1 + t) on the interval [0, x], and the relation
In(1 + x) -In(1 + 0) 1 1 1
- - t ' for ~ E (O,x), hence - - < - - t < 1,
x 1+., l+x 1+.,
we obtain the inequality.
e) The function f(x) = eX on the interval [0, x], x> ° (resp. on [x, 0], x < 0),
satisfies the conditions of Lagrange's theorem,
eX _eo
- - = e E =} eX - 1 = xe E.
x
f) Taking the same function as in the previous case and the interval [1, x], we have
eX _ e l
- - - = eE > e, for some ~ E (l,x).
x-I
h) Taking the function f (x) = ~ on the interval [n - 1, n], we obtain the given
x"
inequality.
°
Solution. Assume that there exists M > such that 1f'(x)l::::; M. Then applying
the Lagrange theorem, for every pair of points Xl> X2 E (a, b) we can write
interval (a, b). Then there exists eE ( a, b) such that for every x E ( a, b) the following
Taylor's formula holds
(X a)2 (x a)3
f(x) = f(a) + (x - a)f'(a) + 2! f"(a) + 3! f"'(a)
f(x)=Pn(x)+o((x-at), x-ta.
For a = 0 we obtain Maclaurin's formula
2 3
f(x) = f(O) + xf'(O) + ~! J"(x) + ~! f"'(O)
x n+ 1
+ ... + -
xn
n!
f(n)(o) +
(n+l)!
f(n+1)(e), eE (a ' b).
The polynomial
x2 xn
Pn(x) = f(O) + xf'(O) + 1"1"(0)
2.
+ ... + ,.
n.
f(n)(o),
f(x) = x4 - 5x 3 - 3x 2 + 7x + 6
~nto powers of x - 2.
254 CHAPTER 6.
it follows
+ 18(x-2)3 (X-2)4
31 +24---'-
. 4!
Thus
X4 - 5x 3 - 3x 2 + 7x +6 -16 - 33(x - 2) - 9(x - 2)2
+ 3(x-2)3+(x-2)4.
In this case the remainder is equal to zero, because the fifth derivative of f is
identically equal to zero.
Example 6.57. Approximate the function f(x) = eX using the Maclaurin's poly-
nomial of the
x2 3
x €
b) eX=I+x+T+o(x2), x~O and R2 = 3Te .
x2 x3 X4 e
c) eX = 1 + x + -2 + I" + o( x 3 ), X ~ 0 and R3 - -4!e ,
3. -
Example 6.58. Apply the Maclaurin's formula to the functions given below.
1
d) f(x) = 1 _ x; e) f(x) = In(I - x); f) (1 + x )'\ a E Q.
DERIVATIVES 255
e) In(l - x) = -
n xk
Lk
k=l
+ o(xn); f) (1 + x)" = 'to (~)xk + o(xn).
In f), 0: E R \ {O} and by definition
(0:) _ (0:) _ 0:(0: - 1) ... (0: - (k -
o - 1, k - k!
1))
' kEN.
Example 6.59. Find Maclaurin's formula for the functions given below.
+ (_1)x)-1/2,
vr=x
_1_ = (1
hence
-1= = E( _l)k
n (
- 1/2)
k xk + o(xn), X -t 0, where for kEN
(
-1/2)= (-~).(-~-1) ... (-~-(k-1)) =(_ )d 2k - 1)!!
k k! 1 2kk!·
Thus
1 ~(2k-1)!! k n
--= = 1
1- X
+ k=l
L...J
2
kk'
.
x + o(x), x -t O.
256 CHAPTER 6.
n 22k+lX2k+l (rr )
sin(2x+rr/3)=E(-1)k (2k+1)! sin 6(2+3k) +o(x2n+2), x---+O.
n (_l)kxk
In(x + e) = 1 + E ~J..l + o(xn), X ---+ O.
k=l
2 - 3x
e) f( x) = In 3 + 2x ; f) f(x) = In(6 + llx + 6x 2 + x 3);
1 - 2X2 x2 + 2
g) f(X)=2+ x-x 2; h) f(x)=x 3 +x2 +x+l
Solutions.
n- 2 3kxk n 3kxk
f(x) x 2 E ~ + o(xn) + 5 E ~ + o(xn)
5+15x+ E (k-2)!
n (3k-2 3k)
k! +5 xk+o(xn)
n (3k-2 )
5+15x+E ~(k(k-l)+5.9) xk+o(xn), x---+O.
DERIVATIVES 257
n (_2)k-2Xk n (_l)kxk n
{; (k-2)! +3-3x+3{; 7.' +O(X)
n ( l)kxk
3 - 3x + L(3 + k(k - 1)2 k- 2) - 71 + o(xn), X - t O.
k=2
c) Using Example 6.58 f) for 0: = 1/2, we get
V
~ x
1- x = 1- - - x
L
n (2k - 3)!! k
+ o(x n ) X -t O.
2 k=2 2kk! '
Hence
x2 (2k - 5)!! k
+ 2)yt'f=X
n n
(x x- 2 - L n
ol-_l 1 \,x + o(x )
k=3
n (2k - 3)!! k n
+ 2-x-2L kk' x +o(x )
k=2 2 .
3x 2 ~ 6(2k - 5)!!(k - 1) xk
2 -4- -L...J 2kk!
+ o(xn), X - t O.
k=3
d) Let us consider the function h(x) = Ix15. It has only four derivatives on Rand
it holds
h(O) = h'(O) = h"(O) = h"'(O) = h(4)(0) = O.
Now the function h has a fifth derivative on the set R \ {OJ, so we can write
h(x) = P4(X) + o(x 4) = O(X4), X - t 0,
since P4(X) == o. Thus we have
x2 X4
cos X + Ixl 5 = 1 - - + , + o(X4),
2 4.
X - t O.
e) Since
2 - 3x
In--=ln2-ln3+ln ( 1 -3X)
- -In ( 1 +2X)
-
3 + 2x 2 3 '
it follows from Example 6.58 e) that
n 3kxk n (_2)kxk
f(x) = In2-ln3-L~k +o(xn)+L k +o(x n )
k=l 2 k=l 3 k
n (( 4)k gk)xk
In2-ln3+L - l-~. +o(xn), x-tO.
k=l 6
258 CHAPTER 6.
f) From
In(6 + llx + 6x 2 + X3) = In(3 + x) + In(2 + x) + In(l + x),
it follows
n (_l)k-lXk n (_l)k-lXk n (_l)k-lXk n
f(x) = ln6+ L 3k k + L 2kk + L 1. +O(X)
k=l k=l k=l
n ( l)k-lXk
ln6 + L - C!,.L (2k + 3k + 6k ) + O(Xn), x -+ O.
k=l
g) From
1 - 2x2 -7 -1
--- - 2+ + --,---,-
2+x-x - 2 3(2-x) 3(1+x)'
it follows for x -+ 0
7 n xk 1 n 1 n ((_2)k+l_7)x k
f(x)=2--'Lk"--'L(-x)k+ o(x n )=-+L Ie ... , +o(x n ).
6 k=O 2 3 k=O 2 k=l 3.2
h) In this case we have
x2 + 2 1 ( 1- x 3)
f(x) = x3+x2+x+l ="2 1+x2+1+x
b) Since it holds
sin 2 x cos 2 x = ~(1 - cos 4x),
we have
n (_1)k-124k-3X2k
f(x) = L ("'\I + o(x2n+1), X ----t O.
k=l
c) From
. 1
sm 4 x + cos 4 X = 4(3 + cos 4x),
we have
n (_1)k24k-2x2k
f( x) = 1 + '"
L...,.; (01_\1
+ o(x2n+l) ,
X ----t O.
k=l
d) Since
1
L( _1)kx2k + o(x2n+1),
n
(arctan x)' = --2 =
l+x k=O
it follows
n x2k+1
arctan x = L( _l)k _ _ + o(x2n+2), X ----t O.
k=O 2k + 1
e) Since
(arcsin x)' = 1
v'f=X2 = 1+~
~ (2k2kk!
- I)!! x 2k + o(x2n+1),
it follows
.
arcsm x = x +L
n(2k-1)11.. x 2k +1 + o(x2n+2)
k=l 2kk!(2k + 1) .
eX _ e- X
f) From sinh x = " ,we obtain
n x2k+1
sinhx=L( k )1+ o(X 2n +2), x----tO.
k=O 2 + 1 .
eX + e- x
g) From cosh x = " ,we obtain
n 2k
cosh x = L (xk)1 + o(x2n+1), X ----t O.
k=O 2 .
e 3x _ eX + e- x - e- 3x 1 1
h) Since sinh x . cosh 2x = , = 2" sinh 3x - 2" sinh x, it follows
1 (n 32k+1x 2k +1 n x2k+1)
sinh x . cosh2x = - L
(k
2 k=O 2 + 1.
)1 - (k )1
k=O 2 + 1 .
L + o(x2n+2)
n x2k+1
2k
~ 2(2k + I)! (3 +1 - 1)
'" + 0 (2n+2)
x , x ----t O.
260 CHAPTER 6.
3x 2 2
b) lim x +2- - I---;; + exp( arctan x) +1
x-+o 1- x
I n - +2x
l+x
c)
. cosh(sin x)
hm
x-+o
-11 _
x
--
2
3x
5
2 2
tanh x - x
Solutions.
a) In this case we shall consider Maclaurin's formulas for the corresponding func-
tions. Since
t t2 3) x
t3 x3
V ~+ t
1 T L
= 1+---+-+0(x
2 8 16 ' --+ 0, t = tanx = x+ 3 +0(x3), x --+ 0,
we get
x2 5x 3
l+x-"2+T+o(x3), x--+O.
x2 x3
eX = 1 + x + "2 + ""6 + 0(x 3),
x3
sin x = x -""6 + 0(x3), X --+ 0
x3
arcsin x = x + ""6 + 0(x 3), X --+ 0, hence
3
arcsinx-sinx=~ +0(x 3), x--+O,
we obtain
4 2 3
. 2Vl + 2 tan x - 2ex + 2X2 . -3 x + o(x )
hm . . = hm = 4.
arCSIn x - sIn x x-+o 1 3 (3)
x-+o
3"x +0 x
DERIVATNES 261
x ) 2 2
+ 2x +""6
x
+ o(x 3 ) = 1 + x + 2x -""6
x
3 3 3
exp(arctanx) = 1+ (x -"3 + o(x 3 ),
2 3
1- x
In - - = -2x - -x + o(x 3)
1+x 3 '
2
- - = 2 + 2x + 2X2 + 2x 3 + o(x 3 ),
I-x
we obtain
2
x + -3x - -2- + exp(arctanx) + 1 13
__ x 3 + o(x3 )
lim 2 1- x = lim 6 = 13/4.
2 3
x-+o 1 1- x
n -- + 2x x-+o --x + 0 (3)
x
l+x 3
c) From the relations
x2 X4
1+- - -8 + o(x 4) X ~ o·,
2 '
(1 _~2r/5 x2
1 ___
10
4
50 =-- + (x 4) ,0 x ~ 0;
x3
tanh x x-"3+0(x4) x~o,
we obtain
11m
·
cosh(sinx) -
tanh x
1- 2
- x
R
5 2
-
3x2
_
5 = lim 20~
x-+o __
x3
-21x4
__ + ox
( 4)
+ 0 (x 4)
= o.
_0
c . (1
) 1x-+o
1m
sin x arctan x
-
tan x arcsin x
1).
262 CHAPTER 6.
Solutions.
b) From
x2 5
sin x = x - x 3/6 + O(X4), cos (sin x) = 1 - "2 + 24 X4 + o(x 3), X -+ 0,
1 1
VI + 2x = 1 + x + o( x ), x2(Vl+2x-l) - X +0(x 3)'
3
x -+ 0,
we obtain
2) 1/(x (-/1+2x-1))
hm (COS(SlllX) + -
2 3 3
. . X . l/(x +o(x ))
x~o 2
= hm (1 + 5x 4/4! + O(X4))
x~o
= l.
I. (1
1m
x~osin x arctan x
-
1 ) I'
tan x arcsin x
= 1m
x~o
X4
X4
+ o( x 4 )
+ o( x 4) = 1
.
limf(x) = 0,
X'~c
limg(x) = 0.
x~c
limf(x)
x---+c
= CXl, limg(x)
x---+c
= CXl.
DERIVATNES 263
prom'd ed -
f'(x)
(-) h as a l"t
zmz as x approac h es c.
g' x
The statement of Theorem 6.64 remains true also if lim f'((x)) = CXJ, and then
x---+c g' x
. f(x)
hm -(-)
x---+c 9 X
= CXJ.
a) lim cos x + 3x - 1 · eX + e- x
b) 11m - 2 cos x.
. ,
x---+o 2x x---+O X sIn 2x
. In cos ax x a _ aX
c) hm 1 b' a, b i= 0;
x---+o n cos x
d) x-+a
lim
aX - aa
, a> 0, a i= 1;
e) hm
. (a + xr - aX
a> 0; f)
.
11m XX - X
x-+o x
0 ,
x---+l1n x - x +1
Solutions.
a) The functions f( x) = cos x + 3x - 1 and g( x) = 2x are differentiable in an
interval containing the point O. The given expression has the undetermined
o
form "-" when x 0 and
o --t
'
. cos x
11m + 3x - 1 l' - sin x
= 1m
+3 = /
3 2.
x---+o 2x x---+o 2
Clearly the conditions of the L'Hospital rule are fulfilled in the cases b)-f), and
therefore we have the following.
. eX + e- X - 2 cos x l' eX - e- X + 2 sin x
b ) hm = 1m -.---~--
x---+o x sin 2x x---+o sm 2x + 2x cos 2x
lim e X +e- x +2c.08X =4/4=1.
4 cos 2x - 4x sm2x
x---+o
264 CHAPTER 60
-asmax
0 In cos ax o cos ax _ 0 a sin ax 0cos bx
c ) 11m---- = hm bOb - hm
x->o In cos bx x->o - SIn x x->o b SIn bx COS ax 0
COS bx
ax
2 sIn
a - - - COS bx 0 a2
lim ax
x->o b2sin bx b2 0
- - 0cos ax
bx
x a - aX ax a- 1 - aX In a 1 -Ina
d) lim = lim - - - - -
x--+a aX - aa x--+a aX In a In a
x->O 2 x x->o 2x
a) 11m 3x2
0 + 2x - 2 b) 11m 21nx
-- b> 0;
0
x2 - 1
0
xb d) lim In(ln x)
c) lim -ax, b> 0, a E R;
x--++oo e x->+oo In x 0
Solutions. The expressions in this example have the undetermined form" 00", and
00
the conditions of the L'Hospital rule are fulfilled 0 So we have the followingo
a) lim 3x 2 + 2x - 2 = lim 6x + 2 = ~ = 3.
x->+oo x2- 1 x->+oo 2x 2
2
b) 0 -
11m 21n
- = 1m - - = 101m -=00
x 10 x 2
x->+oo xb x->+oo bX b- 1 x->+oo bx b
xb
c) Clearly, if a ::::; 0, then lim - ax = +000
x--++oo e
Let us assume that a > 00 Then we can find kEN such that k > band
applying the L'Hospital rule k-times we obtain
k
11m x
0 k!
11m --=00
0
xb xk
Since 0 < -eax -< - ,
e ax
x > 0, using Exercise 4.31 it follows that
xb
lim - =
x-++oo e ax
o.
1
d) lim In(lnx) = lim xlnx - lim _1_ = o.
x-->+ooIn x x-->+oo1 In x
x-->+oo
x
Example 6.67. Determine
a) lim x( elfx - 1); b) lim sin x In cot x;
x---++oo x-->o+
Solutions.
-1
e l/x - 1 el / X (_)
a) lim x(e l / x - 1) = x---++oo
lim --1 lim x2 lim el / x = l.
x---++oo x-++oo -1 X-++<XJ
x x2
-1
. SIn x
· Slnx
. 1ncot x = 1·1m ---
In cot x t
lim - - - = o.
2
b) 11m lim co xsm x
x-->o+ - cos x
= x-->o+ cos 2 x
x-->O+ x-->O+
sinx sin 2 x
c) Let us find first for kEN the limit
lim x"'/k In
",-->0+
(~)
X
.
lim x"'/k
1)
In (- = lim
--1 x ~ lim x"'/k = o. (6.27)
x-->o+ X x-->o+ 0'. -",/k-l x-->o+
-IX 0'.
Thus also
lim
x-->o+
x'" Ink (~)
X
= o.
Choosing kEN such that k > (3, it holds
lim
x-->o+
x'" In f3 (~)
X
= o.
266 CHAPTER 6.
lim x ( , _ 2 ",,,m
d) "-+00 . () x ) _.' - 2
V x- ,. 1
1.
~+
- x-++oo
hm
.",,,,,
· (
.
x
JX2+T )
x
2X2
JX2+T - 2JX2+T 1
lim -2
x-++oo
J1 - 1 :;>+i
"' 1
x
2
+1 _ lim
"-+00
-2-
x'1+ 1
x'
~ 2.
x2
Example 6.68. Determine
a) lim (~ _ _1_) . b) lim (_1__ ~).
x-->o x 2 sin 2 x ' x-->o efC - 1 x
Solutions. In these two examples the undetermined forms "(Xl - (Xl" appear, so
firstly, we have to perform some transformations in order to obtain limits which can
be found by using the L'Hospital rule.
· (1
a) 11m - 2 - - -2 -
1) 1·
·2
sm x - x
= 1m ----,;,----
2
. 2
_r - 2 sm x
- x~ sin 2 x + 4x sin x cos x + X2(coS 2 X - sin 2 x)
-2 1
= lim - - - - - - - - - - - ; ; - - - - - - - - -
x-.o x2 3
1 + 4-.- cos x + -.-2-(COS 2 X
X
- sin 2 x)
sm x SIn x
. (1
b) hm - - - - = lim
x--->O eX - 1
1)
x
x-ex +l
x-.o xe X - x
I-ex
= lim - - - - -
x-.o xe X + eX - 1
_eX
= lim = -1/2.
X-'O xe X + 2e X
Example 6.69. Determine
a) lim(1 + 2X)I/x; b) lim x 1 /(I-x).
X-'O x--+ 1 '
e)
. 2
hm(-arccosx)I/X;
X-+O 7r
f) hm . Cl +
x--->o e
x)l/xf 1x
g) lim
X-.+=
(~arctan
7r
x) X ; h) lim(1
x--->O
+ tanhx)l/x.
DERIVATNES 267
2
limlny = lim In(I + 2x) = lim 1 + 2x = 2.
x->o x->O X x->o 1
Therefore we have
lim(I
x~o
+ 2x)1fx = exp (limln y) = e2 • x~o
lim exp(1n
x->1
u) = exp (lim
x->1
In u) = exp (lim _I_In x) .
x->1 1 - x
So we obtain
( lnx
lim - -
x->1 1 - X
= x->1
lim..L = -1
-1
1)
-
:::} lim x 1 /{I-x) _ -1
x->O - e .
SIn X)
lim(cosx)l/x2 = exp (lim lncosx) = exp ( lim -~ = e- 1 / 2
x->o X->O x2 x->o 2x
e) lim
x->o
(~ arccos x)
1["
l/x = exp (lim
x->o
_In- ,-(.:. :.~X_a_r_c_co_s_x-,),- )
= exp (lim (_ 1 . 1 )) = e-2/1f.
x->o arccos x v'f=X2
. ((1 +
f) hm
x->o
X)1fx)I/X
e
= exp
(.
hm - In
x->o X
1 ((1 + e
X)I/X))
=
. In(l + x) - x
exp (hm
x->o x2
) = exp ( 1)
+hm
.
=
x->O
~-1
2x
e- 1/ 2 .
268 CHAPTER 6.
g) lim
X-++OO
(~arctan
7r
x) x = exp ( lim
X-++OO
xIn (~ arctan x))
7r
exp
r
( x-!~oo ~ . arct
1
1
an x )
= e
-2/7r
.
x2
1 1
h) lim(1
x-+o
+ tanhx)l/x = exp ( lim 1 + tanh x .
X-+O 1
~) -- e.
. _x 2 e x
hm - - -
. -(2xe + xe
hm
X 2 x)
=0.
X-+O+ eX - 1 X-+O+ eX
Therefore we have
lim (eX _ I)X lim In y = e = 1.
X-+O+ = e x-+O 0
d) From
1 1
Iim In( arcsin x) =lm
l' arc~ . ~ 0
=,
X->O+ cot x X->O+ 1
- sin 2 x
we obtain lim (arcsinx)tanx = l.
X->O+
a) lim Xl/x;
X~+OO
b) lim (3x 2
x.....,. +00
+3 X )*.
a) From
Inx 1
lim - - = lim - = 0,
x->+oo X x->+oo X
3 . 1
2x - sinx x sm-
a) lim .; b)lim . 2 X.
x->+oo 2x + SIn x x->O SIn x
Solutions.
hence
·
11m 2x - sin x 1
= .
2x + sin x
x---++oo
In this case we can not apply L'HospitaI's rule, because the following limit
·
11m (2x-sinx), l' 2-cosx
. = 1m ,
(2x + sm x)'
x---++oo x---++oo 2 + cos x
· f'(x)
11m d .
- (-) oes not eXIst,
x_o g' X
so we can not apply L'Hospital's rule. Still, we can calculate the limit:
3' 1 . 1
x SIn - x SIn-
lim . x = lim -._x_ = O.
x_o SIn 2 x x_O SIn 2 x
x2
• If f'(x) > 0 for all x E (a, b), then f is increasing on [a, b].
• If f'(x) <0 for all x E (a, b), then f is decreasing on [a, b].
Theorem 6.74. Let f be a function that is continuous on a closed interval [a, b] and
differentiable on the open interval ( a, b).
• If f is decreasing on [a, b], then f'(x) ::; 0 for all x E (a, b).
Sufficient conditions for extrema are given in the following two statements.
• If f'(x) > 0, for x E (a, c) and f'(x) < 0 for x E (c, b), then f(c) is a local
maximum of f.
DERIVATNES 271
• If f'(x) < 0, for x E (a, c) and f'(x) > 0 for x E (c, b) then f(c) is a local
minimum of f.
• If either f'(x) < 0, or f'(x) > 0 for all E (a, b), except possibly in c, then f(c)
is not a local extremum of a function f.
Theorem 6.79. Assume a function f has at the point c all derivatives up to the
order n > 2, and it holds
f(x) = (x + 5?{lx - 4, x E R.
Clearly we see that f'(x) = 0 for x = -5 and x = 19/7. Also the first derivative
does not exist for x = 4, hence the given function has three critical numbers,
namely Xl = -5, X2 = 19/7, X3 = 4.
Example 6.81. Determine the local extrema of the following functions and the in-
tervals on which they are monotonically increasing or decreasing.
x x
a) f(x) = x 2 -6x+16' xER; b) f(x) = "3 - {/X, x E R;
Solutions.
16 - x 2
a) From f'(x) = ( it follows that f'(x) = 0 for Xl = 4 and
X - 6x + 16
2 )2'
X2 = -4, and the critical points are A(4, 1/2), B( -4, -1/19). Using Theorem
6.78, from
it follows that the function 1 has a maximum at the point A, and it has a
minimum at B. Further on, we have
f'(x) ~
3
_ ~X-2/3 = ~
3 3
(1 __1_)
X 2/ 3
X 2/ 3 - 1 X2 - 1
3X 2 / 3 3X 2 / 3( X 4 / 3 + X 2 / 3 + 1) .
c) From
!,(x) = 8(x 2 - 2) xi- 0,
3X I / 3 '
and at X2.
From the sign of f' it follows that 1 has a local maximum at the point X3 = 0,
even though the first derivative does not exist at o.
Thus we obtain
DERIVATIVES 273
1 1 +x . . .
f) From f'(x) = 1 + - = --, It follows that the functlOn has no extrema pomts.
x x
The function 1 increases on its whole domain, i.e., on (0, +00).
but
1(4)(X) = cosh x + cos x, 1(4)(0) = 21=0.
This means that the given function has an extremum point at x = O. Since
1(4)(0) > 0, at the point x = 0 1 has a minimum.
274 CHAPTER 6.
Example 6.83. Examine the monotonicity and find the extrema points for the func-
tion y = f( x) given parametrically by
_ t3 t 3 - 2t2
X -
t2-+-
1' Y= t2 + l ' t E R.
Solution. From
t 2 (t 2 +3) , t(t-1)(t2+t+4)
'- ----,-
x - (t2+1)2
t
and
Yt= (t 2 + 1)2
, =
Yx
y~
x~
= (t - 1) (t 2 + t
t(t2+3)
+ 4)
,
t ..J.
r·
°
Thus it follows that y~ = 0 for t = 1, i.e., for x = 1/2. In fact the function f has
°
two critical points. Namely, Xl = is obtained from t = 0, and the other, X2 = 1/2,
from t = 1.
• If t < 0, then
X E (-00,0).
X < 0, and from y~ > ° it follows that the function increases for
°
• If t E (0,1), then x E (0,1/2), and from y~ < it follows that the function
decreases for x E (0,1/2). Therefore, at the point Xl = the function has a
maXImum.
°
• Ift E (1, +(0), then X E (1/2, +(0) and from y~ > it follows that the function °
increases for x E (1/2, +(0). Therefore at the point x = 1/2 the function has
a ffilnImum.
f(x) ={
e- I / X2 , xi 0; g(x) = {
xe- I / X2 xi 0;
0, x = 0, 0, x = 0.
Prove the following.
a) f(n)(o) = g(n)(o) = 0.
DERIVATIVES 275
Solutions.
a) It is clear that
a) fn = vn
I "1 (\I'\,.. ; b) fn =
n2
., ..... nn;
_n
12
I" _,
C
)
In - en d) fn = y'n.
Solutions.
a) The function
-"fX ., 1995 - x
f(x) = x + 1995 wIth f (x) = 2-"fX(x + 1995)2
has its maximum at the point x = 1995, because f'(x) > 0 for x < 1995
(the function is increasing), and f'(x) < 0, for x > 1995 (the function is
decreasing). So n=1995.
2
b) The function f(x) = x 3 : 200 has a critical point at x = ?'400 ~ 7.37. Since
f(7) ~ 9.02.10- 2 > f(8) ~ 8.99.10- 2 , it follows that n = 7.
X 12 12xll _ X 12
c) From f(x) = - , f'(x) = , it follows that n = 12.
eX eX
276 CHAPTER 6.
Solution. The functions f and <jJ satisfy the conditions of the Cauchy theorem 6.50,
so we can write
f(X)-f(x o)I=If'(e)I<I, eE(xo,x).
I<jJ(x) - <jJ(xo) <jJ'W-
Since the function <jJ is increasing we obtain
If(x) - f(xo)1 :::; 1<jJ(x) - <jJ(xo) I = <jJ(x) - <jJ(xo) for x;::: Xo·
Example 6.87. If
Similarly we obtain
f(n-2)(X) > g(n-2)(x) for x> Xo,
and continuing this procedure finally we get
x2 x
a) x - 2 < In(l + x) < x, x> 0; b) In(1 + x) > --1'
x+
x> 0;
c) x a - 1 > a(x - 1), a> 2, x> 1; d) \IX - \fO, < -\Ix - a, n> 1, x> a > O.
Solutions.
x2
a) Let us denote by f(x) = x - 2' g(x) = In(1 + x) and h(x) = x. It holds that
f(O) = g(O) = h(O) = 0 and
x2
X - 2 < In(l + x) < x, x> O.
x
b) In this case we denote f (x) In(1 + x) and g(x) Then from
x+l
f(O) = g(O) = 0 and
1 , 1
j'(x) = - - > 9 (x) = ~ \~,
l+x
we get
. x
f(x) > g(x), l.e., In(l + x) > - -
x+l
for x> o.
c) Denoting by f(x) = x a - 1 and g(x) = a(x - 1), in view of f(l) = 0 = g(l) and
d) If we take f (x) = \IX - \fO, and g( x) = -\Ix - a, then this functions satisfy the
conditions of Example 6.87 and the given inequality follows.
x2
a) cosx2::1-2, xER; b) arctan x :S x, x 2:: 0;
2 .
c) -x < smx,
7r
0< X < 2;
7r
d) sinx + tanx > 2x, 0< X
7r
< 2.
Solutions.
278 CHAPTER 6.
x2
a) Taking f(x) = cosx, g(x) = 1- 2' we have f(O) = g(O) = 1 and f'(x) =
- sin x, g'( x) = -x. The function h( x) = x - sin x was considered in Example
6.81 d). Since this function is a monotonically increasing one and h(O) = 0, it
holds
x - sin x > 0, x> 0, i.e., x > sin x, x> 0.
From f'(x) > g'(x), x> 0, it follows that
x2
f(x) > g(x), x> 0, i.e., cos x > 1 - 2' x> 0.
The functions f and 9 are even, which means that the last inequality holds
for x < 0, also.
. 1
b) Puttmg f(x) = x - arctan x, x 2 0, we get f'(x) = 1 - 1 + x 2 > 0, hence
f(x) 2 f(O) for x 2 0, which implies the given inequality.
f'(X3) = f'(X4) = 0. But the equation f'(x) = 0, i.e., cos x - ~7r = 0, has only
one solution on the interval (0, %). So the function f is of the same sign on
this interval. Since it holds (for instance)
f (~)
3 = V3 2 > 0,
2 -"3
d) The functions f(x) = sinx + tan x and g(x) = 2x satisfy the conditions of the
Example 6.87, since f(O) = g(O) = 0, f'(0) = 2 = g'(O), and
6.6 Concavity
6.6.1 Basic notions
Definition 6.90.
Theorem 6.91. If a function f is two times differentiable on an open interval (a, b),
then f is
A necessary condition for the existence of the point of inflection gives the follow-
ing theorem.
Sufficient conditions for the existence of the point of inflection give the following
two theorems.
Theorem 6.95. Let a function f be differentiable at a point c, and two times dif-
ferentiable on an open interval (a, b) containing c, except possibly at c. Then c a
point of inflection of f, if one of the following assumptions holds:
280 CHAPTER 6.
Solutions.
a) From f"(x) = 12(x 2 - 1), it follows that J"(x) > 0 for x E (-00, -1) U(I, +00),
where the function f is concave upward, and for x E (-1, 1) the function is
concave downward.
The points A(I, 3), B( -1, -7) are the points of inflection, because f"(x) = 0,
for Xl,2 = ±1 and from f"'(x) = 24x, it follows that f"'(x) i- 0, for x = ±l.
b) The function f is defined on the interval (0, +00) and can be written as
X -1
f(x) = 5. { xvx' x ~ 1;
I-x
xvx' 0 < x < 1,
~~
x> 1;
x3VX'
f"(x) {
5-x
X3VX' °< x < 1,
and it holds 1"(5) = 0, and
Therefore, on the intervals (0,1) and (5, +=) the function is concave up-
ward and on the interval (1,5) the function is concave downward. The point
A(5, 4/0) is a point of inflection, but the point B(I, 0) is not, because the
function f has no first derivative at x = 1.
e) In this case we have
or
~ 5~
exp(4" + 2h) < x < exp( 4 + 2h).
The inflection points are
Ak
_
-
(~
exp (4"+2h),
exp (~+ 2h))
V2 ' k=0,±1,±2,... ,
we obtain
y~ = cos t(2 sin 2 t + 1), (y~)~ = 3 cos 2t sin t,
and finally
Y:x = -3 sin 3 t cos 2t.
Therefore Y:x = 0
for t = ~, and t = 3:. Since
• Y;x < 0 for t E (0, ~), the function is concave downward there;
• Y;x < 0 for t E (3: ,7r), the function is concave downward there,
it follows that the points t = i and t = 3;, or A( 4,0) and B(2, 0), respectively, are
the points of inflection for the function y = y(x).
Example 6.99. Prove that if f"(x) > 0 for all x E (a, b), then f is concave upward
on (a,b). I.e., prove then that for every pair Xl,X2 E (a,b) and arbitrary numbers
0:1, 0:2 such that
0:1 > 0, 0:2 > 0, 0:1 + 0:2 = 1 (6.28)
the following inequality holds
Solution. Let us suppose that 1"( x) > 0 for all x E (a, b) and let the arbitrary
numbers 0:1> 0:2, satisfy the conditions given by relation (6.28). If Xl, X2 belong to
the interval (a, b), and Xl < X2, then O:lXl + 0:2X2 E (a, b) also. Using Lagrange's
theorem we obtain
where Xl < 6 < alxl + a2X2, and alxl + a2x2 < 6 < X2. Multiplying the last two
equalities with a!, a2 respectively, we get
The function f has a second derivative on the interval (a, b) so we can apply La-
grange's theorem again, giving us
From the last relation, in view of f"(x) > 0, follows relation (6.29).
Example 6.100. Prove the following inequalities.
a)
an+b
...
n> (a+b)n
-2- , a > 0, b > 0, a i- b, n E N;
for the concave upward function on the interval (a, b), and
are concave upward (J"(x) > 0, for X in their domains), relation (6.30) holds.
284 CHAPTER 6.
Example 6.101. If f is a concave upward function on the interval (a, b) and the
points Xl, X2, ... , Xn , belong to (a, b) and all a2, ... , an are nonnegative numbers such
that al + a2 + ... + an = 1, then the so-called Iensen's inequality holds:
f(alXI + ... + anxn) ::; ad(xd + ... + anf(x n ). (6.32)
Prove.
Solution. If n = 2, then we have inequality (6.29). In this relation we have equality
if one of al or a2 is equal to zero. If n > 2 then we apply mathematical induction.
.
We can denote by j3 = a2 + ... + an > and It holds
a2
°
+ ... +
'a n
= 1. /i 73
Let us suppose that
a2
/i an )
f( X2 + ... + 73 xn ::;
a2 ( )
f X2 + ... +
an
7i
f(xn). 73
Then we have
Xl + ... + Xn .
b) ylXI ... Xn ::; , Xi::::: 0, z = 1, ... , n.
n
Solutions.
a) The function f(x) = lnx is concave downward on (0,+00) and for this function
it holds
f(alXI + ... + anxn) ::::: ad(xd + ... + anf(xn). (6.33)
Therefore we can write
alln(xd + ... + an In(x n ) ::; In(alxl + ... + anxn). (6.34)
Graphs of functions
:x:
285
286 CHAPTER 7.
- Since 1"( -2) = -54 < 0, the critical point B( -2,0) is a maximum.
- From 1"( -0.8) > 0, it follows that the function f has a minimum at the
point C (-0.8, -8.40).
Let us remark that the first derivative does not change its sign at x = 1,
and though it holds 1'(1) = 0, it has no extremum at x = 1.
- The function f is concave upward if 1"(x) = 2(x -1 )(10x2 + 16x + 1) > 0,
i.e., for x E (xs,x4)U(1,+00).
- The function f is concave downward if 1" (x) = 2( x-1)( 10x 2 + 16x + 1) <
0, i.e., for x E (-00, xs) U(X4, 1).
- From the previous conclusions it follows that the function f has three
points of inflection A(l,O) D(x4,f(x4)) E(xs,f(xs)). It holds f(X4) ~
-4.56 and f(xs) ~ -3.58.
f(x) = x+1.
x2 +1
Solution. The graph of f is sketched in Figure 7.2.
A ( -1 + V2, V2+1)
2 and B ( -1 - V2, - V2-1)
2 .
- Since 1"( -1 + V2) < 0, the function has a maximum at the critical point
A ( -1 + V2, V22+ 1) .
1" (-1 - V2) > °the function has a minimum at the critical point
1)
- Since
• Asymptotes.
iY
A
E B ·1 10 1
x+1
Fig. 7.2. f(x) = x2 +1
Example 7.3. Determine and sketch the graph of the function
x-5
f(x) = x2 - 9·
• The domain of the given function is (-00, -3) U (-3,3) U (3, +00).
- Since 1"(1)> 0, the function has a minimum at the critical point A(l, 1/2).
- Since 1"(9) < 0, the function has a maximum at the critical point
B(9,1/18).
- In this case we can not order directly the zeros of the numerator of I" (x).
From 1"(13) < 0 and 1"(14) > 0 it follows that for some Xo E (13,14) it
holds I"(xo) = O.
GRAPHS OF FUNCTIONS 289
because
• Asymptotes.
x -5 lim x-5
lim - - - - -00·
x ..... -3- x2 - 9 - , x ..... -3+ x2 _ 9 = +00;
x-5 .
11m x - 5
lim - - - = +00; x2. _-9 = -00.
x ..... 3+
x2 - 9
x .....3-
•
";I
II y
,
B
x
x-5
Fig. 7.3. f(x) = x 2 -9 Fig. 7.4. f(x) = xv'x+3
290 CHAPTER 7.
f(x) = xVx+3.
Solution. The graph of f is sketched in Figure 7.4.
- The zeros of the first derivative are XlV3 and X2 = -V3. The critical
=
. of f are A
pomts (V3)
V3'..y2 , B ( -V3, -..y2
V3) .
- The function f is increasing for X E (-00, -V3) U( V3, +00).
- The function f is decreasing for X E (-V3, -1) U( -1,1) U(I, V3).
f"(x)= 2x(9-x 2 )
9~(X2 - IF'
• Asymptotes.
x
lim f(x)
x-+-l-
= x-+-I-
lim 3~
'\ X
= -00, lim
x .... -l+
x
31"::21
yx- - 1 = +00 '
x = -00, .
11m x
lim 3~
x-+l- V X - - l x .... l+ V'x2 _ 1 = +00.
x
·
11m -
x-++oo
- --
f(x)
x
1·1m ~ --
x-++oo x
° .
292 CHAPTER 7.
L
'31
"IL
";'
~ 31
II'
,,
II'
(a=2) ,
"
I
: A
I _------
,
... :.--~
"
--i_iS
:.,.....:~.)",
x
Fig. 7.5. f(x) = \fx2 _ 1 Fig. 7.6. f(x) = IT
V;-=-;;:
Example 7.6. Determine and sketch the graph of the function
f(x) =
fx3 ,
V;-=-;;: a> O.
f(x)= [1;
-=Ixl' ~
3
x-a
-, x-a
hence x(x-a»OVx=O.
A(3a2' 3a2v'3).
- The function f is increasing for x E (32a, +=).
~ 3a 2 x 3a 2
f"(x) = V~· 4(x - ap = 4Jx(x _ a)S'
A (3a 3v'3a )
2' 2 .
• Asymptotes.
lim
x->a+
g -- =
X -
3
a
+00, lim f(x) = lim
x---+o- x---+o-
J x3
X - a
= 0.
lim (f(x) - x)
x->+=
= x->+=
lim (J x
X -
3
a
- x) ~,2
lim
x->-=
(IT +x)
V~
= -~.2
Example 7.7. Determine and sketch the graph of the function
because the first derivative has two zeros XI,2 = ~, but is not defined
at the points X3 = 0, X4 = -1 and X5 = 1.
- The function f is increasing for x E (- ~,O)U(~,oo).
- The function f is decreasing for x E (-00, - ~) U(O, ~).
- The function has minimums at the points
Note that the tangent lines through the points A and E are parallel with the
y-aXIS .
f"(x) = ~
9
(_1_ _{j(x3+2 _x2
~ 1)5
)
.
lim f"(x) = .
bm -2 ( - 3
1- -
3 + x 2 ) = +00,
x-+-H x-+-H 9 ~ {j(x2 -1)5
lim f"(x)
x~-l-
= -00, lim f"(x)
x~l-
= +00 and lim f"(x)
x--+l+
= -00,
it follows that the function changes its concavity for x = 1 and x = -1.
GRAPHS OF FUNCTIONS 295
• Asymptotes.
- The function has no vertical asymptote.
- It has a horizontal asymptote y = °both when x -t +00 and x -t -00,
because
-1 = 0.
xl!~)V'X2 -1 - V9") = ",l!~oo {j(x2 -1)2 + {jX2(X2 -1) + W
- There is no slanted asymptote.
')I
1
ty
1
:t
D B
:Ie
B -2 01 1
12-xl .
f(x) = 3vx2 + 4
Solution. The graph of f is sketched in Figure 7.8. The given function can be
considered as the following two functions
2-x
fl(X) = 3y'X2TI' for x ~ 2,
x-2
f2(X) = 3.JX2+4' for x> 2.
Let us remark that it holds h(2) = lim h(x) and therefore this is a continuous
"' ..... 2+
function on R.
• The function fl(X) has a zero at the point A(2,O), while the function h(x)
has no zeros.
296 CHAPTER 7.
t l ,2 = 1 ± V3
2
The value tl =
1+V3
2
.
> 1 can not be equal to sm x for any x E R, and
1- V3
therefore we are taking t2 = 2 .
Thus the zeros of f are Xl ~ 3.42, and X2 ~ 5.91.
cos x = ° or 1 - 2 sin x = 0.
'y
l"Y
Solution. The graph of f is sketched in Figure 7.10. The function can be written
as
f() cos 2x - cos 4x . 2 ( • 2 )
X = 2 = sm x 3 - 4 sm x .
. 3
sm x + cos3 x = ( sm
.
x + cos x ) ( sin2 -
1- - 2X) = 0,
. 77r
whose solutIOns are Xl = -37r and X2 = -.
4 4
• The first derivative is
- The points A(O,I), B (%,1) and C C:, -~) are local maximums.
. 7r J2 37r ..
- Thepomts D(4"'T)' E(7r,-I), and F(2,-I)arelocalmlmmums.
• The second derivative ofthe function is 1" (x) = ~(sin x + cos x ) (sin 2x - ~) .
• The inflection points are for
and also for X4:::::J 1.21, Xs:::::J 3.51 and X6:::::J 4.35.
'Y
- ...B'_-- A'... - .,
,~
I.!
E F
f(x) = xe- x / a , a i= 0.
Solution. The graph of f is sketched in Figure 7.12, both for a =2 and a = -2.
- If a> 0, then the function is increasing in (-00, a) and if a < 0, then the
function is increasing in (a, +00 ).
If a > 0, then the function is decreasing in (a, +(0) and if a < 0, then
the function is decreasing in (-00, a).
• Asymptotes.
The function has a horizontal asymptote y = 0 when x -+ += if a > 0
(resp. when x -+ -= if a < 0), because
f(x) = (1 - x2)e- 2x .
Solution. The graph of f is sketched in Figure 7.13.
A (1 - (-1 +
v's
2'
v's)e-(l-Vs))
2
and B (1 + (-1 -
2
v's, v's)e-(HVs))
2 .
.
- Th e functwn . for
f"IS IncreasIng X E ( -=, 1-2v's) U (1 +2v's ,+=.)
- The function f is decreasing for X E(1 -2v's ' 1+2v's) .
• The second derivative is f"(x) = (2 + 8x - 4x2)e- 2x .
- Since f" (1 +2 v's) > 0, it follows that the minimum is at the point
B (1 + (-1 -
v's
2'
v's)e-(HVs))
2 .
- Since f" ( 1 -2 v's) < 0, it follows that the maximum is at the point
A (1 - v's (-1
2'
+ v's)e-(l-Vs))
2 .
302 CHAPTER 7.
- -3 + 2V6e ,
C (1 -V6
2 ' 2
-2+v'6) D (1 + ~, -3 ~ 2V6 e- 2 -v'6) .
- The function is concave upward for
Note that
lim (1 - x2)e- 2x = -00.
x-+-oo
'Y
A 1~!'~
",.
,
/""
"
:It
1 B D
f( x) = xe- 1jx2 .
• It has no zeros.
J"( x ) __ -2x 2
5
+ 4 e -1/x2
x
- The inflection points are
• Asymptotes.
-1/x 2 1
n = lim (xe- 1 / x2
x--->±oo
- x) = lim x(e- 1 / x2
x--->±oo
- 1) = lim e
x--->±oo
v1 = O.
x·-
x2
f "( x ) -_ + 2 e 1/ x
- 5x__
X4 •
Since
f"(2) = - ~: e1 / 2 < 0 and f"( -1) = 3e- 1 > 0,
. fectlOn
- Th em l· . .1S C (-S'
pomt 2 -Se
8 -5/2) .
- The function is concave downward for x E (-2/5,0) U (0, +00).
- The function is concave upward for x E (-00, -2/5).
• Asymptotes.
lim f(x)
X---+O+
= X--+O+
lim (_x_2)e 1 / x = -00; lim f(x)
x---+o-
= x-+o-
lim (_x_2)e 1 / x = O.
- Since
lim f(x)
x---++oo
= lim (-x - 2)el/x
x---++oo
= -00,
the function has no horizontal asymptotes.
GRAPHS OF FUNCTIONS 305
1 - e1 / x 1 - et
lim - 2 = lim - - - 2 = -3.
x-dO<) 1 t-+O± t
x
',:6 10 2 " 1
...U
,
,, II
,,
,,
I
_______________ J.
I
,, I
iJt
-I
Fig. 7.15. f(x) = -(x + 2)e1 / x Fig. 7.16. f(x) = e2x /(1-x 2 )
• Asymptotes.
- The vertical asymptotes are x = -1 and x = 1, because
lim-~2x/(1-x2) = +00, lim e 2x /(1-x 2)_
x--+-l- x ..... -l+ - 0,
f(x) = lnx
x
Solution. The graph of f is sketched in Figure 7.17.
• The domain of the function is (0, +00).
f '()
x = 1- 2
x
In x' hence f '( x ) = 0 for x = e.
• The function is concave upward for x E (e3/ 2, +00), and concave downward for
x E (0, e3 / 2 ).
GRAPHS OF FUNCTIONS 307
• Asymptotes.
· lnx
The vertica1 asymptote is x = 0, b ecause 11m - - = -00.
°when x
X-+O+ X
'Y
y
A B
o "
1
o ,... %
,II
II
f(x) = 1Vi
2 •
n x
Solution. The graph of f is sketched in Figure 7.18.
• The function is concave upward for x E (e- V24 ,l)U(l,e V24 ), and concave
downward for x E (0, e- V24 ) U( eV24 , +00).
• Asymptotes.
wherefrom we get
¢(x) >
x3
° for x E (-l,O)U(O,+oo).
The first addend in the second derivative is grater than zero, and since the
second is also grater then zero, it follows f"(x) > °
for every x from the
domain. This means that
GRAPHS OF FUNCTIONS 309
• Asymptotes.
- The vertical asymptote is x = -1, because
Remark. The point A(O, e) does not belong to the graph of f, since the point 0 is
't
not in its natural domain. However, f can be continuously extended to x = 0 if one
defines f(O) = e (see the Remark after Example 5.37).
,'Y
'Y A
..,'
i ' :
k:
,,,
: I
, x
,
: A
y-l: "--
-----~---------- --x
,0I 1 8
2x
Fig. 7.19. f(x) = (1 + X)l/X Fig. 7.20. f( x) = arcsin 1 + x 2
• The domain of the function is (-00, +00), because (x - 1)2 :::: 0 and thus
2x
-1 < - - < l.
- 1 +x2 -
f'(x) = 2sgn(1 - x 2 )
1 + x2 ,x =f. ±l.
310 CHAPTER 7.
f "( x ) = (1 + X 2 )2 sgn ( 1 - x 2) ,
-4x x ~ ±I
• The function is concave upward for x E (-1,0) U(I, +00) and concave down-
ward for x E (-00, -1) U(O, 1).
• Asymptotes.
There is no vertical asymptote.
The horizontal asymptote is y = 0 when x ---) ±oo, because
. . 2x
hm arCSIn -1--2
x--+±oo + X = O.
- There is no slanted asymptote.
2 1 - x2
- 1 < -1 + - -2 = -- < l. (7.1 )
1+x 1 + x2
GRAPHS OF FUNCTIONS 311
• The function has no inflection points, because it holds f"(x) < 0, for x =I- O.
• Asymptotes.
1 - x2
lim arccos - - - 2 =
1+x
7r.
x~±oo
y
y=1t y = 1t12
.2-' B
~
'.p
x x
o o
1 - x2
Fig. 7.21. f(x) = arccos 1 + x 2 Fig. 7.22. arctan eX - In ( J :2:2X)
1
312 CHAPTER 7.
1"(x) = eX ( _e 2x + 2ex + 1)
(1 + e2x )2
• It has a zero at XB = In(l + .J2), which is the point of inflection.
lim (arctan eX - In (.
x-->+oo V~) +
1 e 2;'- )
= ~.2
The slanted asymptote is y = -x when x -+ -00.
Bibliography
[2] DeLilo, N.J., Advanced calculus with applications, Macmillan Publ. Co., New
York 1982.
[4] Fihtengoljc, T.M., Course on differential and integral calculus, Vol. I, Nauka,
Moscow 1970 (in Russian).
[5] Fridy, J.A., Introductory analysis, Hartcourt Brace Jovanovich Publ., New York
1987.
[6] Gajic, Lj., S. Pilipovic, Collection of exercises from Analysis I, Second part,
University of Novi Sad, Institute of Mathematics, Novi Sad 1990 (in Serbo-
Croatian).
[7] Gaskill, H.S., Narayanaswami, P.P., Foundation of analysis, Harper and Row,
Pub., New York 1989.
[8] Krall, A.M., Applied analysis, D. Reidel Publ. Co., MA 1986.
[9] Kudriavcev, L.D., Course on mathematical analysis, Vol. I, Vis. skola, Moscow
1981 (in Russian).
[10] Kurepa, S., Mathematical analysis, Vol. 2, Tehnicka knjiga, Zagreb 1977 (in
Serbo-Croatian)
[12] Lasko, LL, Boyarcuk, A.K., Gai, Ya.G., Golovac, G.P., Handy textbook on
mathematical analysis, Vol. 1, Visa skola, Kiev 1978 (in Russian).
[13] Loomis, L. H., Sternberg, S., Advanced calculus, John and Bartlett Publ., MA
1990.
[14] Mardesic, S., Mathematical analysis in the n-dimensional space, Skolska knjiga,
Zagreb 1974 (in Serbo-Croatian).
313
314 BIBLIOGRAPHY
[15J Milicic, P.M., Uscumlic, M.P., Collection of exercises from higher mathematics,
Vol. I, Naucna knjiga, Belgrade 1988 (in Serbo-Croatian).
[16J Pownal, Malcolm, W., Real analysis - a first course with foundations, Wm. C.
Brown Publ., IO 1994.
[17J Protter, M.,H., Morrey C.,B., A first course in real analysis, Springer-Verlag,
New York Inc. 1977.
[19J Rudin, W., Principles of mathematical analysis, McGraw Hill, New York 1964.
[20J Stankovic, B., Elements of functional analysis, Naucna knjiga, Belgrade 1975
(in Serbo-Croatian).
[21 J Swokowski, E. W., Calculus with analytic geometry, Second edition, Prindle,
Weber and Schmidt, Boston, Massachusetts 1979.
[22J Zoric, V.A., Mathematical analysis, Nauka, Moscow 1981 (in Russian).
[23J TakaCi, Dj., TakaCi, A., Collection of exercises from Analysis I, First part,
University of Novi Sad, Institute of Mathematics, Novi Sad 1989 (in Serbo-
Croatian).
[24J TakaCi, Dj., TakaCi, A., Collection of exercises from differential and integral
calculus, University of Novi Sad, Institute of Mathematics, Novi Sad 1992 (in
Serbo-Croatian) .
Index
315
316 INDEX
in R, 37 at a point, 181
irrational, 29 on a set, 182
rational, 29 differentiable, 216
elementary, 48
Decreasing function, 270 graph of a, 46
Dedekind's theorem, 35 injective, 45
Derivative inverse, 46
first, 213 irrational, 75
higher order, 216 rational, 74
left-hand side, 213 surjective, 45
right-hand side, 213 tends to infinity, 143
second, 216 uniformly continuous, 204
third, 216 Function concave
Differentiable function, 216 downward, 48, 279
Differential, 216 upward, 48, 279
Dirichlet's function, 59, 185, 199 Fundamental theorem of algebra, 73
Discontinuity
first order, 182 Global
removable, 182 maximum, 47
second order, 182 minimum, 47
Distance, 3, 44 Graph
Distributive law of multiplication over perpendicular line of the, 216
addition, 1 tangent line of the, 216
Divisor of a number, 4 Graph of a function, 46
Domain Greatest integer part, 23
natural, 46 Group, 2
of a function, 45
Harmonic sequence, 100
Elementary function, 48 Hausdorff topological space, 39
Equality of functions, 45 Heine-Borel property, 39
Euler's constant, 110 Higher order derivative, 216
Even function, 47 Horner scheme, 74
Exponential function, 48
Extremum, 270 Iensen's inequality, 284
Implicit function, 215
Field first derivative of, 215
complete totally ordered, 2 Increasing function, 270
of rational numbers, 5 Increment
of real numbers, 1 of the dependent variable, 213
totally ordered, 2 of the independent variable, 213
Finite set, 38 Infimum of a set, 3, 24
First derivative, 213 Infinite set, 38
Function, 45, 46 Inflection point, 279
basic elementary, 48 Injection, 45
bijective, 46 Integer, 4
continuous even, 4
INDEX 317
odd, 4 Maclaurin's
Integers formula, 253
set of, 4 polynomial, 253
Interior point of a set, 38 Mathematical induction principle, 4
Interval Maximum of a function
bounded, 3 global, 47
closed, 3 local, 47
open, 3 Maximum of a set, 3
unbounded, 3 Mean value theorems, 244
unbounded to the left, 3 Metric space, 44
unbounded to the right, 3 Minimum of a function
Inverse global, 47
function, 46 local, 47
trigonometric function, 48 Minimum of a set, 3
Inverse function Monotone sequence, 102
first derivative of, 215 Monotonically
Irrational decreasing function, 46
cut, 29 decreasing sequence, 102
number, 4 increasing function, 46
Irrational function, 75 increasing sequence, 102
Isolated point of a set, 38 non decreasing function, 47
nonincreasing function, 47
L'Hospital's Rule, 263 Monotonicity of functions, 270
Lagrange Theorem, 244 Multiplicative
Laguerre's polynomial, 247 identity element, 1
Largest common divisor, 107 inverse, 1
Left-hand limit, 142
Natural domain of a function, 46
Limes
Natural number, 4
inferior of a sequence, 117
prime, 4
superior of a sequence, 117
Neighborhood of a point, 37
Limit
Number
left-hand , 142
irrational, 4
of a sequence, 83
natural, 4
right-hand, 142
rational, 4
Limit of a function
real, 1
at a point, 141
Number e, 104, 107
at minus infinity, 142 Numbers
at plus infinity, 142 set of natural, 4
Local
extremum, 270 Odd function, 47
maximum, 47, 270 One sided derivative, 213
minimum, 47, 270, 271 Open
Local extrema of functions, 270 interval, 3
Logarithmic function, 48 set, 37
Lower bound of a set, 3 Open covering, 38
318 INDEX
00/00,262
Uniform continuity, 204
Uniformly continuous function, 212
Upper bound of a set, 3
Variable
dependent, 45
independent, 45
Zero of a function, 46
Zero of a polynomial, 73
complex, 73
rational, 73
real, 73
Kluwer Texts in the Mathematical Sciences
1. A.A. Harms and D.R. Wyman: Mathematics and Physics of Neutron Radiography.
1986 ISBN 90-277-2191-2
2. H.A. Mavromatis: Exercises in Quantum Mechanics. A Collection of lllustrative
Problems and Their Solutions. 1987 ISBN 90-277-2288-9
3. V.1. Kukulin, V.M. Krasnopol'sky and J. Horacek: Theory of Resonances. Principles
and Applications. 1989 ISBN 90-277-2364-8
4. M. Anderson and Todd Feil: Lattice-Ordered Groups. An Introduction. 1988
ISBN 90-277-2643-4
5. J. Avery: Hyperspherical Harmonics. Applications in Quantum Theory. 1989
ISBN 0-7923-0165-X
6. H.A. Mavromatis: Exercises in Quantum Mechanics. A Collection of lllustrative
Problems and Their Solutions. Second Revised Edition. 1992 ISBN 0-7923-1557-X
7. G. Micula and P. Pavel: Differential and Integral Equations through Practical
Problems and Exercises. 1992 ISBN 0-7923-1890-0
8. W.S. Anglin: The Queen of Mathematics. An Introduction to Number Theory. 1995
ISBN 0-7923-3287-3
9. Y.G. Borisovich, N.M. Bliznyakov, T.N. Fomenko and Y.A. Izrailevich: Introduction
to Differential and Algebraic Topology. 1995 ISBN 0-7923-3499-X
10. J. Schmeelk, D. Takaci and A. Takaci: Elementary Analysis through Examples and
Exercises. 1995 ISBN 0-7923-3597-X